SURGERY Superexam TOPNOTCH

Réussis tes devoirs et examens dès maintenant avec Quizwiz!

What is the drug of choice for primary peritonitis? A. Penicillin B. Cefazolin C. Cefotaxime D. Metronidazole E. Clindamycin

200 C. Third-generation cephalosporins such as cefotaxime (2 g q8h, administered IV) provide reasonable initial coverage in peritonitis

Afferent loop syndrome may be a complication after which surgical procedure? A. Graham's patching B. Bilroth I C. Bilroth II D. Truncal vagotomy E. Jaboulay pyloroplasty

117 C. Page 26 of Topnotch Handout. Afferent loop syndrome is a complication usually following a Bilroth II (Gastrojejunostomy). This is characterized by acute or chronic intermittent obstruction of the afferent jejunal limb.

The amount of time kidneys for transplantation can be preserved is: A. 6-8hrs B. 16hrs C. 24hrs D. 36-40hrs E. 48-72hrs

1 D. 6-8hrs â€" heart and lungs 16hrs - liver 24hrs - pancreas 36-40hrs â€" kidneys Source: Schwartz’s Principles of Surgery 8th ed p 282

A 43 year old male incurred corneal scarring from eye herpes and lost vision of his left eye. A full thickness corneal transplant was done. Examples of full thickness keratoplasty include: A. Penetrating keratoplasty B. Descemet stripping with endothelial keratoplasty C. Descemet membrane keratoplasty D. A and B E. All of the above

10 A. Penetrating keratoplasty â€" full thickness Partial thickness - Deep anterior lamellar keratoplasty - Descemet stripping with endothelial keratoplasty - Descemet membrane keratoplasty SIMILAR TO PREVIOUS BOARD EXAM CONCEPT/PRINCIPLE Source: Corneal Endothelial Transplant DSEAK, DMEK and DLEK By John Thomas (MD.), Thomas John

20. Which of the following types of esophageal hiatal hernia is characterized by upward dislocation of both the cardia and the gastric fundus? a. type II rolling b. type I sliding c. type III sliding-rolling d. type IV e. NOTA

100 C. Schwartz Principles of SUrgery 9th edition p. 842

A 22 y/o male was brought to ER due to epistaxis. What is the most common site of anterior epistaxis: A. Woodruff’s plexus B. Kiesselbach’s plexus C. Nasal valve D. Nasal sill E. Both A and B

101 B. Kiesselbach’s area (a richly vascularized area of septal mucosa at the junction of the nasal cavity and vestibule.) Woodruff’s plexus â€" posterior bleeds; convergence of sphenopalatine, anterior ethmoid and labial arteries

This fracture runs along the maxillary sinus, inferior orbital rim, orbital floor, medial orbital wall and nasofrontal suture and includes the pterygoid plate:. A. Le fort type 1 B. Le fort type 2 C. Le fort type 3 D. Nasoorbital ethmoidal fracture E. Transverse maxillary fracture

102 B. Le Fort Type I-(Transverse Maxillary Fracture)- Broken pterygoid plates + fracture that runs horizontally above the anterior maxillary alveolar process Le Fort Type II-(Pyramidal Fracture)-Broken pterygoid plates + fracture that runs along maxillary sinus, inferior orbital rim, orbital floor, medial orbital wall, & nasofrontal suture Le Fort Type III-(Craniofacial Dysjunction)- Broken pterygoid plates + zygomatic arch fracture + craniofacial separation

The fossa of Rosenmuller is located ______________ to the pharyngeal ostia of the eustachian tube: A. Posterior B. Superior C. Inferior D. Lateral E. Medial

103 A. SIMILAR TO PREVIOUS BOARD EXAM CONCEPT/PRINCIPLE. Behind the ostium of the eustacian tube (ostium pharyngeum tuba auditiva) is a deep recess, the pharyngeal recess (fossa of Rosenmüller).

Modified radical neck dissection type 1 preserves : A. Sternocleidomastoid muscle (SCM) B. Internal jugular vein (IJV) C. Spinal accessory nerve (SAN) D. Both A and C E. All of the above

104 C. MRND I- Excise LN I-V, SCM, IJV, Submandibular salivary gland, preserves SAN. MRND II-Excise LN I-V, IJV, preserves SAN, SCM. MRND III- Excise LN I-V, Submandibular Salivary Gland, presrves SAN, SCM, IJV

Which of the following statements is true regarding Nissen fundoplication: A. The reconstructed cardia should not relax on deglutition B. The adequate length of the LES in the positive pressure environment of the abdomen should be at least 1cm C. The LES pressure should be restored to a level 3x the resting gastric pressure D. The fundoplication should increase the resistance of the LES to a level greater than the peristaltic power of the esophageal body E. None of the above

105 E. Page 19 of Topnotch Handout. All statements are false. Principles of anti reflux surgery are: 1) restore LES P to a level 2x the resting gastric P and its length to at least 3 cms. 2) place an adequate length of the LES in the (+) P environment of the abdomen (not <1 cm). 3) allow the reconstructed cardia to relax on deglutition. 4) the fundoplication should not increase the resistance of the LES to a level greater than the peristaltic power of the esophageal body. 5) the repair can be placed in the abdomen without undue tension.

A 25 y/o male weighing 90 kgs was admitted because of flame burns that occurred 2 hours earlier. Vital signs were BP = 130/70 PR -â€" 105/min. RR- 24/min T â€" 37.6’C. The total body surface area burned is approximately 32.5%. The estimated fluid required for the first 24 hours is: A. 10800 ml LRS B. 9100 ml LRS C. 8500 ml LRS D. 11200 ml LRS E. 7000 ml LRS

106 B.onus, C.orrE.C.t A.nswE.r shoulD. B.E. 11,700 mlA person weighing 90 kg with burns to 30% of his or her BSA would require 4 x 90 x 32.5=11,700 mL of fluid replacement within 24 hours. The first half of this amount is delivered within 8 hours from the burn incident, and the remaining fluid is delivered in the next 16 hours.

What is the oncogenic risk of a villous adenoma of the colon? A. 10% B. 20% C. 30% D. 40% E. 50%

217 D. Among all colonic adenomas, the villous type carries the highest risk of malignancy, at 40%. In contrast, tubular adenomas carry only a 5% risk.

If the anterior external opening of an anal fistula is located 5 cm from the anal margin the fistula, where is the track of the fistula found? A. Direct track to the Anterior midline B. Direct into the anal canal C. Direct track to the Posterior midline D. Curved track to the Anterior midline E. Curved track to the Posterior midline

107 E. Goodsall's rule relates the external opening of an anal fistula to its internal opening. It states that the external opening situated below the transverse anal line will open into the anal canal in the midline posteriorly. An anterior opening is usually associated with a radial tract. Anterior fistulas will have a direct track into the anal canal. Posterior fistulas will have a curved track with their internal opening lying in the posterior midline of the anal canal. An exception to the rule are anterior fistulas lying more than 3 cm. from the anus, which may have a curved track (similar to posterior fistulas) that opens into the posterior midline of the anal canal.

A 55 y/o male was brought to the ER due to an inguinal mass of 14 hours duration, appear after lifting heavy objects. Before this event mass appears on standing and disappears when lying down. The mass extends to scrotum. Eventually the patient was operated and at surgery,the transverse aponeurosis arch was sutured to shelving edge of inguinal ligament, this method of operation is A. Lichenstein repair B. Bassini repair C. McVay repair D. Shouldice repair E. Pott's repair

108 B. Page 73 of surgery supplement Topnotch Handout.

A 39 y/o female notice a 2cm hard on Left breast 2 yrs ago progressively enlarging to its present size of 12x10cm ulcerating weeping lesion. The Left axilla appear to be free of clinically palpable nodes. The microscopic picture showed leaf-like pattern of complexity branching spaces invaginated by fibrous stroma. What is the recommended treatment? A. Radical mastectomy B. Extended Radical mastectomy C. Simple mastectomy D. Lumpectomy + sentinel lymph node biopsy E. Excision with narrow margins

109 C. Page 10 of Topnotch Handout. This is a phyllodes tumor. Treatment is excision with a margin of normal tissue (1cm) or simple mastectomy. There is no need for axillary dissection (no lymph node involvement)

Which of the following structures are removed in a skin sparing mastectomy? A. All breast tissue B. Nipple areola complex C. Scars from previous biopsy procedures D. A and B E. All of the above

11 E. Breast conservation â€" resection of primary breast cancer + margin of normal appearing breast tissue + assessment of regional lymph node status Radical mastectomy â€" Medial/Lateral pectoral nerves, Long thoracic nerve, Thoracodorsal nerve + Pectoralis Major and minor + All LN + All breast Modified radical mastectomy â€" Same as Radical mastectomy except all nerves are spared, level III LN spared, Pectoralis major spared Source: Schwartz’s Principles of Surgery 8th ed p 460-461 + Topnotch Lecture on Surgery

A 55 y/o female present with dysphagia of 5 months duration. Esophagram revealed a dilated esophagus with narrowing at the distal end (Bird’s beak appearance). The most probable diagnosis is: A. Hypertensive LES B. Diffuse esophageal spasm C. Nutcracker esophagus D. Achalasia E. Hiatal hernia

110 D. Page 21 of Topnotch Handout. Achalasia esophagogram shows bird's beak esophagus, air fluid level, sigmoid esophagus, esophageal dilation.

The triangle of calot is composed of the ff except:: A. Cystic duct B. Common hepatic duct C. Inferior border of the liver D. Common bile duct E. None of the above

111 D. The common bile duct is not part of the triangle of Calot.

Stones in Common duct discovered months or years later after surgery are termed: A. Retained stones B. Recurrent stones C. Primary stones D. Secondary stones E. Tertiary stones

112 B. Stones deliberately left in place or discovered shortly after surgery are retained stones. Stones associated with bile stasis and infection are primary stones. Stones that are formed in the GB and migrate down the cystic duct to the CBD are secondary stones.

When progressive enlargement of a multinodular goiter causes symptomatic tracheal compression, the preferred management in an otherwise good risk patient is: A. iodine treatment B. thyroid hormone treatment C. surgical resection of abnormal thyroid D. RAI E. None of the above

113 C. Symptomatic tracheal compression is a surgical indication.

19. A patient who has undergone parotid surgery noticed that whenever he eats, a portion of his skin in the cheek area sweats. He has: A. Facial nerve injury B. Injury to cervical sympathetic nerves C. Cervical parasympathetic nerve injury D. Great auricular nerve injury E. Auriculo-temporal nerve injury

114 E. This is a case of Frey's syndrome.

In esophageal carcinoma treatment, the best chance for cure is through curative resection. The following are the criteria for exclusion for curative resection, except: A. Age >75 B. Weight loss <20% C. FEV1< 1.25 D. EF < 40% E. Locally advance tumor

115 B. Page 22 of Topnotch Handout. Exclusion criteria for curative resection are the following: 1) Age >75, 2) >20% weight loss, 3) FEV1 < 1.25, 4) EF <40%, 5) locally advance tumor

A 21 y/o male presents to the ER with complaints of left upper quadrant abdominal pain and excreting dark stool for a week. He had mitral valve replacement 4 years ago and he had been on oral warfarin treatment. On physical examination, he was anemic, had a blood pressure of 140/80 mmHg, a heart rate of 114/min and a hematocrit level of 17.6 % and the left upper quadrant was tender on palpation. After volume replacement and blood transfusion, the patient underwent upper GI endoscopy which revealed minimal bleeding in a mucosal tear in the distal esophagus. What is the next course of action? A. Explore laparotomy B. Ex-lap with high gastrotomy C. Ex-lap with oversewing of the tear D. Endoscopic coagulation of bleeders E. None of the above

116 D. Page 23 of Topnotch Handout. This is a case of a Mallory-weiss tear. Majority resolves spontaneously. However for this patient the best course of action would be endoscopic coagulation of bleeders. If endoscopic tx fails, laparotomy+ high gastrotomy + oversewing of the tear is the next course of action.

In diagnosing acute appendicitis, there are different signs which can be elicited. Which sign tells you that the tip of the appendix is directed into the pelvis? A. Obturator sign B. Dunphy's sign C. Psoas sign D. Markle sign E. Rovsing's sign

118 A. Page 29. Obturator sign tells you that the tip of the appendix is directed into the pelvis. Psoas sign tells you that the tip is retroperitoneally directed. Markle sign or jar tenderness is a clinical sign in which pain in the right lower quadrant of the abdomen is elicited by dropping from standing on the toes to the heels with a jarring landing. It is found in patients with localised peritonitis due to acute appendicitis. Dunphy's sign is a medical sign characterized by increased abdominal pain with coughing. Rovsing's sign, palpation of the left lower quadrant of a person's abdomen increases the pain felt in the right lower quadrant, the patient is said to have a positive Rovsing's sign and may have appendicitis.

The ideal sites for colostomy are the following, except: A. Away from bony prominences B. Away from natural skin creases C. Should be in the rectus abdominis D. In a place that is accessible and can be seen by the patient E. None of the above

119 E. Page 31 of Topnotch Handout. This was discussed by Dr. Antonio during the lecture. All of the choices given are conditions for ideal site of colostomy placement.

Which of the following pertains to Osteonecrosis of the proximal femoral epiphysis of the pediatric hip and is thought to be due to vascular compromise? A. Slipped capital femoral epiphysis B. Developmental dysplasia of the hip C. Talipes equinovarus D. Legg-Calve-Perthes disease E. Osgood-Schlatter disease

12 D. Legg-Calve-Perthes disease also known as cox plana is a condition fo the pediatric hip characterized by a flattened misshapen femoral head. The etiology is related to osteonecrosis of the proximal femoral epiphysis and is thought to result from vascular compromise. SIMILAR TO PREVIOUS BOARD EXAM CONCEPT/PRINCIPLE Source: Schwartz’s Principles of Surgery 8th ed p 1605

Due to the liver's anatomy, the hardest segment to resect is the: A. Quadrate lobe B. Caudate Lobe C. Left lateral segment D. Right anterior lobe E. Left medial segment

120 B. Page 36 of Topnotch Handout. The hardest lobe/segment to resect is segment 1/caudate lobe. The caudate lobe (posterior hepatic segment I, Spigelian lobe) is situated upon the postero-superior surface of the liver on the right lobe of the liver, opposite the tenth and eleventh thoracic vertebrae. Caudate lobe resection is one of the most demanding procedures among hepatic resection, owing to its deep and complex location and its proximity to major vessels. The caudate lobe is located beneath the major hepatic veins, between the portal vein and ligamentum venosum, and anterior and partially circumferential to the inferior vena cava (IVC).

How many kilocalories does a 4.5 L of D5LR provide? A. 600 kcal B. 700 kcal C. 850 kcal D. 900 kcal E. 1050 kcal

121 D. D5LR has 50 g of Dextrose. 1 g of glucose = 4 kcal (According to Dr. Antonio Dextrose contains 3.4 kcal/g, however there is no 765 kcal/g in the option); 4.5L x 50 g x 4 kcal/g = 900 kcal. SIMILAR TO PREVIOUS BOARD EXAM CONCEPT.

Patient arrives at the Emergency Department with multiple ice pick stabs onn the chest. He suddenly develops difficulty of breathing and hypotension. On physical examination, there was absent breath sounds on the right chest wall. What will you do next? A. Do an emergency chest xray B. Do immediate chest tube thoracostomy C. Do needle thoracostomy D. Do endotracheal intubation E. Intercostal nerve block

122 C. This is a case of tension pneumothorax which is an emergency condition. Immediate management of patients would be needle thoracostomy at the 2nd ICS MCL.The question is not definitive treatment. The answer will be the next step or the immediate/emergency management SIMILAR TO PREVIOUS BOARD EXAM CONCEPT.

Which of the following finding would warrant immediate referral to a surgeon for tube thoracostomy? A. Hypotension B. Tachypnea C. Bleeding D. Absent breath sounds E. Apnea

123 D. Absent breath sounds is the most important indication for chest tube thoracostomy. SIMILAR TO PREVIOUS BOARD EXAM CONCEPT

The mimimum urine output for adult is? A. 15 cc/hr B. 20 cc/hr C. 25 cc/hr D. 30 cc/hr E. 40 cc/hr

124 D. The most reliable indicator of organ perfusion during rescucitation is the urine output. It should be at least 0.5 ml/kg/hr or 30-60 ml/hr for adults; 1 ml/kg/hr in children and 2 ml/kg/hr for infants. SIMILAR TO PREVIOUS BOARD EXAM CONCEPT.

A 35-year-old male patient presented with abdominal pain, fever, diarrhea and weight loss. Patient had a history of Pulmonary tuberculosis 5 years ago. Which of the following diagnostic test will you request to confirm the diagnosis of ileocecal tuberculosis? A. Whole abdominal ultrasound B. Abdominal CT scan C. Colonoscopy D. Abdominal xray with barium studies E. MRI

125 C. The ileocecal and jejunoileum are the most common sites of involvement of GI Tuberculosis. Definitive diagnosis is based on histopathology and culture of biopsy specimens obtained by colonoscopy or laparotomy. Barcium contrast studies and colonoscopy may show ulcers, strictures, a deformed cecum, incompetent ileocecal valve or fistulas. An abdominal CT scan can define extraluminal pathology, especially lymphadenopathy. SIMILAR TO PREVIOUS BOARD EXAM CONCEPT.

The management for carbuncle: A. Incision and drainage B. Antibiotic C. Wide excision D. Elevation and splinting E. Serial debridement

126 A. More involved, deep-seated infections that result in multiple draining cutaneous sinuses are called carbuncles. Along with furuncles, these lesions often require incision and drainage before healing can be initiiated. (Schwartz). SIMILAR TO PREVIOUS BOARD EXAM CONCEPT.

Patient presented with progressive dysphagia to both solid and liquid for a year now. What is your primary impression? A. Achalasia B. Esophageal cancer C. Cancer of the stomach D. Epiphrenic diverticula E. Diaphragmatic hernia

136 A. Dysphagia to both solids and liquids points to a motility probllem. . Progressive dysphagia for both solids and liquids is the hallmark of achalasia. Progressive dysphagia (initially solid then liquid) is most likely caused by a mechanical obstruction (peptic stricture, esophageal cancer)

An 18 year-old male presented with right lower quadrant pain and mild fever. Your primary diagnosis is acute appendicitis. He is most likely at what stage of appendicitis? A. Necrotizing B. Suppurative C. Perforated D. Gangrenous E. Resolving

127 B. Stages of Appendicitis: EARLY STAGE - obstruction of appendiceal lumen leading to mucosal edema, ulceration, bacterial diapedesis and appendiceal distenstion due to increasing intraluminal pressure and accumulated fluid. The visceral afferent nerve fibers are stimulated, and the patient perceives mild visceral periumbilical or epigastric pain. SUPPURATIVE - increasing intraluminal pressure exceed capillary perfusion pressure associated with obstructed lymphatic and venous drainage. Transmural spread of bacteria causes acute suppurative appendicitis. When the inflammed seros comes in contact with parietal peritoneum, patient experience a shift of pain from periumbilicus to RLQ. GANGRENOUS APPENDICITIS - intramural venous and arterial thromboses ensue. PERFORATED - persisting tissue ischemia results in appendiceal infarction and perforation. SIMILAR TO PREVIOUS BOARD EXAM CONCEPT.

Which of the following is true regarding acute appendicitis in the young and in the elderly? A. The incidence of rupture is common in both age groups B. Both may present atypical clinical features. C. Progression to perforation is more rapid both age groups owing to physiologic and anatomic factors. D. All of the above. E. Only A and B.

128 D. Rupture incidene is higher in the pediatric and geriatric age group. Progression to perforation is rapid in the elderly owing to decreased blood supply or decreased lymphoid tissue. In pediatric population, the more rapid progression to rupture and inability of the underdeveloped greater omentum to contain a rupture lead to increased morbidity in children. Both age groups had atypical presentation making diagnosis difficult. (Source: Schwartz, Sabiston, Medscape)

Patient is about to undergo a small mass excision on her back under local anesthesia. She requested for a prolonged anesthesia since she opted to go to work right after the procedure. What anesthetic agent will you use? A. Lidocaine B. Mepivacaine C. Procaine D. Bupivacaine E. Prilocaine

129 D. Bupivacaine is long-acting local anesthetic with a duration of action of 2-9 hours. Short acting: Procaine; Medium: Lidocaine, Mepivacaine, Prilocaine; Long-acting: Tetracaine, Bupivacaine, Ropivacaine. SIMILAR TO PREVIOUS BOARD EXAM CONCEPT.

Which of the following orthopedic conditions describe a form of neuropathic arthropathy that affects some diabetics with peripheral neuropathy? A. Charcot joints B. Osgood Schlatter Disease C. Paget Schroetter Disease D. Kimmel-Steil Wilson Lesions E. None of the above

13 A. It may also occur with several other diseases that affect the sensory nervous system (alcoholism, leprosy, syphilis, Charcot-Marie- Tooth Disease to name a few). In the United States, diabetes is the number-one cause. SIMILAR TO PREVIOUS BOARD EXAM CONCEPT/PRINCIPLE Source: https://www.aofas.org/footcaremd/conditions/ diabetic-foot/Pages/Charcot-Joints-or- Neuropathic-Arthropathy.aspx

A 62-year-old female presented with 6 month- history of ulcerating lesion in the middle third of her lower lip. This is most likely a: A. Basal cell carcinoma B. Squamous cell carcinoma C. Melanoma D. Adenocarcinoma E. Undifferentiated carcinoma

130 B. The most frequent tumor related to lips is squamous cell carcinoma, with the lower lip more commonly involved than the upper lip. An ulcerating lesion is a common presentation in SCC.

A 65-year-old patient presented with difficulty in urination. Findings showed an enlarged prostate and a PSA level of 9 ng/ml. What is the most appropriate diagnostic test that you should request? A. IVP B. Prostate biopsy C. Renal ultrasound D. CT scan E. MRI

131 B. National Comprehensive Cancer Network Guidelines (NCCN) use 3 ng/ml as PSA threshold that would prompt prostate biopsy. A PSA level of 4-10 ng/ml, the likelihood of finding prostate cancer is about 25%. Needle biopsy of the prostate is indicated for tissue diagnosis in patients who present with elevated PSA levels or abnormal DRE findings. SIMILAR TO PREVIOUS BOARD EXAM CONCEPT.

A 45-year-old male presented with dysphagia and weight loss. Endoscopic finding showed an irregularly lined mass at the gastroesophageal junction. Biopsy was done which will most likely reveal: A. Adenocarcinoma B. Squamous cell carcinoma C. Carcinoid tumor D. Lymphoma E. GIST

132 A. The gross appearance of adenocarcinoma resembles that of squamous cell carcinoma. The most important etiologic factor in the development of primary adenocarcinoma of the esophagus is a metaplastic columnar-lined or Barrett's esophagus, which occurs as a complication in approximately 10-15% of patients with GERD. Most adenocarcinomas of the esophagus develop within the lower thoracic esophagus or gastroesophageal junction.

Patient presented with fever, RUQ pain and anorexia. Laboratory findings revealed elevated WBC and alkaline phosohatase. Ultrasound showed a 5 cm hypoechoic, unilocular lesion with well defined border at the left lobe of the liver. Which of the following characteristic of the abscess warrants aspiration? A. the size of the abscess B. presence of fever C. presence of elevated alkaline phosphatase D. the location of the abscess E. all of the above

133 D. Indications for aspiration: Large abscess >10 cm, failure of medical management, superinfection, abscess of left lobe (may cause rupture; affect the pericardium --> tamponade). * In February 2015 boards, what is asked is the most likely affectation if liver abscess is at the left lobe. (pericardium)

Patient presented with a mass on the anterior thigh which measures 6 cm in diameter. Which of the following should be performed to determine his diagnosis? A. Incision biopsy B. Core-needle biopsy C. Excision biopsy D. Fine-need aspiration biopsy E. Stereotactic biopsy

134 A. Open biopsy is a reliable diagnostic method that allows adequate tissue to be sampled for definitive and specific histologic identification of bone or soft tissue sarcoma. When adequate tissue for diagnosis cannot be obtaine by FNAB or core biopsy, an incisional biopsy is indicated for deep tumors or for superficial soft tissue tumors larger than 3 cm. Excisional biopsy can be performed for easily accessible or truncal lesions smaller than 3 cm. (Schwartz) SIMILAR TO PREVIOUS BOARD EXAM CONCEPT.

The most common site of bronchogenic carcinoma: A. Trachea B. Hilum C. Alveolar septum D. Alveolar sac E. Terminal bronchioles

135 B. Lung carcinoma arise most often in and about the hilus of the lung. (Robbin's). SIMILAR TO PREVIOUS BOARD EXAM CONCEPT

A 33-year old female presented with a solitary nodule on the right anterior neck and significant weight loss. What is the most important test to determine her diagnosis? A. Free T3 B. Neck ultrasound C. CT scan D. Fine Needle aspiration biopsy E. Neck MRI

137 D. FNAB has become the single most important test in the evaluation of patients with thyroid masses and can be performed with or without ultrasound guidance. (Schwartz) * SIMILAR TO PREVIOUS BOARD EXAM CONCEPT. If the question is what is the next step or diagnostic test to request, the answer is TSH.

The most important diagnostic test in caustic injury is: A. Ultrasound B. Endoscopy C. pH testing D. Chest Xray E. Barium swallow

138 B. SIMILAR TO PREVIOUS BOARD EXAM CONCEPT. The gold standard of safely assessing depth, extent of injury, and appropriate therapeutic regimen is esophagogastroduodenoscopy.

A 1-year-old male presented with chronic constipation since birth, poor weight gain and abdominal distention. He had a history passage of meconium at48 hrs after birth. Which of the following will be most helpful in confirming his diagnosis? A. Plain abdominal radiograph B. Barium enema C. Anorectal manometry D. Rectal biopsy E. CT scan

139 D. Full-thickness recal biopsy is the gold standard for establishing the diagnosis of Hirchsprung diseas. It will reveal absence of ganglion cells in the myenteric plexus, increased AchE nerve fibers and hypertrophied nerve fibers.

A 57 year old male with poorly controlled hypertension presents with acut onset painless loss of vision in the left eye. PE reveals a relative afferent pupillary defect and fundoscopy of the affected eye showed “cherry red spot†over a pale background. What is the diagnosis? A. Rhegmatogenous retinal detachment B. Age related macular degeneration C. Central serous retinopathy D. Central retinal artery occlusion E. Central retinal vein occlusion

14 D. Also called amaurosis fugax. Management options include direct infusion of thrombolytic to the ophthalmic artery or by decreasing IOP (IV acetazolamide or vitreous paracentesis) SIMILAR TO PREVIOUS BOARD EXAM CONCEPT/PRINCIPLE Source: http://www.williamsoneyeinstitute.com/retina- center/retinal-artery-occlusions

The most common type of elbow dislocation following an injury is: A. Anterior B. Posterior C. Lateral D. Medial E. Oblique

140 B. Posterior elbow dislocation comprise over 90% of elbow injuries. Typically, this injury is caused by a traumatic fall onto an outstretched arm resulting in an hyper-extension injury.

What is the most common cause of hepatic abscess A. Ruptured appendicitis B. Acute pancreatitis C. Diverticulitis D. Cholangitis E. None of the above

141 D. Cholangitis is the most common cause of hepatic abscess as any obstruction in bile flow is a medium for bacterial proliferation.

ABE, a 35 year old female was diagnosed with suppurative cholangitis, what is the drug of choice A. Cloxacillin B. Penicillin G C. Metronidazole D. Erythromycin E. None of the above

142 C. Metronidazole covers for the most common pathogens associated with suppurative cholangitis

Which of the following statements is true A. Lymphatic capillaries are involved in the absorption of digested sugars B. Lymphatic vessels typically have very few valves C. Macrophages are NOT an example of lymphoid cells D. The expansion and recoil of arteries assists in causing lymph flow E. None of the above

143 C. or D. Lymphatic capillaries are involved in the absorption of digested fats, lymphatic vessels have many valves preventing backflow of lymph, macrophages are found in lymphoid tissue though they are derived from monocytes of myeloid origin and the pulsation of the arteries as well as the muscle contraction and relaxation aid in lymphatic flow.

RJC, a 30 year old male was diagnosed with chronic necrotizing anal fistula, what should the management be? A. Hot Sitz bath B. High dose antibiotics C. Debridement D. Observe E. None of the above

144 C. Debridement is recommended with antibiotics to promote healing by secondary intention.

PM, a 31 year old female presents at the clinic with an anterior neck mass, what should be the initial diagnostic test? A. TSH B. Thyroid scan C. FNAB D. Incision biopsy E. None of the above

145 A. Determine first whether the thyroid function is normal or not.

AJS, a 60 year old male came in to the clinic with painless, palpable lymph nodes and hoarseness, you suspect that this is a malignancy but there is no visible malignancy and the positive nodes are at level II and III, what should you do? A. Chest x-ray B. Laryngoscopy C. CT scan D. Neck dissection E. None of the above

146 B. The symptoms are suggestive that the malignancy may be at the larynx so laryngeal CA should be ruled out first

KD, a 31 year old male who came in due to right lower quadrant pain was seen at the ER, there's no fever and the WBC count was normal. At what phase of appendicitis is the patient in? A. Suppurative B. Gangrenous C. Perforated D. Chronic E. None of the above

147 A. The patient is still at the suppurative stage, there is pus in the appendix but not yet distending the organ to cause ischemia or any systemic manifestations

RY, a 35 year old male was brought to the ER due to signs of peritonitis, history revealed that the patient had a drinking spree 1 week prior to consult, PE showed decreased BP with slight tachycardia and tachypnea and a temperature of 38.8C, what is the most likely diagnosis? A. Acute hemorrhagic pancreatitis B. Necrotizing pancreatitis C. Acute edematous pancreatitis D. Gallstone pancreatitis E. None of the above

148 A. The most likely cause is acute hemorrhagic pancreatitis, on examination, the patient may show tachycardia, tachypnea, hypotension, and hyperthermia. The temperature is usually only mildly elevated in uncomplicated pancreatitis. Hypotension is a sign of sequestration of edematous fluid into the retroperitoneum. (Schwartz, 9th ed)

A patient complains of severe flank pain and hematuria.Upon work-up, CT stonogram showed staghorn calculi. Which of the following is the most common component of the stone? A. Cystine B. Calcium oxalate C. Uric acid D. phosphate E. Magnesium ammonium phosphate

161 E. this stones are also called struvite stones and usually associated with urease producing microoraganism

A man was diagnosed with amebic liver abscess localized at the left hepatic lobe, why should it be aspirated ASAP A. Can go back into the GIT through the stomach B. Can extend or rupture into the pericardium C. Can extend outward into the skin D. Both A and C E. None of the above

149 B. Because of the proximity of the left lobe to the pericardial space just above the diaphragm, left lobe liver abscesses should be treated promptly.

Which of the following aortic aneurysms merit/merits operative management? A. Sudden onset lower back pain in a man diagnosed with aortic aneurym 6 months ago. Last CT showed diameter of 4.5cm. B. Asymptomatic ascending thoracic aortic aneurysm of 5.75cm C. Asymptomatic descending thoracic aortic aneurysm of 5.75cm D. A and B only E. All of the above

15 D. Indications for repair of aortic aneurysms in asymptomatic patients without connective tissue disorders - Ascending aortic aneurysm >5.5cm diameter - Descending aortic aneurysm >6.5cm diameter - Abdominal aorta >5.5cm - Rate of dilatation >1cm/yr for thoracic aorta/ >0.5cm for abdominal aorta Normal aortic diameter: 2-3cm Source: Schwartz’s Principles of Surgery 8th ed p 671, 723

SNJ, a 39 year old male working as a horse jockey fell off from his horse straddling a fence, he was brought to the ER, you should perform the following except? A. Foley catheterization B. Perform cystourethrogram C. Pelvic x-ray D. Both B and C E. None of the above

150 A. Foley catheterization is contraindicated as it may cause further damage to the urethra, evaluate first the pelvis and rule out any urethral injury before inserting a foley catheter

PNJ, a 29 year old male came in to the ER due to ingestion of muriatic acid, 1hour after the ingestion, what should you do A. Perform barium enema B. Perform esophagoscopy C. CT scan D. Insert a NGT E. None of the above

151 B. Perform a baseline esophagoscopy to determine the damage to the esophagus

After 48 hours, PNJ should undergo next? A. Chest x-ray upright B. Esophagoscopy C. CT scan D. NGT insertion E. None of the above

152 A. To rule out any esophageal perforation as air from the perforated esophagus will collect in the mediastinum

The pain from the inflammation of the cystic duct is commonly referred where? A. Epigastric B. Umbilical C. Hypogastric D. Right shoulder E. None of the above

153 D. The visceral afferent fibers share the same as those of the cutaneous fibers of the right shoulder.

The lienorenal ligament contains all of the following except: A. Splenic artery and vein B. Tail of the pancreas C. Renal artery and vein D. Both A and B E. All of the above

154 C. Both structures, Splenic artery and vein as well as the tail of the pancreas are contained in the lienorenal or splenorenal ligament

The majority of the lymph returns to the venous circulation by way of A. Right lymphatic duct B. Thoracic duct C. Cisterna chyli D. Azygos vein E. None of the above

155 B. The thoracic duct drains the left side of the head up to the thorax and both abdomen and lower extremities.

Which of the following statement/s is/are true? A. The thymus and the thyroid are both lymphoid organs B. Lymph nodes are the only lymphoid organs that contain lymphocytes C. Peyer’s patches are found within the distal small intestine D. All lymphoid organs contain both efferent and afferent lymphatic vessels E. All of the above

156 C. The peyer's patches are found in the distal small intestine specifically the ileum.

Which of the following tonsils is most superior? A. Palatine B. Lingual C. Pharyngeal D. Choroidal E. None of the above

157 C. The pharyngeal tonsil is the most superior

You are about to perform a chest tube thoracostomy for a suspected malignant pleural effusion, right, you know that the best site to incise is at the: A. 4th ICS mid axillary line B. 5th ICS anterior axillary line C. 6th ICS mid axillary line D. 8th ICS posterior axillary line E. None of the above

158 A. The best site to insert the chest tube would be at the 4th-5th ICS MAL.

The following are absorbable sutures except: A. Vicryl B. Nylon C. Monocryl D. Chromic E. Cat gut

159 B. Nylon is non-absorbable

Dr. Juan dela Cruz is assisting in a bowel surgery and is about to do an anastomosis. Which of the following is the strongest layer of the of the bowel wall that he has to pay special attention to? A. Mucosa B. Submucosa C. Muscularis Propria D. Serosa E. Adventitia

16 B. Tensile strength Aorta: Tunica media GIT: submucosa Source: Schwartz’s Principles of Surgery 8th ed p 667 + Topnotch handout in surgery

Which of the following is a function of the spleen? A. The spleen is located in the lower right hand quadrant of the abdominal cavity B. The spleen consists primarily of white pulp, which functions in RBC recycling C. The spleen is the only lymphoid organ that entirely lacks white blood cells D. If the spleen is surgically removed, many of its blood cleansing functions can be taken over by the liver E. The spleen is the only lymphoid organ that contains both afferent and efferent lymphatic vessels]

160 D.

A patient came in at the ER due to head trauma. Upon assesment with your senior resident, he said that his findings is very much consistent of an epidural hematoma. Which of the following is true regarding this case? A. venous blood seeping out from the vessel B. CT scan revealed lenticular shaped lesion C. bridging veins are involved D. delayed presentation with fluctuating levels of consciousness E. none of the above

162 B. other choices are characteristic of subdural hematoma

A patient came in at your out patient clinic for excision of a nodule. He said that he wants the anesthesia to have a longer duration of action because he needs to be as painfree as possible for a meeting after the procedure. He also added that he has history of arrythmia. Which of the following local anesthetics is the most appropriate for the patient? A. Lidocaine B. prilocaine C. mepivacaine D. bupivacaine E. ropivacaine

163 E. Among the choices, ropivacaine and bupivacaine offers the longest duration of action but ropivacaine has a lesser cardiotoxicity property SIMILAR TO PREVIOUS BOARD EXAM CONCEPT/PRINCIPLE

Among the following local anesthetics, which of the following has the longest half-life? A. procaine B. lidocaine C. prilocaine D. Bupivacaine E. Mepivacaine

164 D. Schwartz 9th ed. Page 1736. SIMILAR TO PREVIOUS BOARD EXAM CONCEPT/PRINCIPLE

In severe burns, the goal for urine output in adults should be 30ml/h and to ensure optimal end- organ perfusion, the target MAP is _____ mmHg. A. 50 B. 60 C. 70 D. 80 E. 90

165 B. Schwartz 9th ed. Page 200

Mafenide acetate is one of the alternatives for topical antibioitic treatment in burns. Which of the following is a major side effect of the drug that should be observed? A. Metabolic acidosis B. Respiratory acidosis C. neutropenia D. anemia E. ototoxicity

166 A. Schwartz 9th ed. Page 202

A medium-built 35 year old man came in at the ER due to bleeding secondary to gunshot wound. His heart rate is 115, anxious and he is already having orthostatic hypotension. Based on the classification of hemorrhage, approximately how many mililiters is the blood loss? A. <500 B. 500-750 C. 750-1500 D. 1500-2000 E. >2000

167 C. patient is at class II in which the approximated blood loss is 750-1500 or 15-30% of blood, >100 HR, orthostatic hypotension, and patient tends to be anxious. Class I <750ml. Class II is 1500-2000. class IV is >2000. Patient is only on ORTHOSTATIC hypotension. See picture from schwartz 9th ed page 99

Biliary colic generally includes 1-5 hours of constant pain on the epigastrium or right upper quadrant which usually radiates to what part of the body? A. left flank area B. Right flack area C. Right axillary region D. Left scapular region E. Right scapular region

168 E.

In tension pneumothorax, the contralateral lung is compressed and the heart rotates about this/these vessel which will eventually cause circulatory collapse. A. Inferior vena cava B. Superior vena cava C. aorta D. Pulmonary vessels E. A and B

169 E. schwartz 9th ed page 138

A 42-year-old jaundiced female with pancreatic cancer is admitted at the female ward of the hospital you are on duty in. She has been febrile for 24 hours now and the nurse refers her to you for changes in sensorium. You evaluate her and on PE, her vitals were HR 98 RR 20 BP 80/50 T38.9. What is the next step in management? A. IV antibiotics and fluids B. Laparoscopic cholecystectomy with sphincterotomy C. Emergency exploratory laparotomy D. Whipple’s procedure E. Bowel resection with end to end anastomosis

17 A. This is a case of ascending cholangitis secondary to biliary tract obstruction secondary to a pancreatic mass. The patient presented with Raynaud’s pentad. The initial management in such patients is IV antibiotics and fluid resuscitation. About 15% of patients will not respond to antibiotics and fluid resuscitation and an emergency biliary decompression may be required. Source: Schwartz’s Principles of Surgery 8th ed p 1150

A 4 year old male was rushed to the ER because of vehicular accident. The patient is already tachycardic, hypotensive, tachypneic, confused and pale. Fluid resuscitation of lactated ringersn of IV bolus should be immediately started at A. 10 ml/kg B. 20 ml/kg C. 30 ml/kg D. 40 ml/kg E. 50 ml/kg

170 B. schwartz 9th ed page 141

A head trauma patient came in at the ER. Upon assessment, you elicited eye opening through pain, responds with inapropriate words, and withdraws to pain. What is the GCS score of the patient? A. 6 B. 7 C. 8 D. 9 E. 10

171 D. E2V3M4

In cases of trauma, once immediate threats to life have been addressed, secondary survey is initiated. The patient and surrogates should be querried to obtain an AMPLE history. All of the following are component of AMPLE history EXCEPT: A. Allergies B. Medications C. Pregnancy D. Last voiding episode or urine output E. Events related to injury

172 D. Last meal… P is previous illness and pregnancy

This cytokine induces fever through prostaglandin activity in anterior hypothalamus and it also promotes beta endorphin release form pituitary A. Interleukin 1 B. Interleukin 2 C. Interleukin 6 D. Interleukin 8 E. Interferon gamma

173. A.

In response to injury, this cytokine is among the earliest responders and it induces muscle breakdown and cachexia through increased metabolism. A. Interleukin 1 B. Interleukin 2 C. Interleukin 8 D. Interferon gamma E. TNF alpha

174 E.

In most series reports, which of the following is the most common type of primary appendiceal tumors which has also the best 5-year survival? A. Goblet cell carcinoma B. Mucinous carcinoma C. carcinoid D. Signet-ring ccarcinoma E. Adenocarcinoma

175 C. schwartz 9th ed page 1088

Lobular carcinoma in situ of the breast is best managed by: A. Breast conservation surgery B. MRM C. Radiotherapy D. Chemotherapy E. Observation with or without tamoxifen

214 E. Because LCIS is considered a marker for increased risk rather than an inevitable precursor of invasive disease, the current treatment of LCIS is observation with or without tamoxifen.

During appendectomy, a firm yellow bulbar mass approximately 1.8cm located at the tip of the appendix was seen. Quick histologic study was done and shows malignancy. Which of the following is the procedure of choice to be done? A. appendectomy alone B. right hemicolectomy C. extended hemicolectomy D. total colectomy E. exploratory laparotomy

176 A. the mass is most likely carcinoid. If the mass in <1cm or if it is >1-2cm but located at the tip or mid appendix, appendectomy alone should be done. Appendectomy lang. See Picturefrom schartz 9th ed page 1088.

A patient with fistula in ano came in to your clinic for consult. On assessment, you saw that the anterior external opening is approximately 4 cm fromm the anal margin. With this, you expect that the interior opening is: A. connected with a short radial tract to the anterior midline B. connected with a short radial tract to the posterior midline C. connected in a curvilinear fashion to the anterior midline D. connected in a curvilinear fashion to the posterior midline E. none of the above

177 D. aplication of the Goodsall's rule.

A 60 year old 20 pack year smoker came in at the OPD for check up. Your senior resident suspects that the patient has nasopharyngeal carcinoma because of the location of the lymphadenopathy. The lymph node is most likely located at what level? A. I B. II C. III D. IV E. V

178 E. schwartz 9th ed page 490.. Lymphadenopathy of the posterior triangle of the neck should provoke consideration for a nasopharyngeal primary

A 50 year old woman came in to your clinic because of painless anterior neck mass which was associated with dysphagia, dyspnea, choking and hoarseness and signs and symptoms of hypothyroidism. Upon palpation, the mass in hard, woody thyroid gland with fixation ro the sorrounding tissue. A. Reidel's thyroiditis B. Thyroid nodule C. de quervain's thyroiditis D. acute suppurative thyroiditis E. Papillary carcinoma

179 A.

A 15 year old female has a history of intermittent abdominal pain and jaundice. On PE, hepatosplenomegaly was noted. A ultrasound revealed intrahepatic biliary cysts. What is the diagnosis? A. Choledochal cyst type I B. Choledochal cyst type II C. Choledochal cyst type III D. Choledochal cyst type IV E. Choledochal cyst type V

18 E. Choledochal cyst type I â€" saccular/fusiform dilatation Choledochal cyst type II â€" isolated diverticulum Choledochal cyst type III - choledochocele Choledochal cyst type IV â€" dilatations of the intra and extrahepatic/ extrahepatic biliary tree only Choledochal cyst type V â€" dilatation of the intrahepatic biliary tree Source: Schwartz’s Principles of Surgery 8th ed p 1158

The following are true with Thyroid Papillary carcinoma EXCEPT: A. It is the most common thyroid malignancy in iodine sufficient areas B. It has excellent prognosis C. Usual metastatic sites are lungs, bones, liver and brain D. Age is the most important prognostic factor in determining long term survival E. spreads via hematogenous route

180 E. usually spread via lymphatic route

What is the most appropriate definitive management for patients with Grave's disease who is positive for Dalrymple and von Graefe sign? A. Subtotal thyroidectomy B. Near Total thyroidectomy C. Isthmusectomy D. Radioactive Ablation therapy E. Medical treatment

181 B. Dalrymple sign is upper lid retraction, von Graefe sign is lid lag on downgaze, both seen in Grave's patient with ophthalmopathy. Since the patient already have ophthalmopathy, radioactive ablation is no longer considered because it worsens the ophthalmopathy. Near total thyroidectomy is done in patients with coexistent thyroid Ca, refuse RAI, ophthalmopathy, compressive symptoms and severe reactions to thyroid drugs.

What is the most common location of lung cancer? A. 1st generation bronchi B. 2nd generation bronchi C. alveolus D. Terminal bronchiole E. All of the above

182 A. SIMILAR TO PREVIOUS BOARD EXAM CONCEPT/PRINCIPLE.. Bronchogenic carcinoma accounts for about one third of all cancer deaths in men and is becoming increasingly common in women. It commences in most patients in the mucous membrane lining the larger bronchi and is therefore situated close to the hilum of the lung.

During thyroidectomy, a nerve was accidentally severed that lies in close proximity to the inferior thyroid artery. This nerve is: A. Superior laryngeal nerve B. Internal laryngeal nerve C. Recurrent laryngeal nerve D. External laryngeal nerve E. Galla Curci nerve

215 C.

4 weeks old infant presented with projectile non- bilous vomiting. A firm, movable olive-shaped mass can be palpated in the abdomen. What diagnostic test will confirm the diagnosis? A. MRI B. CT scan C. Abdominal x-ray D. Ultrasound E. PET scan

183 D. The diagnosis is hypertrophic pyloric stenosis. The confirmatory test is ultrasound

A patient has fistula-in-ano. The external opening was noted at 2 o'clock position (right anterolateral), 2.5cm from the the anal margin. The internal opening of the fistulous tract is expected to be in what position? A. posterolateral B. posterior midline C. right anterolateral D. left posterolateral E. left anterolateral

184 C. In general, fistulas with an external opening anteriorly connect to the internal opening by a short, radial tract. Fistulas with an external opening posteriorly track in a curvilinear fashion to the posterior midline. However, exceptions to this rule often occur if an anterior external opening is greater than 3 cm from the anal margin. Such fistulas usually track to the posterior midline. This is known as the Goodsall's rule.

In doing pericardiocentesis, what technique utilizes the shortest distance? A. Subxiphoid approach B. Transdiaphragmatic C. Left 4th ICS D. Left 5th ICS E. None of the above

185 A.

15 year old patient was bitten by a stray dog and sustained a 4cm lacerated wound in the middle 1/3 of the leg, lateral side. The following are true of the management for this patient except? A. Thoroughly clean the wound with water and antiseptic solution B. Suture the wound C. Give Anti-rabies vaccine and Ig D. Give tetanus prophylaxis E. None of the above

186 B. You do not suture the wound from dog bites, healing by secondary intention is preferred.

After an appendectomy, patient was given 3L of D5LR as fluid maintenance for 24 hours. How many kilocalories does it provide to the patient? A. 600kcal B. 700kcal C. 800kcal D. 300kcal E. 900kcal

187 A. SIMILAR TO PREVIOUS BOARD EXAM CONCEPT/PRINCIPLE.. 1 L of D5LR contains 50g glucose. 1 g glucose contain 4 kcal. 4x50= 200kcal per liter. Patient consumes 3 L so 200 x 3= 600 kcal

What is the most commonly used suture in circumcision? A. Silk B. Nylon C. Chromic D. Vicryl E. Prolene

188 C. SIMILAR TO PREVIOUS BOARD EXAM CONCEPT/PRINCIPLE.

8 year old female suffered a lacerated wound, 3cm in the left temporal area due to self-accident. Wound suturing was done. 2% lidocaine was used as local anesthetic. What is the toxic dose of lidocaine if the patient's weight is 25kg? A. 6.50 ml B. 6.35 ml C. 6.45 ml D. 6.25 ml E. 6.55 ml

189 D. SIMILAR TO PREVIOUS BOARD EXAM CONCEPT/PRINCIPLE.. toxic dose of lidocaine is 5mg/kg. 5mg/kg x 25kg= 125mg. 2% lidocaine is 20mg/ml so 125mg/20mg/ml= 6.25ml

In adults, Meckel's diverticulum usually presents as: A. Bleeding B. Obstruction C. Abdominal mass D. Post-prandial vomiting E. Jaundice

216 B. In children, bleeding is the most common symptom.

The law of Laplace states that as the diameter of the lumen of cavity increases, the wall tension also increases. Which segment of the GIT has the widest luminal diameter and is thus most prone to rupture and least prone to obstruction? A. Cecum B. Appendix C. Sigmoid colon D. Transverse colon E. Duodenum

19 A. Normal cecum has a luminal diameter of 7.5- 8.5cm and becomes at risk at 10cm. Sigmoid colon is the narrowest part and most vulnerable to obstruction and also most mobile and vulnerable to volvulus. Source: Schwartz’s Principles of Surgery 8th ed p 1015

A patient was brought to the emergency room 12 hours after ingestion of "liquid sosa". After history and PE, what should be done initially in the management? A. Induce emesis B. Neutralize with half-strength vinegar or citrus juice C. Do immediate esophagectomy D. Observe E. Do early endoscopy

190 E. SIMILAR TO PREVIOUS BOARD EXAM CONCEPT/PRINCIPLE.. If a patient presents within the first hour of ingestion, neutralization is attempted. Early endoscopy is recommended 12 to 24 hours after ingestion to identify the grade of the burn. The treatment of caustic lesions of the esophagus is determined by the extent of the injury and addresses the injuries that occur both in the acute and chronic phase.

30 minutes prior, an 18 year old female attempted suicide by ingesting hydrochloric acid. What should be done initially in your management? A. Induce emesis B. Neutralize with milk, egg whites, or antacids C. Do immediate esophagectomy D. Observe E. Do early endoscopy

191 B. SIMILAR TO PREVIOUS BOARD EXAM CONCEPT/PRINCIPLE.. If a patient presents within the first hour of ingestion, neutralization is attempted.

58 year old patient underwent cholecystectomy. 2 days post-operation, patient developed fever. There were no other associated symptoms. What is the most likely cause of the fever? A. Atelectasis B. UTI C. Surgical wound infection D. Nosocomial pneumonia E. Phlebitis

192 E. SIMILAR TO PREVIOUS BOARD EXAM CONCEPT/PRINCIPLE.. Fever in the first 3 days after surgery most likely has a noninfectious cause. Atelectasis is considered if fever developed within 24 hours. Fever developed 2 days post op int he case given so phlebitis is the most likely cause.

What is the most important component of healing in a large surface superficial wound? A. Collagen deposition B. Wound contraction C. Epithelialization D. Maturation E. Inflammation

193 C. SIMILAR TO PREVIOUS BOARD EXAM CONCEPT/PRINCIPLE.. If only the epithelium and superficial dermis are damaged, such as occurs in split-thickness skin graft donor sites or in superficial second-degree burns, then repair consists primarily of re-epithelialization with minimal or no fibroplasia and granulation tissue formation.

43 year old male is diagnosed with Papillary carcinoma of the thyroid gland. Lymph nodes in the central compartment and the jugular chains are positive and there is documented metastasis in the lungs. What is the cancer stage? A. I B. II C. III D. IV E. 0

194 B. Patients with well differentiated thyroid cancer < 45 years old is staged as follows: Stage I: Any T, Any N, M0; Stage II: Any T, Any N, Mi

22 years old female patient complained of amenorrhea and galactorrhea. Diagnostic work- ups revealed a pituitary tumor 0.7cm in size. How will you manage the patient? A. Observe B. Open craniotomy C. Transphenoidal surgery D. Bromocriptine therapy E. Craniectomy

195 D. pituitary tumors <1cm are considered as microadenoma while those that are > 1 cm are macroadenomas. Microadenomas are managed medically with bromocriptine.

There are how many parathyroid glands? A. 1 B. 2 C. 3 D. 4 E. 5

196 D. giveaway question last boards

What is the most important risk factor in the development of breast cancer? A. age B. sex C. Family history D. smoking E. Early menarche, late menopause

197 B. Gender is the most important risk factor in Breast cancer

The following are the nerves that are spared in doing Modified Radical Mastectomy except? A. Long thoracic nerve B. Thoracodorsal nerve C. Medial pectoral nerve D. Lateral pectoral nerve E. None of the above

198 E. All the given choices are spared in MRM

Widened mediastinum is a finding most commonly associated with what condition? A. Aortic dissection B. Pericardial effusion C. Pleural effusion D. Esophageal rupture E. Peptic ulcer perforation

199 A. Widened mediastinum is the most sensitive CXR finding suggesting tear of the aorta

Which of the following treatment used for the management of burn wounds can be absorbed systemically and cause metabolic acidosis? A. Silver nitrate B. Silver sulfadiazine C. Mafenide acetate D. A and B E. All of the above

2 C. Silver sulfadiazine â€" antimicrobial activity, soothing quality, inexpensive and easily applied, neutropenia Mafenide acetate â€" antimicrobial activity, painful, metabolic acidosis Silver nitrate â€" antimicrobial activity, hyponatremia, methemoglobinemia, black staining SIMILAR TO PREVIOUS BOARD EXAM CONCEPT/PRINCIPLE Source: Schwartz’s Principles of Surgery 8th ed p 202

A 42-year old man with gunshot wound to the RUQ and epigastric area arrives in the ER drowsy, pale with a BP = 80/60, HR = 120, RR = 35. Abdomen is distended, tense and with diffuse rebound tenderness. What is the most appropriate procedure? A. Laparotomy B. Stat CT scan of the abdomen C. Stat Diagnostic peritoneal lavage D. Stat FAST E. Observation

20 A. Patient is already unstable hence should be opened for exploratory laparotomy. Source: Topnotch handout on Surgery

A post-operative patient is to be given a penicillin- type of antibiotic for an abdominal infection. However, history revealed a strong allergy to this class of medications. Which among the following drugs should be MOST avoided as it possesses the highest cross-reactivity with penicillin? A. Cefazolin B. Clindamycin C. Metronidazole D. Imipenem + Cilastatin E. Sulbactam + Ampicillin

201 D. Among all antibiotics, carbapenems possess the highest cross-reactivity with penicillins.

A 4 kg-infant with 20% TBSA partial thickness burns on the trunk was admitted at the Burn Unit of your hospital. To accurately assess whether your fluid resuscitation is adequate, this infant's urine output should measure at least: A. 2 mL/hr B. 4 mL/hr C. 6 mL/hr D. 8 mL/hr E. 10 mL/hr

202 D. In burn patients, adequate fluid resuscitation of an infant achieves at least 2 mL/kg/hr of urine output.

A patient with a gunshot wound at the torso is suspected to have a tear of the descending thoracic aorta. For optimal exposure, what surgical approach is appropriate? A. Left anterolateral thoracotomy B. Left posterolateral thoracotomy C. Left anterolateral thoracotomy and median sternotomy with supraclavicular extension D. Bilateral anterolateral thoracotomy E. Median sternotomy

203 B. Because the descending thoracic aorta is located posteriorly, this structure is best approached using a left posterolateral thoracotomy.

A 43 year old female patient with a 1 cm palpable mass of the right breast for 5 years underwent core-needle biopsy which revealed invasive ductal carcinoma. Breast conservation surgery was done. Lymph nodes were negative for cancer cells. ER and PR tests were also negative. What is the appropriate next step? A. Hormone therapy B. Adjuvant chemotherapy C. Combined hormone therapy and chemotherapy D. Prophylactic BCS of the left breast. E. No more intervention is warranted. Careful follow-up is advised.

204 B. Adjuvant chemotherapy for early invasive breast cancer is considered for all node-positive cancers, all cancers that are larger than 1 cm in size, and node-negative cancers larger than 0.5 cm in size when adverse prognostic features are present.

The Sistrunk procedure is performed for: A. Mandibular masses B. Laryngeal cancer C. Thyroglossal duct cysts D. Cholesteatoma E. Nasal polyps

205 C. The Sistrunk procedure consist of en bloc cystectomy and excision of the central hyoid bone to minimize recurrence.

In a patient with severe aortic stenosis, the patient's native pulmonary valve is used to replace the stenotic aorta and a homograft is used in place of the pulmonary valve. The procedure described is: A. Norwood procedure B. Ross procedure C. Fontan procedure D. Blalock-Taussig procedure E. Brock's procedure

206 B. The Ross procedure may be indicated for younger patients who require aortic valve replacemnt and want to avoid the need for anticoagulation.

The triple layer descibed by Bassini in his method of inguinal hernia repair consist of: A. Skin, external oblique, internal oblique, B. External oblique, internal oblique, rectus abdominis C. Skin, rectus abdominis, external oblique D. Rectus abdominis, internal oblique, transversus abdominis E. Internal oblique, transversus abdominis, transversalis fascia

207 E. Bassini repair involves reconstruction of the posterior wall by suturing this triple layer medially to the inguinal ligament laterally and possibly the iliopubic tract.

The colon of a 55 year-old male was inadvertently perforated during a colonoscopy procedure. What segment is LEAST likely to be involved? A. Cecum B. Ascending colon C. Transverse colon D. Descending colon E. Sigmoid colon

208 E. The sigmoid colon is least to perforate but most likely to undergo volvulus. The cecum is the part of the colon most likely to perforate due to its thinner wall.

During work-up of a 40 year-old female patient presenting with virilizing features, an adrenal mass was identified by imaging studies. What is the single most important criterion to diagnose carcinoma in this patient? A. Irregular margins B. Size of 8 cm C. Heteregoneity D. Adjacent lympadenopathy E. Presence of hemorrhage within the tumor

209 B. The size of the adrenal mass on imaging studies is the sinbgle most important criterion to help diagnose malignancy.

Thyroglossal duct cysts are most commonly located A. On the anterior border of the sternocleidomastoid muscle B. In the midline at the level of the hyoid C. Over the medial clavicular head D. In the midline just superior to the thyroid gland E. Behind the base of the tongue near the foramen cecum

21 B. Thyroglossal duct cysts are the most commonly encountered congenital cervical anomalies. During the fifth week of gestation, the thyroglossal duct lumen starts to obliterate, and the duct disappears by the eighth week of gestation. Rarely, the thyroglossal duct may persist in whole, or in part.

Which statement is NOT correct regarding hypertrophic pyloric stenosis? A. HPS is never a surgical emergency. B. It can be accurately diagnosed by ultrasound. C. Affected patients present with non-bilious vomiting. D. It presents early in the neonatal period; 95% present within one week of life. E. It commonly affects first-born males.

210 D. HPS classically present between 3 and 6 weeks of age.

A suicidal patient drank a bottle of Liquid Sosa after having failed the physician licensure exam. You suspect esophageal perforation. As the surgeon on duty, you would approach this patient by a: A. Right thoracotomy B. Left thoracotomy C. Median sternotomy D. Median sternotomy and right thoracotomy E. Median sternotomy and left thoracotomy

211 A. In caustic injury, the midportion of the esophagus is most likely affected. This segment is best approached via a right thoracotomy.

A 65 year-old hypertensive male has developed “food fear†over the last three months. According to his daughter, he has also lost considerable amount of weight. No changes in bowel movement were noted. The patient is hysterically screaming in the emergency room because of his diffuse abdominal pain. However, on physical examination, you only noted minimal tenderness in the epigastric area. At this point, you strongly suspect: A. Perforated peptic ulcer B. Acute cholecystitis C. Acute pancreatitis D. Diverticulitis E. Acute mesenteric ischemia

212 E. Abdominal pain for which the severity is out of proportion to the degree of tenderness on examination is the hallmark of acute mesenteric ischemia.

Surgical wound infections usually become manifest ________ after surgery. A. 1-3 days B. 3-5 days C. 5-7 days D. 7-10 days E. 10-14 days

213 D.

A 58 year-old male patient has a 5 cm carcinoma of the anus with extension into the distal rectum. What procedure is most appropriate? A. Miles procedure B. Low anterior resection C. Extended low anterior resection D. High anterior resection E. Hartmann's procedure

218 A. Miles procedure or abdominoperineal resection (APR) involves removal of the entire rectum, anal canal, and anus with construction of a permanent colostomy from the descending or sigmoid colon.

Retraction is less problematic in a colostomy than an ileostomy because: A. A colostomy has a larger lumen and thicker wall. B. A colostomy is relatively stronger than an ileostomy. C. Stool is less irritating to skin than small intestinal fluids. D. A colostomy is easier to clean than an ileostomy. E. Movement of abdominal wall muscles does not affect the colostomy.

219 C. Parastomal hernia is the most common late complication of a colostomy.

A patient with hypertension is diagnosed with hyperaldosteronism. A CT scan shows bilaterally enlarged adrenals without a mass. The most appropriate next intervention is A. Unilateral adrenalectomy B. Bilateral adrenalectomy C. Selective venous catheterization D. Medical management E. Observation

22 C. If an adrenal tumor is not seen or if both adrenals are abnormal on CT scan, selective venous catheterization and adrenal vein sampling for aldosterone can help determine if the aldosteronoma is unilateral, because this can benefit from surgery. Bilateral adrenal tumors or adrenal hyperplasia are best managed medically.

A 45 year-old patient presents with a 2 cm ill- defined mass on his right ear that has been present for two years. The mass is cream-colored with pearly borders and visible small blood vessels. Which statement is true regarding this mass? A. Lymph node metastasis is usually present at the time of diagnosis. B. It has an excellent prognosis. C. This is best treated by wide radical resection. D. This type of skin cancer has both a radial and a vertical growth phase. E. Actinic keratosis is a precursor of this condition.

220 B. Basal cell carcinoma is the most common type of skin cancer. It is usually slow growing. Metastasis and death are extremely rare. Choice D is melanoma; choice E is squamous cell carcinoma.

A 34 y/o male was brought to ER d/t MVA. BP 130/90 PR 75 RR 10 , GCS 10 ( E2V4M4), there are multiple fractures on bilateral extremities. Cranial CT revealed 1.4 cm crescent shaped mixed densities noted on bilateral cerebral convexity. which of the following is the procedure of choice to be done at this point ? A. perform ER external ventricular drain insertion B. open craniotomy C. medical decompression with hypertonic saline D. emergency third ventriculostomy

221 B. immediate surgical intervention is necessary in acute subdural hematoma when the ff CT findings are noted: thickness >1cm, midline shift >5mm, or GCS drop by 2 or more points during the hospital stay. Burrholing or EVD is not ideal since in acute hematoma, the blood is thick and congealed already unlike in chronic ones. the question is what is the best intervention. however, it should be noted that medical decompression should be done at all points of therapeutic planning.

A 56 y/o male was brought to ER d/t sudden onset of severe headache, during the ER stay, he become lethargic. PE findings revealed (+) kernig sign., there is also bilateral CN VI palsy. MMT (L) 3/5; ® 5/5. what is the hunt and hess classification of this patient ? A. Class 1 B. Class 2 C. Class 3 D. Class 4 E. Class 5

222 C. class I - asymptomatic, or mild H/A and slight nuchal rigidity; II- Cr N palsy, moderate to severe headachr, nuchal rigidity; III- mild focal deficit, lethargy or confusion; IV- stuporous, moderate to severe hemiparesis, early decerebrate rigidity, 5- deep coma, decerebrate rigidity, moribound appearance.

Which of the following is the most effective adjuvant therapy in preventing recurrence of bladder cancer in situ after transurethral resection ? A. Alpha -interferon B. Bacille calmette Guerin C. Doxorubicin D. Mitomycin C instillation E. any of the above

223 C. based on statistics, intravesical BCG is the most effective agent. It is usually given in six weeks instillation, with a repeat course if a complete response is not attained.

what is the standard treatment for a patient with prostatic carcinoma with multiple retroperitoneal lymph node and documented vertebral metastasis? A. Radical prostatectomy with adjuvant chemotherapy B. Bilateral scrotal orchiectomy with Leuprolide C. neoadjuvant chemotherapy with radical prostatectomy and radiotherapy D. palliative therapy E. none of the above

224 B. the standard treatment for distant metastatic disease is androgen ablation therapy to lower serum testosterone. This is achieved by bilateral scrotal orchiectomy and/or administration of LHRH agonist.

which of the following is not part of cardinal signs of Kanavel tenosynovitis ? A. Pain on passive flexion B. Fusiform swelling of the fingers C. Tenderness along the tendon sheath D. None of the above

225 A. kanavel signs indicte suppurative tenosynovitis of the flexor tendon sheath. Clinical presentation include fingers held in flexion, fusiform digital swelling, tenderness of tendon sheath, pain on passive extension

what is the most common soft tissue sarcoma in the adults? A. Fibrosarcoma B. Rhabdomyosarcoma C. Liposarcoma D. lymphangiosarcoma E. Desmoid tumors

226 C.

Which of the following is true regarding Hurthle cell carcinoma of the thyroid?? A. It is a form of anaplastic thyroid cancer B. It metastasizes via the lymphatics to regional lymph node C. Treatment consists of near-total or total thyroidectomy D. Microscopically, it consists of clusters of cells separated by areas of collagen and amyloid E. all of the above

227 C. Hurthle cell cancer is a type of follicular cancer, but it tends to recur more often than the other types. It spreads hematogenously to distant sites. Amyloid deposits in the stroma of thyroid tumor are diagnostic of medullary carcinoma. The treatment of choice is near total thyroidectomy to facilitate later body scanning for metastasis and treatement with RAI.

What is the most serious complication of an end colostomy?? A. Bleeding B. Skin breakdown C. Parastomal hernia D. Colonic perforation during irrigation E. None of the above

228 C. The most frequent serious complication of end colostomies is parastomal herniation, which commonly occurs when the stoma is placed lateral to, rather than through, the rectus muscle

A 30 y/o male with a gunshot wound came to the emergency room. Patient was confused with BP of 80/60, HR 140. Based on the physical finding, how much is the estimated blood loss of the patient? A. <750 ml B. 750-1500ml C. 1500-2000 ml D. >2000ml E. none of the above

229 C. The patient presents with class III hemorrhage since the patient is confused, hypotensive and tachycardic. Class III hemorrhage results from blood loss of 1500-2000 ml or 30-40% blood loss.

A 60-year-old woman develops a right-sided colonic adenocarcinoma. She has a history of alleged colonoscopies; however, review of images from her most recent colonoscopy reveals a prominent fold in the region that subsequently developed cancer. Biopsy from this area would have most likely revealed which of the following? A. Tubular adenoma B. Hyperplastic polyp C. Peutz-Jeghers polyp D. Sessile serrated adenoma E. Inflammatory polyp

23 D. Sessile serrated adenomas can be very subtle and are easily missed on endoscopy. Under the microscope, they closely resemble hyperplastic polyps but show more complicated glands, often with “boot-shaped†configurations at the gland bases. Unlike tubular adenomas, they lack the overtly dysplastic epithelium and often do not form polypoid lesions. Hyperplastic polyps, Peutz-Jeghers polyps, and inflammatory polyps are not malignant precursors.

All of the following are true about branchial cleft cyst, except? A. It is derived from the branchial cleft apparatus that persists after fetal development B. 1st branchial cleft cyst and sinuses are associated intimately with the internal acoustic meatus and the parotid gland C. The 2nd branchial cleft cyst is found along the anterior border of SCM muscle D. The 3rd branchial cleft remnant courses posterior to the common carotid artery, ending in the pyriform sinus region E. none of the above

230 B. The 1st branchial cleft cyst and sinuses are associated intimately with the external acoustic canal and parotid gland.

9. A 50 y/o patient came to your clinic presenting with dysphagia. Manometric study was done, which revealed an elevated LES pressure, normal LES relaxation, and normal peristalsis in the esophageal body. What is your initial diagnosis? A. Hypertensive lower esophageal sphincter B. Achalasia C. Diffuse esophageal spasm D. Nutcracker esophagus E. none of the above

231 A. This result describes hypertensive lower esophageal sphincter. Achalasia would result to incomplete lower esophageal sphincter relaxation, aperistasis in the esophageal body, elevated LES pressure, and increased intraesophageal baseline pressures relative to gastric baselin. Diffuse esophageal spasm would result to simultaneous nonperistaltic contraction, repetitive and multipeaked contractions, spontaneous contractions, intermittent normal peristalsis. Nutcracker esophagus has mean peristaltic amplitude in the distal esophagus greater than or equal to 180, increased mean duration of contraction, and normal peristaltic sequence.

True about carcinoid tumor except? A. It is a firm, yellow, bulbar mass, most commonly seen in the appendix B. Most are located at the base of the appendix C. Tumor size of <1cm rarely result in extension outside the appendix or adjacent to the mass, therefore treatment is only appendectomy D. A and B E. B and C

232 B. Carcinoid tumors are usually located at the tip of the appendix. For tumors larger than 1-2 cm located at the base or with lymph node metastases, right hemicolectomy is indicated.

A 69 y/o male presents with a history of suddent onset of obstructive jaundice accompanied by recurrent abdominal pain and involuntary weight loss of 10%. CT scan was performed which reveals 3.9 cm mixed hypodensity lesion on the head of the pancreas. percutaneous biopsy was done revealing irregular, anaplastic, highly mitotic cells with glandular features. which of the following CT findings will preclude curative resection via whipples procedure? A. encasement of gastroduodenal artery B. lack of pulsation of the superior mesenteric artery C. encroachment of the plane between the tumor and the portal vein D. multiple matted lymphadenopathies in the celiac axis E. malignant refractory ascites.

233 B. preoperative CT findings that determine unresectability of a pancreatic head mass include encasement of the SMA, celiac axis and occlusion of the superior mesenteric vein or portal vein. Loss of plane b/w tumor and the vessels is not a contraindication for curative resection.

89 year old went for consult secondary to dysphagia. He already had significant weight loss because of food aversion secondary persistent reflux of swallowed food. PE of the neck shows bulging, soft, nontender mass on the right anterior cervical triangle. what is the best approach for this case ? A. endoscopy B. barium swallow C. CT scan of the head and neck D. therapeutic trial of PPI E. perform cricopharyngeal myotomy

234 B. this is a classic case of Upper esophageal diverticulum aka zenkers diverticulum. The best approach is to perform barium swallow for preop work up before going to surgical correction. Endoscopy is a relative contraindication because of the risk for rupture. PPI will give only slight therapeutic benefit. CT scan is not necessary since barium swallow will suffice.

A 48 y/o male presented to your clinic with obstructive form of jaundice. You opted to request for MRCP which reveal a case of type III choledochal cyst. What is the standard treatment for this case? A. Cholecystectomy with cyst excision B. transduodenal sphincteroplasty C. liver transplantation D. roux en y choledochojejunostomy E. all of the above

235 B. Type I - fusiform dilatation of the CBD - cholecystectomy, cyst excison and a roux en Y choledochojejunostomy. Type II -diverticulum of CBD - excision. Type III- choledochocele of the intraduodenal CBD - sphincteroplasty or choledochoduodenostomy. Type 4- caroli's disease, cystic dilatations of intrahepatic duct, - liver transplantation.

A 78 y/o male was rushed to ER d/t severe back pain associated pulsatile tender abdominal mass at the periumbilical area.he was hypotensive,tachycardic and had gradual deterioration of consciousness. he was immediately brought to OR for exploratory laparotomy. Which of the following is the most common surgical complication for this case ? A. renal failure B. ischemic colitis of the ascending colon C. anterior spinal artery syndrome D. acute leg ischemia E. infection

236 A. the most common surgical complication for emergency repair of AAA is renal failure accounting about 20-25% of all cases. Ischemic colitis usually involves the sigmoid colon. This is secondary to the ligation of IMA during surgery. Anterior spinal artery syndrome is secondary to occlusion of the artery of adamkiewicz. acute leg ischemia is d/t clamp injury of the iliac arteries

Which of the following is true about Paget’s disease of the breast? A. It presents as chronic, eczematous eruption of the nipple, but may progress to an ulcerated, weeping lesion B. It is usually associated with extensive LCIS and is not associated with invasive cancer C. Pathognomonic of this condition is the presence of small, dark vacuolated cells in the rete pegs of epithelium D. all of the above E. none of the above

237 A. Paget’s disease of the nipple is usually associated with extensive DCIS and is associated with invasive cancer. Pathognomonic of this condition is the presence of large, pale, vacuolated cells (Paget cells) in the rete pegs of the epithelium

A 26 y/o male complains of cough, upperback pain accompanied by dysphagia. Chest xray revealed 5 cm mass located at the retrocardiac area. what is the most likely diagnosis for this mass ? A. Ectopic thymoma B. Neurogenic tumor C. myxoma D. bronchogenic carcinoma E. none of the above

238 B. the most common posterior mediastinum mass is a neurogenic tumor.

Which of the following is not part of the pentalogy of Cantrell? A. Gastroschisis B. Diaphragmatic hernia C. Bifid sternum D. Absent pericardium E. VSD

239 A. omphalocoele and not gastroschisis is a component of cantrell pentalogy. Associated anomalies are rare in gastroschisis.

The most common type of hiatal hernia is A. Type I B. Type II C. Type III D. Type IV E. Type V

24 A. SIMILAR TO PREVIOUS BOARD EXAM CONCEPT/PRINCIPLE. There were 3 questions about esophageaWith the advent of clinical radiology, it became evident that a diaphragmatic hernia was a relatively common abnormality and was not always accompanied by symptoms. Three types of esophageal hiatal hernia were identified: (a) the sliding hernia, type I, characterized by an upward dislocation of the cardia in the posterior mediastinum; (b) the rolling or paraesophageal hernia (PEH), type II, characterized by an upward dislocation of the gastric fundus alongside a normally positioned cardia; and (c) the combined sliding-rolling or mixed hernia, type III, characterized by an upward dislocation of both the cardia and the gastric fundus. The end stage of type I and type II hernias occurs when the whole stomach migrates up into the chest by rotating 180° around its longitudinal axis, with the cardia and pylorus as fixed points. In this situation the abnormality is usually referred to as an intrathoracic stomach. In some taxonomies, a type IV hiatal hernia is declared when an additional organ, usually the colon, herniates as well. When radiographic examinations are done in response to GI symptoms, the incidence of a sliding hiatal hernia is seven times higher than that of a PEH. The PEH is also known as the giant hiatal hernia. Over time the pressure gradient between the abdomen and chest enlarges the hiatal hernia. In many cases the Type 1 sliding hernia will evolve into a type II mixed hernia. Type II Hernias are rare.

A 43 y/o male was rushed to the ER secondary to severe epigastric pain accompanied by black tarry stools. After stabilization with PPI. He underwent upper GI endoscopy and eventually classified as type 3 ulcer under modified Johnson classification. where is the most probable ulcer site and what is its correlated acid secretion status ? A. near GE junction, acid hypersecretion B. near GE junction, acid hyposecretion C. prepyloric area, acid hyposecretion D. prepyloric area, acid hypersecretion E. none of the above

240 D. type 1 -m/c type near angularis incisura, acid hyposecretion. Type 2- same as type 1 but with associated active or quiescent duodenal ulcer, acid hypersecretion. Type 4- near GE junction, acid hyposecretion. Type 5 -NSAID induced, nonspecific site.

2 day old infant was brought to the emergency room, infant has been noted to be feeding well but would always vomit a few minutes after breastfeeding. Patient noted to be slightly dehydrated, irritable and crying. Olive shaped mass was palpated by the physician at the ER, which of the following diagnostic tests would be a good initial test to request? A. barium swallow B. barium enema C. abdominal ultrasound D. scout film of the abdomen E. upper GI endoscopy under sedation

241 C. SIMILAR TO PREVIOUS BOARD EXAM CONCEPT/PRINCIPLE, initial non invasive test for hypertrophic pyloric stenosis, abdominal ultrasound can be used

50 year old patient chronically constipated presents with 2 episodes of bright red bleeding amount to 1 cup, this was present during episodes of bowel movements, the doctor performs digital rectal examination, all of the following are the locations of the hemorrhoidal cushions EXCEPT A. right posterolateral B. left lateral C. left posterolateral D. right anterolateral E. none (all are hemorrhoidal cushions)

242 C. SIMILAR TO PREVIOUS BOARD EXAM CONCEPT/PRINCIPLE, hemorrhoidal cushions left lateral and right anterior and posterior lateral

A 50 year old patient complaints of anal pain, occasional intermittent fever, purulent discharge per anus and draining purulent sinus that was found to be 5cm from the anal margin to the right anteriorly, the doctor knows that the draining sinus track will eventually end at which position? A. posterior right lateral B. posterior left lateral C. anterior midline D. posterior midline E. anterior right lateral

243 D. SIMILAR TO PREVIOUS BOARD EXAM CONCEPT/PRINCIPLE, goodsall's rule >3cm anterior, and posterior openings track curvilinear and eventually end up in the posterior midline

A 35 year old male comes in the out patient department with complains of a 3 year history of a back mass on the upper back region, he noted gradual changes in its size sometimes increasing sometimes shrinking in size with occasional pain and drainage of foul smelling small amount of yellowish discharge, the physician examines it closely and notes a central punctum, which of the following is the most likely diagnosis? A. sebaceous cyst B. epidermal inclusion cyst C. ganglion cyst D. trichilemmal cyst E. dermoid cyst

244 A. SIMILAR TO PREVIOUS BOARD EXAM CONCEPT/PRINCIPLE, hallmark given at that time was central punctum sebaceous cyst

50 year old female accidentally slipped but as she fell she used her right outstretched hand to break her fall as she fell in the forward position, soon after she developed pain near the anterior shoulder area, which of the following injuries was most likely sustained? A. rotator cuff tear B. clavicular fracture C. fracture of the midshaft of humerus D. fracture dislocation of the elbow E. supraspinatus tear

245 B. SIMILAR TO PREVIOUS BOARD EXAM CONCEPT/PRINCIPLE, almost the exact case outstretched hand, I was looking for colles fracture but there was none, clavicular was the next best answer

In relation to the above question, initial diagnostics revealed a linear nondisplaced fracture of the site of pathology, what is the most appropriate management of this case? A. open reduction external fixation B. closed reduction C. open reduction with intramedullary nailing D. arm sling application E. long arm cast

246 D. SIMILAR TO PREVIOUS BOARD EXAM CONCEPT/PRINCIPLE, nondisplaced linear fracture of clavicle armsling only

A 55 year old male patient with 5 year history of fairly controlled hypertension and diabetes and appendectomy 20 years ago came in with a chief complaint of abdominal pain characterized as generalized, intermittent accompanied by episodes of vomiting. There was also anorexia, passage of flatus, last bowel movement was noted to be 3 days ago, focused PE revealed (+) mild abdominal tenderness near the umbilical epigastric areas, (-) rebound tenderness, (-) pain blood on DRE, a simple abdominal xray supine and upright was seen, which of the following are the most likely findings? A. kidney shaped mass over the right upper quadrant B. bean shaped mass over the left upper quadrant C. multiple air fluid levels in stepladder appearance D. sentinel loop sign E. hyperlucency under the right hemidiaphragm

247 C. SIMILAR TO PREVIOUS BOARD EXAM CONCEPT/PRINCIPLE, in SBO, scout film shows multiple air fluid levels

A 30 year old male was involved in a gun shot incident, patient was rushed to the ER, BP was 80/50, pulse was 135, RR was 33, patient noted to be agitated, noted sucking chest wound on the right, decreased breath sounds on the right, tracheal deviation to the left, hyperresonant over the right hemithorax, which of the following is the best course of action? A. insert a chest tube over the right hemithorax B. do needle decompression over the right hemithorax C. start fluid resuscitation 2 large bore IV cannulas D. intubate and hook to mechanical ventilation E. refer for emergency pericardiostomy

248 B. SIMILAR TO PREVIOUS BOARD EXAM CONCEPT/PRINCIPLE, tension pneumothorax warrants immediate needle decompression

A 40 year old male smoker comes in with a 3 year history of a gradually enlarging right mass located near at the anterior inferior auricular area, patient comes in for consult due to persistence of mass, no other symptoms was noted which of the following is the best approach for diagnosis? A. core needle aspiration biopsy B. fine needle aspiration biopsy C. incisional biopsy D. excision biopsy E. Ultrasound

249 D. SIMILAR TO PREVIOUS BOARD EXAM CONCEPT/PRINCIPLE, parotid mass - exision biopsy, parotid pancreatic and hepatic masses usually does not warrant needle biopsy

Which of the following infections have been correlated with nasopharyngeal carcinoma? A. Herpes simplex virus B. Epstein Barr virus C. Cytomegalovirus D. Human immunodeficiency virus E. Human Papilloma Virus

25 B. There are plenty of ENT questions in our Surgery Exam. Risk factors for nasopharyngeal carcinoma include area of habitation, ethnicity, and tobacco use. There is an increased incidence of nasopharyngeal cancer in southern China, Africa, Alaska, and in Greenland Eskimos. A strong correlation exists between nasopharyngeal cancer and the presence of EBV infection, such that EBV titers may be used as a means to follow a patient’s response to treatment.

In relation to the above question, histopath examination of the mass would most likely reveal which of the following pathologies? A. mucoepidermoid carcinoma B. plemorphic adenoma C. adenoid cystic carcinoma D. warthin's tumor E. malignant lymphoma

250 B. SIMILAR TO PREVIOUS BOARD EXAM CONCEPT/PRINCIPLE, benign features, slow growing mass, no other symptoms parotid mass - pleomorphic adenoma

7 year old boy came to the ER with abdominal pain, abdominal pain has been on for a day characterized as generalized. Patient had history also of an upper respiratory tract infection and vomiting diarrhea 3 days prior to the onset of abdominal pain. On physical exam, mild cervical lympadenoapthy, there was direct tenderness on the umbilical and right lower quadrant areas with minimal rebound tenderness, there was abdominal guarding, DRE minimal pain or discomfort no blood, CBC revealed WBCs elevated at 14000, patient was slightly febrile at 37.8 degrees celsius, urinalysis RBC 0-1 WBC 0-1 bacterioa +2 Epithelial cells +3, no casts or crystals, what is the most likely initial impression? A. acute appendicitis B. acute mesenteric ischemia C. acute mesenteric lymphadenitis D. acute pancreatitis E. acute gastroenteritis

251 C. SIMILAR TO PREVIOUS BOARD EXAM CONCEPT/PRINCIPLE, very difficult because it could go anywhere as the case evolves, but best answer as of now is acute mesenteric lymphadenitis by history and PE

Compute for the caloric intake if a patient is placed on NPO and a total of 3L of D5W has been infused? A. 600 B. 800 C. 1000 D. 1200 E. 1500

252 A. SIMILAR TO PREVIOUS BOARD EXAM CONCEPT/PRINCIPLE., D5W contains 50g x3L = 150g glucose x 4 kcal/g = 600

A 70kg 50 year old male patient sustained the following burns as he was escaping from a burning building, partial thickness burns over the lower anterior right leg, full circumferential burns of the left leg, partial thickness burns over the anterior half of the trunk and posterior lower half of the back, how many mls of lactated ringers should be infused in the first 8 hours following parkland formula? A. 4220mL B. 5670mL C. 6000mL D. 7440mL E. 11340mL

253 B. SIMILAR TO PREVIOUS BOARD EXAM CONCEPT/PRINCIPLE. asked us to compute, whole left leg = 18, lower leg right anterior 4.5, anterior half trunk 9, posterior half trunk 9 = 40.5 x 70kg x4mL = 11340/2 =5670

A 23 year old male sustained a facial laceration over the forehead after a fist fight, which of the following will be used for closure of the wound? A. Nylon 4-0 B. chromic 2-0 C. Nylon 5-0 D. Silk 2-0 E. Silk-0

254 C. SIMILAR TO PREVIOUS BOARD EXAM CONCEPT/PRINCIPLE., 5-0 or 6-0 smaller suture sizes for the skin, generally should be nonabsorbable for trauma wounds

Based on the question above, the sutures on the face are expected to be removed within? A. 2 days B. 5 days C. 7 days D. 10 days E. 14 days

255 B. SIMILAR TO PREVIOUS BOARD EXAM CONCEPT/PRINCIPLE, there are increasing number of stem questions, face 3-5 days, scalp 7-10 days, trunk 7-10 days, arms and legs 10-14 days

Which of the following sutures are considered nonabsorbable? A. polyglactin B. polydioxone C. polypropylene D. chromic E. catgut

256 C. SIMILAR TO PREVIOUS BOARD EXAM CONCEPT/PRINCIPLE a lot of suture questions please review them, polyglactin or vicryl, polydioxone or PDS chromic and catgut are absorbable, polypropylene or prolene, polyamide nylon and silk are non absorbable

Compute for the maintenance fluid requirements to be given to a 30kg child surgical patient placed on NPO A. 1000ml B. 1500ml C. 1700ml D. 1800ml E. 2000ml

257 C. first 10kg = 100ml/kg = 1000, next 10-20kg = 50ml/kg = 500, each kg> 20 20ml/kg = 200 = 1700 in total

40 year old female patient underwent cholecystectomy for cholelithiasis, what is the class of surgical wound at which this is classified? A. clean (class 1) B. clean/contaminated (class 2) C. contaminated (class 3) D. dirty (class 4) E. class 1D

258 B. review examples of surgery of different classes, 1D if with prosthesis, breast and hernia 1, hollow viscus not colon = 2, contaminated penetrating trauma, enterotomy 3, diverticulitis perforated, traumatic, necrotizing wounds = 4

A 20 year old male patient was involved in a motorcycle accident and has lost consciousness, all of the following diagnostic procedures are considered vital EXCEPT? A. plain cranial CT scan B. lateral cervical xray C. chest xray D. pelvis A-P xray E. scout film of the abdomen

259 E. for blunt trauma, important xrays are the followig except scout film of the abdomen, FAST UTZ or CT scan is usually used for abdomen, cranial CT scan is important to rule out cranial injuires or bleed

A classic Le Fort Type I fracture involves the: A. Forehead B. Nose C. Maxilla D. Mandible E. Zygomatic

26 C. Le Fort I fractures occur transversely across the alveolus, above the level of the teeth apices. In a pure Le Fort I fracture, the palatal vault is mobile while the nasal pyramid and orbital rims are stable. The Le Fort II fracture extends through the nasofrontal buttress, medial wall of the orbit, across the infraorbital rim, and through the gomaticomaxillary articulation. The nasal dorsum, palate, and medial part of the infraorbital rim are mobile. The Le Fort III fracture is also known as craniofacial disjunction. The frontozygomaticomaxillary, frontomaxillary, and frontonasal suture lines are disrupted. The entire face is mobile from the cranium.

A 20 year old male patient sustained a lower abdomina stab wound, a diagnostic peritoneal lavage was done, which of the following findings are considered positive which warrants laparoscopy or exploration? A. RBC count >50,000/ml B. WBC count >5000/ml C. amylase >15IU/I D. alkaline phospatase >2IU/l E. bilirubin >1mg/dl

260 D. RBC>100,000, WBC >500, amylase>19, bilirubin >0.01

An alcoholic patient presents at the ER due to severe chest pain, back pain, and dyspnea. His relatives tell you that he has had several bouts of retching prior to the onset of symptoms. If you suspect this to be a case of esophageal perforation, what will be your diagnostic method of choice to demonstrate leak or extravasation? A. Chest X-ray B. Gastrografin swallow C. Endoscopy D. Barium swallow E. None of the above

261 B..Gastrografin(water-soluble) swallow is preferred in demonstrating leaks/extravasation

A modified radical mastectomy will include: A. Removal of all breast tissue, pectoralis major and minor, and axillary lymph node dissection B. Removal of all breast tissue, pectoralis fascia, and axillary lymph node dissection of level I and II C. Removal of all breast tissue, pectoralis minor muscle, and axillary lymph node dissection of level I and II D. Removal of all breast tissue, pectoralis fascia, and axillary lymph node dissection of level I, II and III E. Removal of breast tissue only

262 B. A refers to radical mastectomy. E is simple mastectomy

A 56 year old male patient presents to your clinic due to weight loss and presence a palpable, nontender, fixed lymph node located at the posterior triangle of the neck (level V). He has an unremarkable history other than smoking for approximately 30 years. Biopsy reveals squamous cell carcinoma. You suspect that this malignancy most likely originated from what region of the head and neck? A. Nasopharynx B. Oropharynx C. Laryngopharynx D. Oral Cavity E. Larynx

263 A. Oral cavity - I,II,III; Oropharynx, Laryngopharynx, Larynx - II, III, IV

Surgical treatment of gastric adenocarcinoma involves gastrectomy and lymphadenectomy. Some studies state that adequate lymphadenectomy requires a D2 dissection during gastric resection. Which of the following refers to the D2 group of lymph nodes? A. Nodes around the celiac axis, splenic hilum, and splenic artery B. Greater curvature perigastric nodes C. Lesser curvature perigastric nodes D. Hepatoduodenal and mesenteric root nodes E. Para-aortic and paracolic nodes

264 A. A is D2, B and C is D1, D is D3, E is D4

A 34 year old patient presents to your clinic due to an anterior neck mass. FNAB indicates follicular type and you suspect follicular adenoma so you perform thyroid lobectomy with isthmusectomy. Post-operative biopsy reveal capsular invasion. What will be your next step? A. Do nothing. It is most probably benign. B. Perform radioactive iodine ablation to destroy the remaining thyroid tissue C. Perform completion thyroidectomy D. Do conservative management and monitor Tg levels E. None of the above

265 C. Completion surgery to remove as much thyroid tissue as possible before radioactive iodine

Which of the following statements is true regarding benign liver lesions? A. Hemangioma requires excision even when asymptomatic due to high risk of rupture and bleeding B. Focal nodular hyperplasia lacks a central stellate scar on CT and appears cold on Tc scan C. Hepatic adenoma is associated with OCP use and has no risk of malignant degeneration D. A and B E. None of the above

266 E. Asymptomatic hemangioma requires no treatment; Focal nodular hyperplasia has a stellate scar and appears hot on scan with no risk of maliganant transformation. Hepatic adenoma is associated with OCP use and risk of maligant transformation

An anal fissure that is NOT associated with other conditions such as Crohn's disease or TB is most commonly located in what position? A. anterior midline B. posterior midline C. lateral position D. A and C E. A and B

267 B. Any other position is suspicious of an underlying etiology such as TB or Crohn's

You perform suturing on a traumatic laceration located on the patient's right lower leg. You will inform the patient to come back to the OPD for suture removal after: A. 3-5 days B. 5-7 days C. 7-10 days D. 10-14 days E. 1-2 days

268 D. 3-5 for face; 5-7 for scalp; 10-14 for trunk and extremities

Liver abscesses may either be bacterial or amebic in etiology. Which of the following supports the diagnosis of the latter? A. Single abscess in the right lobe B. Characterized by fever and jaundice C. Usually caused by an ascending biliary infection D. B and C only E. All of the above

269 A. B and C are more consistent with bacterial abscess

The yearly mammogram of a 65-year-old postmenopausal woman shows an irregular area of microcalcification that has grown in size compared with her mammogram from 2 years ago. She missed her mammogram last year. Physical examination is unrevealing without lymphadenopathy or nodularity in the breasts. You refer her to a surgeon, and a 2-cm invasive ductal carcinoma is removed from her left breast. Sentinel node biopsy shows two positive lymph nodes, and axillary dissection reveals five additional positive nodes. The tumor expresses the estrogen receptor (ER ). Which of the following interventions would increase her chance of cure? A. Chemotherapy followed by hormonal therapy B. Radiation therapy C. Total mastectomy D. Hormonal therapy alone E. High-dose chemotherapy with stem cell support

27 A. This patient is at high risk of recurrence by virtue of having disease in her lymph nodes. Radiation therapy or surgical excision of the breast cancer usually provides local control and adequate protection against recurrence of disease in the tumor bed. However, micrometastatic disease can only be addressed through the administration of systemic therapy such as cytotoxic chemotherapy. In patients with hormonally sensitive tumors (e.g., estrogen- receptor positive), hormonal therapy has been shown to be very effective in the prevention of disease recurrence after chemotherapy. Randomized clinical trials have shown that lumpectomy and radiation therapy is equal to a total mastectomy in terms of overall survival. For tumors that can be completely removed with adequate margins (10 mm), lumpectomy offers better cosmetic and psychological results, is less invasive, and allows for faster recovery time.

A 65 kg 32 year old male patient suffers from extensive flame deep partial thickness burns involving the anterior trunk, anterior right and anterior left thigh, perineum, and right forearm. How will you give your fluid resuscitation? A. 4225 during the first 24 hours B. 4550 mL during the first 12 hours and 4550 mL in the next 12 hours C. 4550 mL during the first 8 hours and 4550 mL in the next 16 hours D. 4225 mL during the first 12 hours and 4225 mL in the next 12 hours E. 4225 mL during the first 8 hours and 4225 mL in the next 16 hours

270 E. %TBSA x 4ml x kg; 1/2 in first 8 hours and 1/2 next 16 hours

You are about to assist on a surgical repair of a femoral hernia. In order to impress your consultant, you read up on the topic. Which of the following is true of femoral hernia? A. A Bassini repair is most often used and it involves reapproximation with the Cooper's ligament B. The most common type of hernia in women is femoral hernia C. Femoral hernias are located lateral to the femoral vein D. Management of femoral hernia usually involves surgery due to high risk of strangulation E. All are correct

271 D. McVey repair is usually done. Although femoral hernia is most commonly found in women, the most common type of hernia in women is still indirect hernia. Femoral hernia is medial to the femoral vein

An 11 month old infant is brought to the ER by her mother due to complaints of bilious vomiting and irritability. Further history reveals that the infant has poor appetite and assessment of growth reveals failure to thrive. Abdominal radiography reveals dilated proximal bowel and paucity of air in the distal bowel. Which is the most likely diagnosis? A. Intussusception B. Malrotation of midgut volvulus C. Duodenal atresia D. Hirschprung disease E. Meconium ileus

272 B. Primary consideration in bilious vomiting is midgut volvulus. The abdominal radiograph finding described is characteristic but not specific for the condition.

A 27 year old male patient in a vehicular crash is brought to the ER, You assess that the patient is hypotensive and you suspect blunt abdominal trauma. You decide to perform a diagnostic peritoneal lavage. Where you you place the catheter? A. 2 cm lateral and left to the umbilicus B. midline infraumbilical C. midline supraumbilical D. 2cm lateral and right to the umbilicus E. None of the above

273 B. Supraumbilical approach for pregnant and patients with unstable pelvis/suspected pelvic hematoma. Infraumbilical for all others

A male patient undergoes surgery for acute appendicitis. Intraoperatively, an incidental finding of a yellowish mass is found located on the appendiceal base measuring 1.8 cm. What is the next step in the management? A. Perform an appendectomy B. Perform an appendectomy plus resection of terminal ileum C. Perform a right hemicolectomy D. Do not proceed with the surgery and have the patient undergo chemotherapy post-op. E. None of the above

274 C. Right hemicolectomy for >2 cm, or if located at the appendicial base or affecting ileum.

What is the preferred site for kidney transplant? A. Where the diseased kidney is originally located B. Intraperitoneal at the level of L1 C. Retroperitneal at the level of L1 D. Extraperitoneal iliac fossa E. None of the above

275 D.

A 72 year old male patient presents at the ER with a sudden onset of abdominal distention, vague abdominal pain, and failure to pass stool or flatus. There is a history of constipation but the rest is unremarkable. Abdominal radiograph reveals a bent inner tube narrowing into a bird's beak. How will this patient be managed? A. Perform an endoscopic decompression B. Perform emergent surgery C. Do watchful waiting D. Administer neostigmine to increase intestinal motility E. None of the above

276 A. A case of sigmoid volvulus. Treatment for sigmoid volvulus is endoscopic decompression. Treatment for cecal volvulus is surgery

Extensive introduction of bacteria into a normal sterile area or gross spillage of viscus contents belong to what classification of surgical wound? A. Clean B. Clean/Contaminated C. Contaminated D. Dirty

277 C. Review types of surgical wound

Which of the following hormones becomes elevated 3 to 4-fold as a response to injury and returns to baseline after 24-48 hours? A. Cortisol B. Growth Hormone C. Epinephrine D. Glucagon E. Insulin

278 C. Catecholamines - elevated for 24-48 hours; Cortisol may remain elevated in up to 4 weeks in burn patients

You are managing a patient at the ER who presented with hematochezia and hemodynamic instability and a rapid work-up is imperative. You suspect a lower GI bleed. After initial resuscitation and stabilization, what will be the first thing you do in working up the patient? A. Do an urgent colonoscopy B. Perform an angiography with tagged RBC scan C. Do small bowel studies D. Do a nasogastric tube aspiration and urgent upper endoscopy if positive E. None of the above

279 D. Even when suspecting lower GI bleed, the first step is to rule out upper GI bleed because it's the most common cause

The Norwood procedure is used in the treatment of A. Coarctation of the aorta B. TGA C. Hypoplastic left heart syndrome D. Truncus Arterious E. TAPVC

28 C. Hypoplastic left heart syndrome - Norwood + Glenn + modified Fontan; TGA - palliative: Rastelli, arterial switch: Jenning, atrial switch:Senning/Mustard; TOF - palliative: Blalock-Taussig shunt

In the management of small bowel obstruction, conservative management is the initial recommendation for which of the following EXCEPT? A. Carcinomatosis B. Partial Bowel Obstruction C. Obstruction in the early post-operative period D. Intestinal obstruction due to Crohn's disease E. None of the above

280 E. All cases are initially treated conservatively.

A patient presented to the ER with a temperature of 39 C ,BP of 110/70, a heart rate of 115 bpm and a respiratory rate of 23. There was note of a CVA tenderness on the right. CBC showed leukocytosis at 17,000/uL and urinalysis showed pyruria of 30- 35 pus cells/hpf. Which of the following best describes the patient's condition? A. SIRS B. Sepsis C. Septic shock D. Refractory septic shock E. None of the above

281 B. Sepsis = SIRS + identifiable source of infection. SIRS: 1) temp of >38 or <36 2) HR >90, 3) RR >20 4) WBC >12,000 or <4000 or>10% band

A 24 year old male was involved in a motor vehicular accident (MVA). On evaluation, he was noted to have 6 rib fractures. How much is the expected blood loss from this kind of injury? A. 300-500 mL B. 1500-1800 mL C. 2000-2200 mL D. 600-1200 mL E. 100-200 mL

282 D. For each rib fracture there is approximately 100-200 mL of blood loss. For tibial fracture 300-500 mL, for femur fractures 800-1000 mL, and for pelvic fractures >1000 mL

A 60 year old male, asymptomatic and a known diabetic, underwent an ultrasound of the whole abdomen as part of his executive checkup. There were noted findings of cholelithiasis in the ultrasound. What will be the best treatment option for this patient? A. Dietary modification B. Ursodeoxycholic acid C. Elective laparoscopic cholecystectomy D. Both A and B E. None of the above

283 C. An elective lap chole is indicated for individuals who are diabetic even if they are asymptomatic

A 35 year old construction worker sustained a closed head injury. Moments later, a clear liquid is seen draining from his nose. Which of the following tests is most sensitive in determining if the liquid is a cerebrospinal fluid (CSF)? A. Glucose level B. Protein electrophoresis C. Beta transferrin D. Halo test E. Fluid:serum LDH level

284 C. for initial screening, a halo test can be performed. A drop of fluid is allowed to fall on an adosrbent surface such as a facial tissue. If blood is mixed with CSF, the drop will form a double ring with a darker center spot surrounded by a light halo of CSF. If it is indeterminate, the fluid can be sent for beta transferrin setting which will be only positive if CSF is present

Panendoscopy with blind biopsies are done for patients with evidence of cervical lymph node metastasis without clinical or radiologic evidence of a primary tumor. Which of the following sites is sampled in order to search for the primary source? A. Base of the tongue B. Nasopharynx C. Tonsillar fossa D. Pyriform sinus E. All of the above

285 E. 70% of unknown primaries with cervical node metastasis is located above the clavicle hence a focused search on the said locations is warranted

A 45 year old female experieced a sudden onset of painless loss of vision over her left eye. A central retinal artery occlusion (CRAO) is the primary impression of the attending Ophthalmologist. Which of the following statement does not describe the said condition? A. A foveal cherry red spot is characteristically seen on fundoscopic examination B. A relative afferent pupillary defect can precede the fundal abnormalities by an hour C. Irreversible damage can occur after 90 minutes of complete CRAO D. Systemic anticoagulation is the cornerstone of treatment E. IV acetazolamide can be given to reduce IOP thereby increasing retinal perfusion

286 D. Systemic anticoagulations are generally not employed in the treatmet of CRAO. Sudden decrease in intraocular pressure resulting in increased retinal perfusion can be achieved with anterior chamber paracentesis and intravenous acetazolamide. This is particularly indicated in embolic central retinal artery occlusion. Inhaled oxygenâ€"carbon dioxide mixture induces retinal vasodilation and increases the PO2 at the retinal surface. Thrombolytic therapy, infused directly into the ophthalmic artery or administered systemically, continues to be evaluated.

Amebic liver abscess of the left lobe should be primarily aspirated because of risk of rupture with concomittant involvement of what structure? A. Gallbladder B. Duodenun C. Abdominal aorta D. Diaphragm E. Pericardium

287 E. Aspiration of the abscess is rarely needed and should be reserved for patients with large abscesses, abscesses that do not respond to medical therapy, abscesses that appear to be superinfected, and abscesses of the left lobe of the liver that may rupture into the pericardium.

A 76 year old male, presented to the ER due to sudden onset of severe abdominal pain. He has been having recurrent burning epigastric pain for the past 2 weeks for which he self medicated with Ibuprofen as a pain reliever. He is a known hypertensive and diabetic and claims to be compliant with medications. VS are as follows: BP 80/50, PR 133 RR 23 T 37.3. Abdominal examination revealed a rigid abdomen, with diffuse tenderness. A scout film of the abdomen was done which showed evidence of pneumoperitoneum. The primary impression of the attending physician is a perforated duodenal ulcer. What is the procedure of choice for this patient? A. Vagotomy + Drainage B. Vagotomy + Antrectomy C. Graham Patch closure only D. Graham patch with highly selective vagotomy E. Oversew with Vagotomy + Drainage

288 C. Simple patch closure alone should be done in patients with hemodynamic instability and/or exudative peritonitis signifying a perforation >24 hours. In a relatively low risk patient addition of HSV may be considered due to negligible mortality with this approacj

Which of the following drugs has the strongest causal relationship with retroperitoneal fibrosis? A. Esomeprazole B. Paroxetine C. Methysergide D. Clofazimine E. Valsartan

289 C. The strongest case for a causal relationship between medication and retroperitoneal fibrosis is made for methysergide. Other medications that have been linked to retroperitoneal fibrosis include beta blockers, hydralazine, -methyldopa, and entacapone. The retroperitoneal fibrosis regresses on discontinuation of these medications.

The most useful radiologic test for diagnosing acute cholecystitis: A. ultrasound B. CT-scan C. HIDA scan D. MRI

337 A. The sonographic Murphy's sign: focal tenderness over the gallbladder when compressed by the sonographic probe. CT scan is a less sensitive test for diagnosing acute cholecystitis. A normal HIDA scan excludes acute chole.

An aortic dissection that extends from the left subclavian artery to the aortic bifurcation is a A. DeBakey Type I dissection B. DeBakey Type II dissection C. DeBakey Type IIIa dissection D. DeBakey Type IIIb dissection E. DeBakey Type IV dissection

29 D. Dissections are categorized according to their anatomic location and extent to guide treatment. The two traditional classification schemes that remain in common use are the DeBakey and the Stanford classification systems. In their current forms, both of these schemes describe the segments of aorta that are involved in the dissection, rather than the site of the initial intimal tear. The main drawback of the Stanford classification system is that it does not distinguish between patients with isolated ascending aortic dissection and patients with dissection involving the entire aorta. DeBakey type I: involves ascending and descending aorta (= Stanford A) DeBakey type II: involves ascending aorta only (= Stanford A) DeBakey type III: involves descending aorta only, commencing after the origin of the left subclavian artery (= Stanford B). Type III dissections are further divided into IIIa and IIIb. DeBakey Type IIIa refers to dissections that originate distal to the left subclavian artery but extend proximally and distally, mostly above the diaphragm. DeBakey Type IIIb refers to dissections that originate distal to the left subclavian artery, extend only distally, and may extend below the diaphragm

A 24 year old baseball player was accidentally struck by a bat in the face during practice. Upon examination, the ER officer assessed the presence of a septal hematoma. Which of the following is the most appropriate treatment for this case? A. Incision and Drainage B. Observation C. Aspiration D. Operative repair of the fracture E. None of the above

290 A. The nose is the most common facial fracture. It is important to perform an intransal examination to determine whether a septal hematoma is present. If present a septal hematoma must be incised, drained and packed to prevent pressure necrosis if the nasal septum and long term midvault collapse

Which of the following statements does not describe the Hereditary Nonpolyposis Colon Cancer (HNPCC) or Lynch Syndrome? A. It is more common than Familial Adenomatous Polyposis (FAP) B. Genetic defects arise from errors in mismatch repair C. Cancer appears in the proximal colon more often than in sporadic colorectal cancer D. It is inherited in an autosomal dominant pattern E. It is also asscoiated with extracolonic malignancies, with ovarian cancer being the most common

291 E. HNPCC is more common than FAP, but is still extremely rare (1 to 3%). The genetic defects associated with HNPCC arise from errors in mismatch repair. HNPCC is inherited in an autosomal dominant pattern and is characterized by the development of colorectal carcinoma at an early age (average age: 40 to 45 years) Cancers appear in the proximal colon more often than in sporadic colorectal cancer and have a better prognosis regardless of stage. HNPCC also may be associated with extracolonic malignancies, including endometrial, which is most common

A 55 year old woman, menopause for 2 years, was tested positive for BRCA1 gene mutation. She sought medical advice regarding her risk of acquiring breast cancer. Which of the following is the best recommendation? A. Mammogram every 6 months alone B. Mammogram every 6 months with Tamoxifen C. Prophylactic bilateral mastectomy alone D. Prophylactic bilateral mastectomy, followed by prophylactic BSO E. None of the above

292 D. For postmenopausal BRCA1 and BRCA2 mutation carriers who have not had a mastectomy, it may be advisable to avoid hormone replacement therapy, because no data exist regarding the effect of the therapy on the penetrance of breast cancer susceptibility genes. Prophylactic oophorectomy is a reasonable prevention option in mutation carriers. The American College of Obstetrics and Gynecology recommends that women with a documented BRCA1 or BRCA2 mutation consider prophylactic oophorectomy at the completion of childbearing or at the time of menopause

What is the primary treatment for oat cell carcinoma of the lung? A. Surgery alone B. Surgery followed by chemotherapy C. Immunotherapy D. Chemotherapy and radiotherapy E. Radiotherapy alone

293 D. Oat cell or small cell carcinoma accounts for 20% of lung CA and generally is not treated surgically due to its aggressive nature and widespread metastasis. Regardless of stage and presentation, treatment is primarily chemotherapy and radiation.

A 56 year old male presented with a gradually enlarging anterior neck mass accompanied by hoarseness and dysphagia. FNAB was done which revealed sheets of infiltrating neoplastic cells separated by collagen and amyloid. What is the best treatment option for this patient ? A. Subtotal thyroidectomy B. RAI C. External beam radiotherapy D. Total thyroidectomy alone E. Total thyroidectomy + routine bilateral central neck node dissection

294 E. Dx: Medullary Thyroid Carcinoma. Total thyroidectomy is the treatment of choice for patients with MTC because of the high incidence of multicentricity, the more aggressive course, and the fact that 131I therapy usually is not effective. Central compartment nodes frequently are involved early in the disease process, so that a bilateral central neck node dissection should be routinely performed.

What is the most common complication seen in adults with Meckel's diverticulum? A. Diverticulitis B. Obstruction C. Bleeding D. Perforation E. None of the above

295 B. Intestinal obstruction is the most common presentation in adults with Meckel's divertiula. Bleeding is the most common presentation in children with Meckel's diverticula

A 40 year old female, asymptomatic with no known co-morbids, was found to have elevated serum calcium on her routine pre-employment check up. A serum PTH was done which showed elevated levels. The impression of the physician is a primary hyperparathyroidism. Which of the following is an indication for parathyroidectomy in an asymptomatic individual ? A. Serum calcium >1 mg/dL above the upper limits of normal B. Creatinine clearance reduced by 30% C. Age less than 50 D. Elevated 24 hour urinary calcium (>400 mg/day) E. All of the above

296 E. All of the aforementioned are indications for parathyroidectomy in asymptomatic individuals. Other indications would include: life threatening hypercalcemic crisis, (+) kidney stones, decreased BMD (>2.5 SD)

Which of the following types of renal calculi will not be visualized on CT stonogram? A. Indinavir B. Uric acid C. Calcium oxalate D. Magnesium ammonium phosphate E. Cysteine

297 A. Calcium and struvite-containing stones often are visible on plain radiographs, but CT scans will demonstrate all calculi except those composed of crystalline-excreted indinavir, an antiretroviral medication.

Absolute contraindications for cholecystectomy include? A. uncontrolled coagulopathy B. end stage liver disease C. both D. neither

338 C. Schwartz p.1151

In patients with normal preoperative values, blood loss of up to ___ can be replaced with crystalloid solution: A. 5% B. 10% C. 15% D. 20% E. 25%

339 D. Schwartz p.79

A 1 year old boy presented with a foul smelling, grayish yellow aural discharge over the left ear accompanied by hearing loss. The attending ENT diagnosed the patient with suppurative otitis media. If left untreated which of the following is the most common intracranial complication of this disease? A. Brain abscess B. Otitic hydrocephalus C. Meningitis D. Lateral sinus thrombophlebitis E. Petrositis

298 C. the most common intracranial complication of suppurative OM is meningitis

A skin biopsy obtained from 41 year old woman showed findings consistent with a malignant melanoma. The malignant cells were demonstrated to have invaded up to the layer of the papillary dermis. What is the Clark's level of staging for this case? A. I B. IV C. III D. V E. II

299 E. Clark's level I: superficial to the basement membrane, II: up to papillary dermis, III: up to papillary/reticular dermis junction, IV: reticular dermis, V: up to subcutaneous fat

A 70 year old male incurred 20% TBSA burns. If PE revealed normal and stable vital signs, how much fluids should be given to him in the first hour? A. 250cc B. 350cc C. 450cc D. 550cc E. 1 L fast drip

3 B. Parkland formula = 4ml/kg/%burned ½ given in the first 8hrs then the next half given over the next 16hrs. 70x20x4/2/8 = 350cc Source: Schwartz’s Principles of Surgery 8th ed p 200

The treatment of stage II pressure sores is A. Local wound care only B. Extensive débridement and local wound care C. Direct closure D. Skin grafting E. Observation

30 A. Stage I and II ulcers are treated conservatively with dressing changes and basic pressure ulcer prevention strategies as already discussed. Patients with stage III or IV ulcers should be evaluated for surgery.

Which valve is primarily affected in Ebstein's anomaly? A. Aortic B. Mitral C. Pulmonary D. Tricuspid E. Both A and C

300 D. The predominant maldevelopment in this lesion is the inferior displacement of the tricuspid valve into the right ventricle

A 45 year old known duodenal ulcer patient undergoes laparotomy for signs of peritonitis more than 24 hrs after onset of acute symptoms. At surgery there is massive peritoneal contamination and the patient is hypotensive.what is the most appropriate treatment for this patient ? A. hemigastrectomy with truncal vagotomy B. highly selective vagotomy C. gastrojejunostomy D. Graham’s patching with peritoneal toilette E. Bilroth II gastrectomy

301 D. Simple patch closure alone should be done in patients with hemodynamic instability and or exudative peritonitis. Schwartz 9th ed p 921.

15. salivary gland tumor which has significant propensity to invade adjacent nerves and progress in a retrograde fashion to the central nervous system A. mucoepidermoid CA B. neurilemmoma C. squamous cell CA D. adenoid cystic CA E. Warthins tumor

302 D. adenoid cystic carcinoma, which has a propensity for neural invasion is the second most common salivary gland carcinoma in adults. Scwartz 9th ed page 507

A 50 y/o female whoe practices reverse smoking presents with a 2x2cm ulcer with heaped-up border at the R floor of the mouth of 10 months duration. Pre showed 1x2xm submandibular mass on the right together with several enlarged discrete masses on the right upper jugular area altogether measuring 3x4cm.what is the most appropriate neck dissection should be done in this patient? A. Radical neck dissection B. Lateral neck dissection C. Supraomohyoid neck dissection D. Modified neck dissection E. posterolateral neck dissection

303 D. B,C,E are Selective node dissection done only in patients who are clinically negative neck nodes. This is contrary to our patient. MRND has a better functional outcome compared to radical neck dissection.

. The most important criteria to distinguish a malignant adrenal incidentaloma from a benign one on imaging studies is: A. tumor size B. irregularity of margins C. presence of adjacent lymphadenopathy D. tumor heterogeneity E. vascular invasion

304 A. Determination of the malignant potential of an incidentaloma is related to size. Lesions >6cm have an appropriate risk of malignancy of about 35%. Scwartz 9th ed p1401

A 48 y/o male presented to your clinic with obstructive form of jaundice. You opted to request for MRCP which reveal a case of type III choledochal cyst. What is the standard treatment for this case? A. Cholecystectomy with cyst excision B. transduodenal sphincteroplasty C. liver transplantation D. roux en y choledochojejunostomy E. all of the above

305 B. Type I - fusiform dilatation of the CBD - cholecystectomy, cyst excison and a roux en Y choledochojejunostomy. Type II -diverticulum of CBD - excision. Type III- choledochocele of the intraduodenal CBD - sphincteroplasty or choledochoduodenostomy. Type 4- caroli's disease, cystic dilatations of intrahepatic duct, - liver transplantation.

A 35 y/o male was brought to ER secondary to Right lower quadrant pain and fever. PE reveals direct and rebound tenderness on RLQ. During the operation, a firm , yellow bulbar mass measuring 1.3 cm located at the mid appendiceal area was noted along with congested appendix. what is the next step in the management? A. appendectomy B. right hemicolectomy C. appendectomy with lymph node dissection D. tumor resection with exploration of small bowels E. none of the above

306 A. treatment of appendiceal carcinoids depends on the size of the mass and its location. Tumors <1cm, appendectomy is recommended. For those >2.0 cm, the treatment is right hemicolectomy. For tumors 1-2cm in size, resection is dependent on the location. mid and distal appendiceal area tumors- appendectomy. while those located at the base or those with lymph node metastasis, right hemicolectomy is warranted. Scwartz 9th ed p 1088.

A 65 y/o male was referred secondary to incidental finding of a 3cm mass located at the left upper lobe. Follow up CT scan revealed microcalcification but with an absence of lymph node metastasis. Patient is a smoker with a 35 pack years history. what is the next best step in the managment ? A. CT guided biopsy B. observation with repeat CXR after 3-6 months C. excision of the mass D. right upper lobe lobectomy E. radiation therapy

307 D. malignancy risk is higher because of his smoking history, therefore, observation is not indicated. Currently the patient is on stage 1 of the disease. biopsy has clinically defined benefit over direct surgical intervention.

which of the following is true about the Zone I neck injury: A. Refers to the area between the angle of mandible and cricoids B. Notorious for occult hemothorax C. Must be explored liberally because of high rate of occult injury D. All of the above

308 B.

How many lymph node are required to be dissected in axillary lymph node disection in breast cancer? A. 8 B. 9 C. 10 D. 11 E. 12

309 C. accurate predictions regarding the occurrence of distant metastasis were possible after resection of 10 or more level 1 and level II axillary LN. Schwartz 9th ed page 451

The most common location for a congenital diaphragmatic hernia is A. Left posterolateral B. Right posterolateral C. Left anteromedial D. Right anteromedial E. Right anterolateral

31 A. SIMILAR TO PREVIOUS BOARD EXAM CONCEPT/PRINCIPLE. The most common variant of a congenital diaphragmatic hernia (CDH) is a posterolateral defect, also known as a Bochdalek’s hernia. This anomaly is encountered more commonly on the left (80 to 90% of cases).

which of the following is the most common site of metastasis in breast carcinoma? A. lungs B. bone C. Liver D. brain E. Spleen

310 B. the following are the most common site of metastasis in breast cancer in descending order: bone, lungs, pleura, soft tissue, liver. Schwarts 9th ed page 443

which of the following will have the highest Resting energy expenditure ? A. Severe sepsis B. rhabdomyolysis C. Intraabdominal surgery D. Major burns E. Starvation

311 D. Schwartz 9th ed page 36.

Tilaux sign is correlated to what abdominal pathology ? A. Omental cyst B. Abdominal wall hematoma C. Mesenteric cyst D. GIST E. Choledochal cyst

312 C. tilaux sign is positive if the abdominal mass is permissive in lateral direction but restricts movement in superoinferior direction. This is highly correlated with mesenteric cyst.

Which of the following risk factor will have the highest association with gallbladder carcinoma ? A. 1cm polyp B. Cholesterol stones C. Porcelain gallbladder D. Choledochal cyst E. Primary sclerosing cholangitis

313 B. the most important risk factor for gallbladder carcinoma is the presence of cholelithiasis. Up to 95% of carcinoma have gallstones. Schwarts 9th ed p1160. ( SIMILAR TO PREVIOUS BOARD EXAM CONCEPT/PRINCIPLE)

A 69 y/o male presents with a history of suddent onset of obstructive jaundice accompanied by recurrent abdominal pain and involuntary weight loss of 10%. CT scan was performed which reveals 3.9 cm mixed hypodensity lesion on the head of the pancreas. percutaneous biopsy was done revealing irregular, anaplastic, highly mitotic cells with glandular features. which of the following CT findings will preclude curative resection via whipples procedure? A. encasement of gastroduodenal artery B. lack of pulsation of the superior mesenteric artery C. encroachment of the plane between the tumor and the portal vein D. multiple matted lymphadenopathies in the celiac axis E. malignant refractory ascites.

314 B. preoperative CT findings that determine unresectability of a pancreatic head mass include encasement of the SMA, celiac axis and occlusion of the superior mesenteric vein or portal vein. Loss of plane b/w tumor and the vessels is not a contraindication for curative resection.

Inadequate supplementation of this nutrient is responsible for the development of diversion colitis? A. butyrate B. Glutamine C. arginine D. Palmitate E. MCT

315 A. the metabolism and viability of colonic cells is dependent on supply of short chain fatty acids ( butyrate)

what is the recommended age for surgical repositioning of an undescended testis? A. 1 y/o B. 2y/o C. 3y/o D. 4y/o E. 5y/o

316 B. Scwartz 9th ed page 1446. 2 years old is the recommended age for orchiopexy.

which of the following is not true about hepatic injury secondary to blunt trauma? A. Delineating the source of hepatic hemorrhage can be done by pringle manuever B. Transfusion of 4 units of pRBC in 24 hours is an indication for hepatic angiography C. cholecystectomy is required if the right hepatic artery is to be ligated D. left lobe injury is not responsive to extensive packing E. none of the above

317 B. the indication for angiography to control hepatic hemorrhage is transfusion of 4 units pRBC in 6 hours or 6 units of pRBC in 24 hours. Schwartz 9th ed p 175

the higher rates of anastomotic failure observed during repair of duodenum is secondary to which of the following cause? A. Deficient closure of the submucosa B. Lack of collateral supply C. Absence of serosa D. Undue tension of sutures E. B and C

318 C. serosal healing is essential for quickly achieving watertight seal from the luminal side of the bowel. The importance of the serosa is underscored by the signifantly higher rates of anastomotic failure observed clinically in segments of bowel that are extraperitoneal. Schwarts 9th ed p 216

what is the correct sequence of the processes occuring in graft take? A. Inosculation-imbibition- revascularization B. Imbibition- revascularization- inosculation C. Imbibition-inosculation- revascularization D. None of the above

319 C. imbibition - thin film of plasma separates the graft from the wound bed. Inosculation- the process by which new blood vessels either directly invade the graft or anastomose to open dermal channels and restoring the pink hue of the skin. Schwartz 9th ed p1651

A 60-year-old man with newly diagnosed prostate cancer comes to see you. He underwent transrectal ultrasonography with a needle biopsy showing adenocarcinoma. Except for elevated PSA, he is currently asymptomatic. The fi rst test to order in staging for his prostate cancer is: A. Full-body plain film x-rays B. Bone scan C. CT of head D. CEA level E. Repeat PSA now

32 B. The correct answer is to do a bone scan. This is the first staging test done in the workup of prostate cancer. If abnormalities are found on the bone scan, you then proceed to do plain film x-rays of the areas to exclude other possible causes. Then you would perform surgical staging with removal and examination of the surrounding nodes (often done with prostatectomy).

which of the following is not true about esophageal perforation? A. most common location is left lateral just above the GEJ B. the most important prognostic indicator is the timing of primary closure (within 48 hours) C. non operative management includes hyperalimentation, and histamine blockers D. elevated serum amylase is a common finding E. none of the above

320 B. the most favorable outcome is obtained following primary closure of the perforation within 24 hours resulting in 80-90% survival. Schwartz 9th ed p 875

The strongest chemotactic factor for fibroblasts during the proliferation phase of wound healing: A. VEGF B. PDGF C. TNF D. IFN-y E. Il-1

321 B.

True in the management of caustic injuries to the esophagus, except: A. Alkali agents cause liquefactive necrosis. B. Asymptomatic patients can be observed. C. In patients with mild acid injury, neutralization of offending agent with NaHCO3 may be considered. D. Any degree of injury may result in strictures and squamous cell carcinoma. E. Alkali injuries have a worse prognosis.

322 C. NaHCO3 generates CO2, increasing the danger of perforation.

Regarding the blood supply to the stomach, this is also known as the coronary vein: A. left gastric vein B. right gastric vein C. portal vein D. right gastroepiploic vein E. left gastroepiploic vein

323 A.

the clinical triad of Zollinger-Ellison syndrome does not include: A. hypersecretion of HCl B. steatorrhea C. severe peptic ulcer disease D. gastrinoma

324 B. ZES clinical triad: hypersecretion of HCl, severe PUD, and gastrinoma

Damage to this nerve during mastectomies causes numbness in the upper, inner aspect of the arm: A. intercostobrachial nerve B. anterior thoracic nerve C. thoracodorsal nerve D. long thoracic nerve

325 A. The anterior thoracic nerve innervates the pectoralis muscles, and damage to this causes atrophy of these muscles. The thoracodorsal nerve innervates the latissimus dorsi, and damage results in a weak internal rotation and abduction of the arm. The serratus anterior is innervated by the long thoracic nerve. Damage to this causes the winged scapula.

A pancoast tumor is a superior sulcus tumor affecting the brachial plexus and sympathetic trunk causing: A. ptosis B. mydriasis C. anhydrosis D. A & C E. all of the above

326 D. Horner's syndrome in a patient with Pancoast tumor presents with ptosis, miosis, and anhydrosis.

The following radiographic features suggest malignancy in a solitary lung nodule, except: A. corona radiata sign B. size > 2 cm C. doubling time < 20 days D. lobulations E. none of the above

327 C. A doubling time of 30-450 days is one of the radiographic features that suggest malignancy. Benign features: smooth calcifications, < 2cm size, doubling time < 20 days or > 450 days.

True about septic shock: A. increased cardiac index B. low systemic vascular resistance C. increased venous capacitance D. all of the above E. none of the above

328 D.

How much IVF should be administered in a 63 kg man with third degree burns to both entire upper extremities, anterior torso in the first 8 hours post-injury? A. 3000 cc LR B. 3500 cc LR C. 4000 cc LR D. 4500 cc LR E. 5000 cc LR

329 D. 18% (both entire upper extremities) + 18% (anterior torso) = 36% x 4 x 63 kg = 4536 cc for the first 8 hours post-injury

Patients with symptoms from a Chiari I malformation may complain of: A. Seizures B. Extremity weakness C. Eye pain D. Ataxia E. Dysautonomia

33 B. SIMILAR TO PREVIOUS BOARD EXAM CONCEPT/PRINCIPLE. Chiari I malformation is the caudal displacement of the cerebellar tonsils below the foramen magnum. It may be seen as an incidental finding on MRI scans in asymptomatic patients. Symptomatic patients usually present with headache, neck pain, or symptoms of myelopathy, including numbness or weakness in the extremities. A syrinx may be associated, but the brain stem and lower cranial nerves are normal in Chiari I malformations. Chiari II malformations are more severe and involve caudal displacement of the lower brain stem and stretching of the lower cranial nerves. Symptomatic patients may be treated with suboccipital craniectomy to remove the posterior arch of the foramen magnum, along with removal of the posterior ring of C1. Removal of these bony structures relieves the compression of the cerebellar tonsils and cervicomedullary junction, and may allow reestablishment of normal CSF flow patterns.

Classic chronologic order of symptoms in acute appendicitis: A. vomiting - anorexia - abdominal pain B. anorexia - abdominal pain - vomiting C. anorexia - vomiting - abdominal pain D. abdominal pain - vomiting - anorexia E. abdominal pain - anorexia - vomiting

330 B.

Most common type of anorectal abscess: A. ischiorectal B. perianal C. intersphincteric D. pelvic

331 B.

The following are hard signs of peripheral arterial injury, except: A. pulsatile hemorrhage B. absent pulses C. acute ischemia D. associated nerve injury E. none of the above

332 D. Operation is mandatory if there are hard signs of peripheral arterial injury. Further evaluation is indicated in the presence of soft signs, which include proximity to vasculature, significant hematoma, associated nerve injury, A-A index of <0.9, and thrill or bruit. Schwartz p.158 table 7-8

The most common presenting complaint in a patient with anorectal abscess is: A. fever B. hematochezia C. anal pain D. draining pus from anus E. swelling at anal verge

333 C.

The most common cause of a defective sphincter is: A. lower esophageal sphincter with a mean resting pressure of < 6 mmHg B. overall sphincter length of < 2 cm C. intraabdominal sphincter length of < 1 cm D. none of the above

334 C. A, B and C define a permanently defective sphincter.The most common cause of a defective sphincter is inadequate abdominal length.

The accuracy of a preoperative diagnosis of an acute appendicitis ahould be? A. 70% B. 755 C. 80% D. 85% E. 90%

335 D.

True of appendicitis in a patient with AIDS/HIV: A. CD4 count is unrelated to rupture. B. HIV infected patients manifest absolute leukocytosis. C. Presentation in HIV patients is strikingly different from the general population. D. Post-op morbidity rates are similar E. Risk for rupture is increased in HIV patients.

336 E. Appendicitis in patients with HIV/AIDS: a low CD4 count is associated with a higher chance of rupture. These patients manifest relative leukocytosis. The presentation is similar to that of the general population, and post-operative morbidity rates are higher. Schwartz pp.1083- 1084

A clerk rotating on a radiology elective was presented an x-ray of the abdomen, which showed multiple airâ€"fluid levels with dilated loops of small bowel, paucity of air in the colon, and no air in the rectum. The radiology consultant asked the clerk which of the following clinical features would most likely be found in this patient: A. Hypoactive bowel sounds B. Pain out of proportion to physical examination C. Crampy abdominal pain that waxes and wanes D. Diarrhea E. A flat, rigid abdomen

34 C. The patient has a mechanical intestinal obstruction, as the description of the airâ€"fluid levels indicates a mechanical intestinal obstruction. Mechanical intestinal obstruction may be the result of extrinsic, intramural, or intraluminal causes. Symptoms include crampy abdominal pain that waxes and wanes, obstipation or constipation, nausea and vomiting, and abdominal distention. Physical examination of the abdomen reveals high- pitched bowel sounds and rushes and tinkles, as well as marked abdominal distention and tympany on percussion. Pain out of proportion to the physical examination is most suggestive of acute mesenteric ischemia.

True of fresh frozen plasma, except: A. It is the only source of factor VIII. B. It carries infection risks similar to those of other component therapies. C. It is a source of Vitamin K dependent factors. D. none of the above

340 A.

A 23 year old male came in with a 4 cm laceration on his right cheek after engaging in a brawl. Which of the following sutures is most appropriate for closing this patient’s laceration? A. Vicryl 3-0 B. Chromic 3-0 C. Silk 4-0 D. Nylon 6-0

341 D.

A 23 year old male came in with a 4 cm laceration on his right cheek after engaging in a brawl. He was sent home after suturing of his laceration was done. When will you advise him to come back for removal of his sutures? A. After 3 days B. After 1 week C. After 10 days D. After 2 weeks

342 A. Reference: Topnotch Surgery Pearls Handouts

A 17 year old football player sustained a fall on his outstretched right arm during practice, after which there was noted pain and swelling of the right clavicular area. X-ray confirmed a fracture of the middle third of the clavicle. At the ER, what would be the most appropriate initial management for this type of fracture? A. Analgesics only B. Ipsilateral sling to immobilize the shoulder C. Figure of eight splint D. Send to the OR for open reduction and internal fixation

343 B. Immobilization with a sling or a figure of eight splint are equally effective in the treatment of uncomplicated clavicle fractures, with a sling being preferable because it is more comfortable for the patient and may have less complications. Reference: http://www.hopkinsortho.org/claviclefx.html;

A 38 year old man consults because of a 4 cm nodule at the right lobe of the thyroid noted 3 years ago. There are no accompanying symptoms. He denies family history of cancer nor radiation exposure. On PE, the nodule is hard, moves with deglutition, without any palpable cervical lymph nodes. The rest of the examination is unremarkable. The initial diagnostic test to perform in evaluating this nodule is: A. FNAB B. Radioactive iodine scan C. CT scan of the neck D. Ultrasound of the neck

344 A. Review algorithm on work-up of solitary thyroid nodule Reference: Schwartz, Principles of Surgery, 9th ed. p. 1359

A 30 year old woman underwent right lobectomy and isthmusectomy for a 1 cm papillary cancer. She will require: A. Observation only B. Completion thyroidectomy C. RAI ablation therapy D. B and C

345 A. For minimal papillary thyroid cancers (1 cm or less), unilateral lobectomy and isthmusectomy is adequate. Reference: Schwartz. Principles of Surgery, 9th ed. p. 1363

An 85 year old female patient with atrial fibrillation developed sudden onset of severe abdominal pain. On PE, abdominal findings are minimal. Vital signs show tachycardia with normal BP and temperature. The most likely diagnosis is: A. Non-occlusive mesenteric ischemia B. Acute mesenteric ischemia due to embolism C. Mesenteric vascular thrombosis D. Mesenteric vasculitis

346 B. Severe abdominal pain out of proportion to the PE findings is characteristic of acute mesenteric ischemia. Because of this patient’s history of atrial fibrillation, the most likely cause of the mesenteric ischemia is an embolus.

A 75 year old male presents with bulging of the midline abdomen especially on straining. On PE, there is a separation of the midline. Which of the following statements is true about this condition? A. It is due to a separation of the external and internal oblique aponeurosis B. It is sometimes mistaken for a Spigelian hernia C. It is always congenital in aetiology D.None of the above

347 D. Diagnosis: Diastasis recti. This is a separation of the pillars of rectus abdominis muscles, and may either be congenital or acquired in aetiology. A Spigelian hernia is not located at the midline and is not usually mistaken for diastasis recti.

A 52 year old female underwent breast conservation surgery with adjuvant radiotherapy for a stage 1 breast cancer. What is her expected 5 year survival rate? A. 100% B. 95% C. 80% D. 70%

348 A. Based on data from the American College of Surgeons National Cancer Database, the 5 year survival rate for breast cancer by stage are as follows: Stage I - 100% Stage IIA - 92% Stage IIB - 81% Stage IIIA - 67% Stage IIIB - 54% Reference: Schwartz, Principles of Surgery, 9th ed. p. 457

A 54 year old female with a history of rheumatic heart disease and valve replacement surgery suddenly developed severe abdominal pain with some tenderness and a palpable mass around 3 cm in size. There is also a bluish discolouration around the umbilicus. The most likely diagnosis is: A. Desmoid tumor B. Rectus sheath hematoma C. Acute appendicitis D. Acute cholecystitis

349 B..Rectus sheath hematoma may occur spontaneously in patients receiving anticoagulant therapy, which is probably the case in a patient who has undergone valve replacement surgery.

A 51-year-old man with recurrent peptic ulcer disease had a fasting gastrin level of 1000. Which of the following organs is the most common site of origin of the tumor associated with this syndrome? A. Stomach B. Duodenum C. Lymph nodes D. Spleen E. Pancreas

35 E. Zollingerâ€"Ellison syndrome is caused by a nonâ€" islet cell tumor that produces gastrin and is associated with gastric acid hypersecretion and peptic ulcer disease. Tumors are biologically malignant in 60% of cases, and the most common site involved is the pancreas. Most gastrinomas are found in the gastrinoma triangle (Passaro's triangle): this is bound by the junction of cystic and common bile ducts, junction of the second and third parts of the duodenum, and the junction of the neck and body of the pancreas.

A 45 year old female was brought to the ER because of colicky abdominal pain, vomiting, and abdominal distention. Condition started 3 days PTA as occasional cramps epigastric pain accompanied by vomiting, abdominal distention, and obstipation. Past medical history revealed appendectomy 1 year ago. On PE, there was high grade fever and (+) rebound tenderness. The patient is most likely suffering from: A. Mechanical intestinal obstruction due to post- op adhesions B. Intestinal distention from prolonged ileus C. Mechanical obstruction due to benign intestinal tumor D. Mechanical obstruction due to malignant neoplasm

350 A. Intra-abdominal adhesions related to prior abdominal surgery account for up to 75% of cases of small bowel obstruction. Reference: Schwartz, Principles of Surgery, 9th ed. p. 988

On rectal exam, a 42 year old female is found to have a hard, nodular mass 4 cm from the anal verge. Initial diagnostic work up should include one of the following: A. Bone scan B. Scout film of the abdomen C. Proctosigmoidoscopy D. CT scan of the abdomen

351 C.

A rare, premalignant condition in which the gallbladder wall becomes calcified: A. Porcelain gallbladder B. Adenomyomatosis of the gallbladder C. Hydrops of the gallbladder D. Empyema of the gallbladder

352 A.

A 60 year old male consulted because of dysphagia of 5 months duration, initially to solid foods. At present, he is having problems swallowing even his own saliva. Other accompanying symptoms include anorexia, weight loss of 50 lbs in 4 months. Which of the following is the most likely initial diagnosis? A. Achalasia B. Esophageal CA C. Severe esophagitis from reflux D. Zenker’s diverticulum

353 B.

The most important prognostic factor for recurrent disease and survival in breast cancer is: A. Age B. Axillary nodal status C. ER and PR status D. Histologic grade

354 B.

A 28 year old bus driver with his seatbelt on, figured in a vehicular crash vs another bus. At the ER, 30 minutes post-injury, his VS were noted as follows: BP=130/70, PR=100/min, RR=20/min, GCS=15. Abdomen is slightly globular, with a contusion hematoma and direct tenderness at the RUQ, no rebound tenderness. What is the most appropriate initial diagnostic procedure which will help you decide on the proper management? A. FAST B. Triple contrast CT scan C. DPL D. Laparoscopy

355 A. In this case of blunt abdominal trauma, patient is hemodynamically stable, and abdominal findings are equivocal (overt peritonitis is not present), therefore, FAST is warranted. Review algorithm for evaluation of suspected abdominal trauma. Reference: Topnotch Surgery Pearls Handouts Schwartz, Principles of Surgery, 9th ed. p. 155

A 40 year old female presented with a 5 cm solitary, firm thyroid nodule as well as a scalp mass on the parieto-occipital area which showed lytic changes on X-ray. Biopsy of the skull lesion revealed colloid material, same as that seen on FNAB of the thyroid nodule. What is the most likely diagnosis? A. Anaplastic thyroid cancer B. Papillary thyroid cancer C. Follicular thyroid cancer D. Medullary thyroid cancer

356 C. Follicular thyroid cancer is more likely to metastasize than the other differentiated thyroid cancers.

The appropriate management for lobular carcinoma in situ includes which of the following: A. Close follow-up B. Radiation after excision C. Biopsy of the opposite breast D. Mastectomy and regional node dissection

357 A. Reference: Topnotch Surgery Handouts Schwartz, Principles of Surgery, 9th ed. p. 454

A 28 year old male presents with a 3 year history of left inguino-scrotal mass. On examination, the mass measures 5x4 cm, slightly tender, and is reducible. The next step would be: A. Start IV antibiotics B. Request for ultrasound of the mass C. Advise elective surgery D. Admit the patient and perform emergency surgery

358 C.

A 50 year old female consulted because of yellowish discolouration of the skin and sclera. She was diagnosed to have multiple gallstones 2 years ago based on ultrasound results. On PE, patient is icteric, afebrile, with RUQ tenderness. The most likely cause of this patient’s jaundice is: A. Hepatitis B. Liver abscess C. Acute cholecystitis D. Common bile duct stones

359 D.

Paraphimosis refers to: A. Inability to retract the foreskin B. Inability to reduce the foreskin after it has been retracted C. Infection of the foreskin near the urethral meatus D. Excessive length of foreskin E. Incomplete embryologic formation of the foreskin

36 B. Paraphimosis is a common problem that represents a true medical emergency. When foreskin is retracted for prolonged periods, constriction of the glans penis may ensue. This is particularly likely in hospitalized patients who are confined to bed or who have altered mentation. Edema often forms in the genitals of supine patients due to the dependent position of that area. Patients with diminished consciousness will not be aware of the penile pain from paraphimosis, which may delay recognition of the problem until too late. Delay can be catastrophic as penile necrosis may occur due to ischemia.

A 43 year old female who practices reverse smoking presents with a 2.5x2 cm ulcer with heaped up borders on the right floor of the mouth. PE showed a 1x2 cm submandibular mass on the right together with several enlarged discrete masses on the right upper jugular area. The most practical way of establishing the diagnosis is to do: A. Punch biopsy B. Fine needle aspiration biopsy C. Excision biopsy D. Comprehensive history and physical examination

360 A.

A 58M with a 3 week history of jaundice is brought to the ER because of lethargy. Upon seeing the patient, he is febrile (39.5C) and hypotensive (80/45) with RUQ pain. The initial management for this patient would be? A. Broad spectrum antimicrobial therapy B. Fluid resuscitation C. Relief of elevated biliary pressure D. Surgery to remove the gallbladder E. A and B

361 E. The initial treatment of patients with cholangitis includes IV antibiotics and fluid resuscitation. These patients may require intensive care unit monitoring and vasopressor sup- port. Most patients will respond to these measures. However, the obstructed bile duct must be drained as soon as the patient has been stabilized. Schwartz 10th pg 1323

A 48/M diabetic sought consult at the OPD for recurrent abdominal pain occuring during the night and sometimes after eating a heavy meal. The pain typically lasts about 3 hours on average and resolves on its own. In between episodes, the patient is well. How will you manage this patient? A. Prompt cholecystectomy B. Cystectomy during an episode of abdominal pain C. Observation D. Diet modification E. C and D

362 A. iabetic patients with symptomatic gallstones should have a cholecystec- tomy promptly, as they are more prone to develop acute cholecystitis that is often severe. Schwartz 10th pg 1320

The Child-Turcott-Pugh scoring is for predicting surgical risk of intra-abdominal operations on cirrhotic patients. A 56/M was diagnosed with chronic liver disease from chronic hep B infection. On PE, he was oriented to three spheres, conversant and not in distress and negative for shifting dullness Labs done showed bilirubin 3 mg/dL, albumin 1.5 g/dL, INR 1.6 and abdominal UTZ showed minimal intraabdominal fluid. What is the patient's CTP score? A. Class A B. Class B C. Class C D. Class D E. None of the above

363 B. Each variable score 1, 2 or 3. Bilirubin <2 mg/dL, 2â€"3 mg/dL, >3 mg/dL; albumin >3.5 g/dL, 2.8â€"3.5 g/dL, <2.8 g/dL, PT INR <1.7, 1.7â€" 2.2, >2.2; encephalopathy none, controlled, uncontrolled; ascites none, controlled, uncontrolled Child A 5-6 Child B 7-9 Child C 10-15 Schwartz 10th pg 1280

Which of the following is/are TRUE regarding the surgical metabolism of carbohydrates? A. The presence of fructose, galactose and mannitol in the circulation evokes an insulin response B. Infusion of 50g per day of glucose facilitates fat entry into the TCA cycle and reduces ketosis C. Insulin has been shown to reverse protein catabolism during severe stress by stimulating protein synthesis in skeletal muscles and by inhibiting hepato- cyte protein degradation. D. B and C E. All of the above

364 D. Neither fructose or galactose within the circulation nor exogenous mannitol (for neurologic injury) evokes an insu- lin response. The exogenous administration of small amounts of glucose (approximately 50 g/d) facilitates fat entry into the TCA cycle and reduces ketosis. The administration of insulin, however, has been shown to reverse protein catabolism during severe stress by stimulating protein synthesis in skeletal muscles and by inhibiting hepato- cyte protein degradation. Schwartz 10th pg 48

Which of the following is associated with the highest expected infection rates? A. Hernia repair B. Cholecystectomy C. Colorectal surgery D. Penetrating abdominal trauma E. Necrotizing soft tissue infections

365 C. Class I 1-2%; Class II - 2.1-9.5%; Class II colorectal - 4-14%; Class III - 3.4-13.2%; Class IV - 3.1-12.8% Schwartz 10th pg 148

The following are current indications for ED thoracotomy EXCEPT? A. Patients sustaining witnessed penetrating trauma to the torso with <15 min of prehospital CPR B. Patients sustaining witnessed blunt trauma with <10 min of prehospital CPR C. Persistent severe postinjury hypotension (SBP ≤60 mm Hg) due to cardiac tamponade D. Air embolism E. No exception

366 E. Indications Salvageable postinjury cardiac arrest: Patients sustaining witnessed penetrating trauma to the torso with <15 min of prehospital CPR Patients sustaining witnessed blunt trauma with <10 min of prehospital CPR Patients sustaining witnessed penetrating trauma to the neck or extremities with <5 min of prehospital CPR Persistent severe postinjury hypotension (SBP ≤60 mm Hg) due to: Cardiac tamponade Hemorrhageâ€"intrathoracic, intra-abdominal, extremity,cervical Air embolism Contraindications Penetrating trauma: CPR >15 min and no signs of life (pupillary response, respiratory effort, motor activity) Blunt trauma: CPR >10 min and no signs of life or asystole without associated tamponade Schwartz 10th pg 167

A 34/M was stabbed in the neck by an unknown assailant 15 minutes ago. He was immediately rushed to the ER and upon seeing the patient BP was 100/60, HR 98, RR 29, T 37 not in apparent distress. The wound sustained was at the level of the cricoid. He complains of hoarseness and dysphagia. What is the initial step in the management of the patient? A. Immediately do operative exploration B. CT angiography C. Observe D. Local wound exploration E. Close the wound with nylon 3-0

367 B. Hemodynamically Unstable and/or with Hard signs: massive hemoptysis, rapidly expanding hematoma: Operative exploration regardless of zone Hemodynamically Stable and with soft signs: dysphagia, venous bleeding, subcutaneous emphysema, hematoma, hoarseness, stridor, odynophagia: Zone 1-2 do CTA neck/chest if positive operative exploration, Zone 3 if CTA positive do angioembolization; Asymptomatic: Zone 1 and Zone II transcervical GSW do CTA if positive operative exploration, Zone II injuries (Not transcervical GSW) and Zone III observe Schwartz 10th pg 176

Which of the following is/are TRUE regarding management of burn injuries? A. Patients with acute burn injuries should be given prophylactic antibiotics to prevent overwhelming sepsis B. Resuscitation involves giving 3 to 4 mL/kg/% TBSA of lactated Ringer’s, half given during the first 8 hours after burn and the remaining half given over the subsequent 16 hours C. Urine output should be 30 mL/h in adults and 1 to 1.5 mL/kg/h in pediatric patients D. B and C E. All of the above

368 D. Patients with acute burn injuries should never receive pro- phylactic antibiotics. This intervention has been clearly demon- strated to promote development of fungal infections and resistant organism and was abandoned in the mid-1980s. Schwartz 10th pg 228 and 230

MRM involves removal of the following: A. Level I, II and III lymph nodes B. Long thoracic, thoracodorsal and lateral pectoral nerves C. Pectoralis major and minor D. A and C E. None of the above

369 E. MRM removes the breast plus nipple areola complex, Level I and II LNs. Removal of choice A to C plus the medial pectoral nerve is done in radical mastectomy.Topnotch

Which of the following is NOT part of Cantrell’s pentalogy? A. Omphalocele B. Ectopia cordis C. Posterolateral diaphragmatic hernia D. Cardiac anomalies E. cleft sternum

37 C. Omphalocele has an incidence of approximately 1 in 5000 live births and occurs in association with special syndromes such as exstrophy of the cloaca (vesicointestinal fissure), the Beckwith- Wiedemann constellation of anomalies (macroglossia, macrosomia, hypoglycemia, visceromegaly, and omphalocele) and the Cantrell pentalogy (lower thoracic wall malformations such as cleft sternum, ectopia cordis, epigastric omphalocele, anterior midline diaphragmatic hernia, and cardiac anomalies)

Weight of a normal thyroid gland: A. 10g B. 20g C. 30g D. 40g E. 50g

370 B. Schwartz 10th pg 1523

You managed to suture a lacerated wound in the ED on the cheek of a patient after an apparent brawl. When would you ask the patient to come back for suture removal? A. 1-2 days B. 3-5 days C. 5-7 days D. 7-12 days E. 10-14 days

405 B. Sutures done on the face can be removed after 3 - 5 days.

The only drug/s shown to reduce short and long- term perioperative cardiac morbidity and mortality in non-cardiac surgery: A. Nitrates B. Anti-platelet agents C. Beta-blockers D. Calcium channel blockers E. ACE inhibitors

371 C. Adverse cardiac events occur in 2% to 5% of patients undergoing noncardiac surgery and as many as 34% of patients undergoing vascular surgery. Certain perioperative cardiac events, such as myocardial infarction, are associated with a mortality rate of 40% to 70% per event, prolonged hospitaliza- tion, and higher costs. Appropriately administered β-blockers reduce perioperative ischemia, especially in at-risk patients. It has been found that nearly half of the fatal cardiac events could be preventable with β- blocker therapy. Schwartz 10th pg 374

Which of the following is/are TRUE regarding graft take? A. Imbibition refers to the first 24 to 48 hours after skin grafting, wherein a thin film of fibrin and plasma separates the graft from the underlying wound bed B. Angiogenesis occurs after 48 hours and during this time, fine vascular network begins to form within the fibrin layer C. Graft failure can be caused by infection, shear, hematoma or seroma formation D. A and C. E. All of the Above

372 D. Choice B refers to inosculation Schwartz 10th pg 1833

A 34/M smoker complains of recurrent headache and unilateral ear discharge with associated hearing loss. On PE, you noted matted lymph nodes in the posterior neck. What is the standard treatment for this disorder? A. Chemoradiation B. Radical resection of the tumor C. Chemotherapy D. Radiotherapy E. None of the above

373 A. Symptoms associated with nasopharyngeal tumors include nasal obstruction, posterior (level V) neck mass, epistaxis, head- ache, serous otitis media with hearing loss, and otalgia. Cranial nerve involvement is indicative of skull base extension and advanced disease. The standard treatment for nasopharyngeal carcinoma is chemo- radiation. Combination therapy produces superior survival rates for nasopharyngeal carcinoma in comparison to radiation alone. Schwartz 10th pg 594

Main suspected organism for contact lens users with microbial keratitis? A. Fungi B. Pseudomonas aeruginosa C. Staphylococcus aureus D. Herpes simplex E. Adenovirus

374 B.

Elective repair of descending thoracic aneurysms in asymptomatic patients should be done when? A. diameter > 4.5 cm B. diameter > 5.5 cm C. diameter > 6 cm D. rate of dilatation < 0.5 cm per year E. B and C

375 C. Recommend elective operation in asymptomatic patients when the diameter of an ascending aortic aneurysm is >5.5 cm, when the diameter of a descending thoracic aortic aneurysm is >6.0 cm, or when the rate of dilata- tion is >0.5 cm/y. For patients undergoing aortic valve replacement or repair, smaller ascending aortic aneurysms (>4.5 cm) are considered for concomitant repair.

Treatment modalities available for the treatment of hemorrhoids include the following EXCEPT? A. Rubber band ligation B. Infrared photocoagulation C. Sclerotherapy D. Operative hemorrhoidectomy E. No exception

376 E. Schwartz 10th pg 1223

TRUE of Passaro's triangle EXCEPT: A. Up to 90% of ZES tumors are found within the triangle B. The medial point of the triangle is found at the confluence of the 2nd and 3rd part of the duodenum C. Tumors located outside the triangle have the worst prognosis D. A and B E. All are true

377 B. B- the inferolateral point Schwartz 10th pg 1392

The Amsterdam criteria for clinical diagnosis of HNPCC EXCEPT: A. Three affected relatives with histologically verified adenocarcinoma of the large bowel B. Two successive generations of a family C. At least one patient diagnosed before age 50 D. Presence of other HNPCC-related carcinomas such as FAP E. No exception

378 D. The Amsterdam criteria for clinical diagnosis of HNPCC are three affected relatives with histologically verified adenocarcinoma of the large bowel (one must be a first-degree relative of one of the others) in two successive generations of a family with one patient diagnosed before age 50 years. FAP is not an HNPCC related CA Schwartz 10th pg 1207

Most common location of aneurysms? A. Anterior communicating artery B. Posterior communicating artery C. Middle cerebral artery bifurcation D. Basilar artery tip E. Vertebral artery

379 A. Choices arranged from most common to least common with the following frequencies: 30%, 25%, 20, 10%, 5% Schwartz 10th pg 1730

A 30-year-old man is evaluated for a thyroid nodule. T he patient reports that his father died from thyroid cancer and that a brother had a history of recurrent renal stones. Blood calcitonin concentration is 2000 pg/mL (normal is < 100); serum calcium and phosphate levels are normal. T he patient is referred to a thyroid surgeon. Which of the following studies should also be obtained? A. Obtain a liver scan. B. Measure parathormone level. C. Measure urinary catecholamines. D. Administer suppressive doses of thyroxine and measure levels of thyroid-stimulating hormone. E. Treat the patient with radioactive iodine.

38 C. For the patient described, the markedly increased calcitonin level indicates the diagnosis of medullary carcinoma of the thyroid. In view of the family history, the patient most likely has multiple endocrine neoplasia (MEN) type 2A, which includes medullary carcinoma of the thyroid gland, pheochromocytoma, and parathyroid hyperplasia. Pheochromocytoma may exist without sustained hypertension, as indicated by excessive urinary catecholamines. Before thyroid surgery is performed on this patient, a pheochromocytoma must be ruled out through urinary catecholamine determinations; the presence of such a tumor might expose him to a hypertensive crisis during surgery. T he serum calcium serves as a screening test for hyperparathyroidism. At surgery, the entire thyroid gland must be removed because foci of parafollicular cell hyperplasia, a premalignant lesion, may be scattered throughout the gland. Successful removal of the medullary carcinoma can be monitored with serum calcitonin levels. Medullary carcinoma of the thyroid rarely metastases to the liver, so a liver scan would be unnecessary if liver enzymes are normal. Thyroxine will be needed after surgery, but MEN type 2 is not associated with hypothyroidism. Radioactive iodine can be used to treat malignancies that arise from the follicular cells of the thyroid; parafollicular cells, however, do not take up iodine and do not respond to radioactive iodine. Hyperparathyroidism, while unlikely in this eucalcemic patient, is probably present in his brother.

A 14/M presents with severe local pain and tenderness in the area of the tibial tubercle. The patient was previously healthy and engages in multiple activities such as football, running and cycling. How will you manage the patient? A. Activity restriction B. NSAIDs C. Surgical correction D. Steroids E. None of the above

380 A. Osgood-Schlatter disease is a common problem most often seen in athletically active adolescents. This disorder is characterized by ossification in the distal patellar tendon at the point of its tibial insertion. This disorder is thought to result from mechanical stress on the tendinous insertion. Radiographs show calcified ossicles within the tendon at its insertion. The disease presents with severe local pain and tenderness in the area of the tibial tubercle. Treatment for the disease is activity restriction. If the symptoms are improved, athletic participation can be resumed. Schwartz 10th pg. 1783

Full-thickness rupture of the esophageal wall as a result of forceful vomiting, an entity known as Boerhaave syndrome, most commonly occurs in which part of the esophagus? A. Right posterolateral wall of the distal esophagus B. Right posterolateral wall of the mid esophagus C. Left anterolateral wall of the distal esophagus D. Left anterolateral wall of the mid esophagus E. Left posterolateral wall of the distal esophagus

381 E. SIMILAR TO PREVIOUS BOARD EXAM CONCEPT/PRINCIPLE.

A 60-year old male presents with left lower quadrant abdominal pain with direct tenderness, fever and leukocytosis. Which of the following diagnostic modalities will be your test of choice to confirm your diagnosis and to evaluate this patient? A. Barium enema B. Holoabdominal ultrasound C. Plain radiograph of the abdomen D. Contrast-enhanced CT scan E. Colonoscopy

382 D. SIMILAR TO PREVIOUS BOARD EXAM CONCEPT/PRINCIPLE.

Which of the following structures are spared during Whipple's procedure? A. Gallbladder B. Duodenum C. Common bile duct D. Distal stomach E. Proximal jejunum

383 D. SIMILAR TO PREVIOUS BOARD EXAM CONCEPT/PRINCIPLE. The following structures are removed during a standard Whipple's procedure: pancreatic head, duodenum, proximal jejunum, gallbladder and CBD.

The critical diameter of an abdominal aortic aneurysm where there is significant risk of rupture is: A. 4.5 cm B. 5 cm C. 5.5 cm D. 6 cm E. 7 cm

384 C. SIMILAR TO PREVIOUS BOARD EXAM CONCEPT/PRINCIPLE

A 40-year old female presented with a 3-cm palpable, non-movable, painless mass on the upper outer quadrant of her right breast. No axillary nodes are appreciated. Ultrasound showed that the mass is hypoechoic with indistinct margins. What is the next best step in your management? A. Order diagnostic mammography. B. Perform core-needle biopsy. C. Excise the mass and send to the laboratory for biopsy. D. Do fine-needle biopsy. E. Admit the patient and perform breast conservation surgery.

385 A. SIMILAR TO PREVIOUS BOARD EXAM CONCEPT/PRINCIPLE. You should first request diagnostic mammography to look for other mass lesions in the breasts.

A patient with Crohn disease has been sustaining fluid losses due to the presence of colovesical fistula. Which of the following IV replacement fluids would be the best choice to give to this patient? A. plain NSS B. Ringer's lactate C. D5LR D. Colloid solution E. Any of the above

386 A. SIMILAR TO PREVIOUS BOARD EXAM CONCEPT/PRINCIPLE. For fluid losses from the stomach and colon, give NSS. Add KCl if with hypokalemia. For fluid losses from the pancreas and small bowel, give LR.

How many calories would you obtain from 3L of D5LR? A. 200 kcal B. 300 kcal C. 400 kcal D. 500 kcal E. 600 kcal

387 E. SIMILAR TO PREVIOUS BOARD EXAM CONCEPT/PRINCIPLE. D5LR = 200 kcal/L. D5W = 170 kcal/L.

Which among the following microorganisms is most commonly implicated in fatal burn wound infections? A. Staphylococcus aureus B. Pseudomonas aeruginosa C. Klebsiella granulomatis D. Acinetobacter baumanii E. Streptococcus pyogenes

388 B.

Ultrasound is the most common diagnotic modality requested for initial evaluation of problems affecting the biliary tree. The common bile duct is said to be dilated if its width is more than how many millimeters? A. >4 mm B. >6 mm C. >8 mm D. >10 mm E. >12 mm

389 B.

A 61-year-old man with a history of gastroesophageal reflux disease and Barrett’s esophagus is undergoing a screening endoscopy. The biopsy results show high-grade dysplasia. What is the recommended treatment approach? A Refer the patient to a surgeon for esophagectomy. B Repeat surveillance endoscopy in 6 months. C Repeat surveillance endoscopy in 1 year. D No further endoscopy is indicated unless the patient becomes symptomatic. E Initiate chemotherapy.

39 A. Barrett’s esophagus with high-grade dysplasia requires defi nitive treatment, and referral for esophagectomy is recommended. For patients who are medically unstable or for those who refuse surgery, endoscopic resection or photodynamic therapy are viable alternatives. Surveillance alone is inappropriate, making B and C incorrect. (Surveillance endoscopy at 6 months and 1 year are recommended for patients with Barrett’s esophagus with low- grade dysplasia and no dysplasia, respectively.) Chemotherapy is not indicated unless carcinoma in found.

Which of the following CT scan findings would strongly suggest a diagnosis of hepatocellular carcinoma? A. Strong arterial enhancement and early washout on portal venous phase of a 5-cm hepatic mass B. Multiple non-enhancing hepatic masses ranging from 0.5 to 2.0 cm in size C. Enhancement on both arterial and portal venous phases with delayed washout of a 4-cm hepatic mass D. Initial peripheral enhancement with gradual centripetal fill-in of a 6.0-cm hepatic mass E. 3-cm lobulated hepatic mass in the left lobe with scattered areas of fat attenuation

390 A.

Which of the following ultrasound findings is most specific for chronic calculous cholecystitis? A. Thickened gallbladder wall >3mm B. Contracted gallbladder, persistently <2cm despite adequate fasting C. Presence of pericholecystic fluid D. Gallbladder hydrops, >5 cm E. Visualization of highly echogenic foci with posterior shadowing within the gallbladder lumen.

391 B. SIMILAR TO PREVIOUS BOARD EXAM CONCEPT/PRINCIPLE. A thickened gallbladder wall is a non-specific finding of cholecystitis; it may occur in both acute and chronic cases. The presence of stones (gravity-dependent echogenic foci within the lumen) indicates cholelithiasis, but not necessarily cholecystitis.

A 36-year old male is to undergo elective surgery for complicated duodenal ulcer disease. Which of the following antibiotics will you give as prophylaxis? A. Cefazolin B. Metronidazole C. Ampicillin-sulbactam D. Cefoxitin E. Penicillin G

392 D. Colorectal and small bowel surgery: cefazolin + metronidazole. Gastroduodenal surgery: cefoxitin or cefotetan. Biliary tract with active infection: ampicillin-sulbactam.

An uncomplicated appendectomy is classified as what type of surgical wound? A. Class I: clean B. Class ID: clean with prosthetic device C. Class II: clean/contaminated D. Class III: contaminated E. Class IV: dirty

393 C. Class I: hernia repair, breast biopsy. Class II: appendectomy, elective GI surgery (except those involving the colon), cholecystectomy. Class III: penetrating trauma, enterotomy. Class IV: perforated diverticulitis, necrotizing infections.

Which of the following statements describes keloid formation but not hypertrophic scars? A. Dense accumulation of fibrous tissue B. Lesion extending beyond the border of the original wound C. Thick, raised scars over a previous wound D. Treatment with intralesional steroid injection has been effective E. Due to failure of collagen breakdown

394 B. The rest of the statements apply to both keloid and hypertrophic scars.

A 25-year old female presented to the emergency department due to a 5-cm laceration on her right cheek sustained from a broken shard of glass. Which of the following sutures would you use for this patient? A. Silk 6-0 B. Chromic 6-0 C. Nylon 6-0 D. Cotton 4-0 E. Vicryl 4-0

395 C. SIMILAR TO PREVIOUS BOARD EXAM CONCEPT/PRINCIPLE. Use a 6-0 suture for the face. Nylon would be the best choice since it is synthetic and would least likely cause allergy and/or inflammatory reaction, thus promoting better aesthetic outcome. Chromic and silk are biological sutures.

In relation to the patient described in the preceding question, when will you advice her to return for suture removal? A. 3-5 days B. 5-7 days C. 7-12 days D. 10-14 days E. 14-28 days

396 A. Face: 3-5 days. Neck: 5-7 days. Scalp: 7-12 days. Extremities: 10-14 days.

The most important reason for requesting a KUB- IVP in a patient with a stab wound in the left flank and fross hematuria is to assess which of the following? A. Degree of renal injury B. Location for best surgical approach C. Site of possible perforation D. Functional status of the right kidney E. Viability of the left kidney

397 D.

Which of the following describes the most common type of Salter-Harris fracture? A. Fracture across the growth plate without epiphyseal and metaphyseal involvement B. Fracture across the growth plate with metaphyseal involvement but not epiphyseal involvement C. Fracture across the growth plate with epiphyseal involvement but not metaphyseal involvement D. Fracture across the growth plate with metaphyseal and epiphyseal involvement E. Compression fracture on the growth plate

398 B. A: type I. B: type II. C: type III. D: type IV. E: type V. Type II is the most common. Type III, IV and V have poorer prognosis.

Which of the following is the only extraocular muscle that does not have any fascial attachment to any other extraocular muscles? As such, during surgery, when it is accidentally disinserted, it becomes difficult to capture and reinsert it in its proper position. A. Superior rectus B. Inferior rectus C. Lateral rectus D. Medial rectus E. Inferior oblique

399 D.

In which type of hiatal hernia does the fundus of the stomach herniate? A. Type I B. Type II C. Type III D. Type IV E. Type V

4 B. Type I (sliding) â€" cardia Type II (rolling) â€" fundus Type III â€" cardia and fundus Type IV â€" intestines SIMILAR TO PREVIOUS BOARD EXAM CONCEPT/PRINCIPLE Source: Schwartz’s Principles of Surgery 8th ed p 842

A 6-year-old presents with a tibial fracture of the metaphysis extending across the growth plate. This would be a A. Salter-Harris type 1 fracture B. Salter-Harris type 2 fracture C. Salter-Harris type 3 fracture D. Salter-Harris type 4 fracture E. Salter-Harris type 5 fracture

40 B. Classification of growth plate injuries has important implications as doctors communicate about the treatment of a patient. The exact type of physeal injury is important for the prognosis and treatment of the fracture. Salter and Harris described a very useful classification of growth plate injuries. A type I injury is a simple transverse failure of the physis without involvement of the ossified epiphysis or metaphysis. A Salter-Harris type II fracture contains a component of fracture through the growth plate in continuity with a fracture of the metaphysis. Salter-Harris type III fracture occurs partially through the epiphysis and partially through the growth plate. These fractures are essentially always intra-articular. A Salter-Harris type IV injury is one which has a fracture line extending through the physis extending from the metaphysis through into the epiphysis. Finally, a Salter-Harris type V injury is a subtle injury where the physis itself is injured but not displaced.

What is the most common intracranial complication of otitis media? A. Temporal lobe abscess B. Meningitis C. Epidural abscess D. Subdural effusion E. Encephalitis

400 B. SIMILAR TO PREVIOUS BOARD EXAM CONCEPT/PRINCIPLE.

An intern without your supervision administered 10 mL of 2% Lidocaine into a patient for wound suture. What dose was administered? A. 20 mg B. 40 mg C. 100 mg D. 200 mg E. None of the above

401 D. SIMILAR TO PREVIOUS BOARD EXAM CONCEPT/PRINCIPLE. A 2% Lidocaine preparation is 20mg/mL (10 x 20 = 200mg)

A malnourished Colon Adenocarcinoma patient was transfused 2 liters of D5LR, how may kilocalories was provided to the patient? A. 240 kcal B. 340 kcal C. 440 kcal D. 540 kcal E. 640 kcal

402 B. a liter of D5LR provides 170 kcal. (SIMILAR TO PREVIOUS BOARD EXAM CONCEPT/PRINCIPLE)

A cytokine secreted by the Macrophages which was found to be the major inducer of muscle catabolism and cachexia during stress. A. IL1 B. IL2 C. IL3 D. TNF-alpha E. TNF-beta

403 D.

A patient with Familial Adenomatous Polyposis underwent Total Proctocolectomy, which of the following Electrolytes will most likely be deficient in this patient and thus must be replaced? A. Sodium B. Potassium C. Chloride D. Bicarbonate E. Calcium

404 B. Colonic secretions contain the highest Potassium amounting to 35 mEq/L.

After an apparent heated argument with a "Siga", a patient sustained a penetrating Gunshot wound in the neck above the cricoid cartilage and the angle of the mandible, on the right. Patient is hemodynamically stable with noted hoarseness of the voice. Which of the following would be the appropriate management? A. Observe only, the patient is hemodynamically stable B. Do a CT-Scan C. Explore Immediately D. Do Angiography E. None of the above

406 C. Although the patient is hemodynamically stable, patient presents with hoarseness which is a sign that a neck structure was severed, Neck exploration should be done immediately.

A 50 kg burn patient sustained 2nd and 3rd degree burns on the anterior right thigh, leg and foot, the perineum and the whole anterior abdomen, How will the IV fluid be initiated? A. 3.2 L PLR fast drip B. 2.35 L PLR fast drip C. 2.35 L PLR for 8 hours D. 1.9 PLR fast drip E. 1.9 PLR for 8 hours

407 E. 19% TBSA (4x50x19=3800) 3,800 mL /2=1,900mL to run for 8 hours, the other half to run for 6 hours.

A trauma patient with gross hemorrhage from a large hacking wound in the thigh with the following Vital signs: CR:125 bpm, RR: 22 cpm, BP: 110/80, may have an approximate blood loss of around: A. less than 500 mL B. 500 - 750 mL C. 750 - 1,500 mL D. 1,500 - 2,000 mL E. More than 2,000 mL

408 C. patien is probably in stage II hypovolemic shock/ compensated shock.

While watching TV, a family requested help from "Sagip Kapamilya" for their child to undergo a Kasai Procedure, you knew that the proper timing for a Kasai Procedure is: A. Immediately at birth B. Prior to 2 months C. At 1 year old D. before development of speech E. when the child reached 10 kg

409 B. Kasai Procedure oe Early hepaticoportoenterostomy is a surgical procedure for Biliary Atresia which is properly timed prior to 2 months old which is done to lessen the chance for progression to a liver transplant.

Absolute contraindications to Breast Conservative Surgery A. Prior RT B. Preganancy C. Positive margins D. Multicentric E. AOTA

41 E. Absolute contraindications to BCS include A. Prior RT B. Preganancy C. Positive margins D. Multicentric E. Diffuse microcalcifications

A burn patient will undergo reconstructive surgery due to a 10% TBSA burn on the right lower extremity which of the following is a more appropriate management? A. Do a Split thickness graft B. Do a Full thickness graft C. Do a local flap D. Do a pedicled flap E. Do a free flap

410 A. Split thickness skin graft is used for large surface area wounds, frequently burns in which coverage is more impportant than function.

Few years after mastectomy a woman developed Stewart-Treves' Syndrome. Which of the following is the most appropriate management? A. Observe only and apply warm compress B. Prescribe antibiotics C. Keep affected arm elevated D. Do contralateral mastectomy E. Amputation of the limb

411 E. Stewart-Treves' Syndrome is the development of Lymphoangiosarcoma of the limb due to the absence of lymphatic drainage, amputation of the affected limb is the management.

A patient with a neck mass is suffering from Ludwig's Angina, What is the primary problem? A. Peritonsillar Absence B. Laryngeal Tumor C. Nasopharyngeal CA D. Adenoid Cystic Carcinoma E. Retropharyngeal Abscess

412 E. Retropharyngeal abscess presents with the following symptoms: fever, irritability, drooling, nuchal rigidity, dysphagia and airway obstruction (Ludwg's Angina)

Following an abdominal operation a patient had postoperative ileus, colonic motility will return after a period of: A. 12 hours B. 24 hours C. 48 hours D. 3-5 days E. 4-6 days

413 D. Colonic motility is the last to return after a postoperative ileus, SI motility within the 1st 24 hours, gastric motility within 48 hours.

A patient with Liver Cirrhosis will undergo a surgical shunt to decrease portal hypertension, which of the following has a lower incidence of hepatic encephalopathy? A. End-to-side portocaval shunt B. Side-to-side portocaval shunt C. Drapanas Shunt D. Distal splenorenal shunt E. all of the above

414 D. Selective shunts such as the Warren shunt or the distal splenorenal shunt has lower incidence of hepatic enephalopathy. SIMILAR TO PREVIOUS BOARD EXAM CONCEPT/PRINCIPLE

A young woman with prolonged OCP use was recently diagnosed with Hepatic Adenoma, Which of the following statements would you tell the patient? A. It is the most common benign lesion found in the liver B. It usually do not rupture spontaneously C. It has been there since the patient was born therefore is congenital D. It has a risk for malignant transformation E. It can rupture and cause Tumor Lysis Syndrome

415 D. Hepatic Adenoma may transform into a well- diferrentiated HCC, may also rupture and cause intraperitoneal bleeding

Which of the following compartments is not included in the abdominal assesment using FAST? A. Pericardium B. Hepatorenal Recess C. Splenorenal recess D. Foramen of Winslow E. Pouch of Douglas

416 D.

A 30 year old gravid patient on her 32nd week of gestation was diagnosed with Stage IIIB invasive ductal CA, what is the most appropriate/practical management? A. Terminate pregnancy at any trimester and do MRM B. Do lumpectomy only then complete MRM at Puerperium C. Initiate chemotherapy, All surgeries and radiotherapy at Puerperium D. Do MRM, Chemotherapy then radiation at puerperium E. Do Breast Conservation therapy

417 E. BCS is done in pregnant patients on their 3rd trimester, Chemo and RT should be done at Puerperium.

A patient with diverticulitis underwent a CT-scan of the abdomen, at which Hinchey stage is he in if there is noted Large mesenteric Abscess? A. Stage I B. Stage II C. Stage III D. Stage IV E. Stage V

418 B.

On a patient with suspected lower GI bleeding, after resuscitation and stabilizing the patient, what should be the next step of management? A. Direct to OR for immediate exploration B. Do Immediate EGD C. Do an urgent Colonoscopy D. Insert an NGT E. Administer Procoagulants

419 D. Insertion of an NGT should be done next to rule out an Upper GI bleeding first, because the most common cause of a LGIB is still UGIB.

To confirm a diagnosis of achalasia, the following should be requested A. EGD B. Manometry C. CT scan D. Barium swallow E. 24 hours pH monitoring

42 B. To confirm a diagnosis of achalasia, manometry should be requested.

A patient with a Tumor of the GI tract is undergoing chemotherapy with Imatinib, a Tyrosine Kinase inhibitor, in this case against c- KIT mutation, in which cell did the tumor probably arise from? A. Brunner's Gland B. Interstitial Cells of Cajal C. Goblet Cells D. Auerbach's Plexus E. Mucosa Associated Lymphatic Tissues

420 B. Imatinib is the Chemotherapeutic Drug of choice for GIST(Gastrointestinal Stromal tumor) which arises from Interstitial Cells of Cajal

What is considered to be the earliest sign of an on- going blood loss? A. Hypotension B. Cold clammy extremities C. Tachypnea D. Tachycardia

421 D.

Air emboli can form after a blunt or penetrating trauma involves an airway with entry of air into damaged pulmonary vein and before finally reaching the left heart. However, the volume of air required that will result in clinically significant embolus is at least: A. 50 cc B. 100 cc C. 200 cc D. 500 cc

422 B.

Meckel's diverticulum is associated with what is known as the "rule of 2's." One of these includes: A. Typically found about 2 feet from the ileocecal valve B. Most commonly seen in children older than 2 years C. Less than 2 cm long D. All of the above

423 A. The rule of 2's: 2% of the population, within 2 feet of the ileocecal valve, 2 inches in length, 2 types of heterotopic Mucosa, and presentation before the age of 2. The most common clinical presentations are: 1) lower gastro-intestinal bleeding secondary to an ulceration produced by heterotopic gastric mucosa; 2) intestinal obstruction; from internal volvulus or intussusception; 3) and local inflammation with or without perforation resembling acute appendicitis.

A 47-year old male with long-standing uncontrolled hypertension and a history of drug abuse presents to the emergency department with severe pain described as "tearing" sensation that radiates through his back. Which of the following conditions/findings will best serve as an absolute indication for immediate surgical intervention? A. Recurrent shooting chest pain B. Headache with at least 8/10 pain scale score C. Widened mediastinum with rub D. Urinary retention with rising BUN/creatinine levels

424 C. This is a case of aortic dissection which is defined as the separation of the layers within the aortic wall (intima-media) with blood entering the space leading to propagation of the dissection. Clues here are "uncontrolled hypertension, hx of drug abuse, and sudden onset of a severe "tearing" chest pain (classic). Presence of widened mediastinum with rub suggests significant propagation with strong consideration of developing cardiac tamponade. More specifically, emergency surgical intervention is preferred in Stanford type A (DeBakey type I and II) ascending aortic dissection and complicated Stanford type B (DeBakey type III) aortic dissections with specific clinical or radiologic evidence.

Carcinoembryonic antigen (CEA) can be used as a tumor marker for: A. Prostate B. Pancreas C. Breast D. Lung

425 C. Must know the tumor markers. CEA is found in malignancies of the GI (colon), cervix, ovary, breast, lung, and urinary tract. AFP for germ cell tumor and hepatocellular CA. CA 15-3 in breast CA. CA 19-9 mainly in pancreatic cancer but also colorectal. CA 125, mainly ovarian. Calcitonin for medullary thyroid carcinoma. hCG for gestational trophoblastic disease, choriocarcinoma, and germ cell tumor. Inhibin for sex cord-gonadal stromal tumor. PSA for prostatic cancer. S-100 for melanoma, sarcoma, astrocytoma. Vimentin in renal cell CA, sarcoma and endometrial CA.

True of extracellular volume excess: A. Edema is normally prevented by action of the lymphatics B. Involves increases in both plasma and interstitial fluid volumes C. Symptoms are usually restricted to cardiovascular changes D. All of the above E. None of the above

426 D.

A 25-year old motorcycle rider was involved in a vehicular accident. He was rushed to the trauma section and blunt abdominal trauma is suspected. He has BP of 120/80, HR 102, good peripheral pulses, soft abdomen with no signs of peritonitis; no other significant findings. FAST is negative. What is the next best step to perform? A. Patient is cleared for discharge. B. Repeat FAST in 30 minutes. C. A deep peritoneal aspirate is needed. D. Laparotomy is warranted.

427 B.

What is the most common type of skin cancer? A. Basal cell carcinoma B. Melanoma C. Squamous cell carcinoma D. Actinic keratosis

428 A.

Which of the following is a risk factor for breast cancer? A. Nulliparity B. Obesity C. Smoking D. HRT E. All of the above

429 E. All are risk factors.

What is the basal caloric requirement (kcal/Kg/day) of a normal healthy individual? A. 20 B. 25 C. 30 D. 35 E. 40

43 B. basal requirement: 25; mild stress: 25-30; moderate stress: 30; severe stress 35-40

This is considered to be the most common complication of modified radical mastectomy: A. Injury to the long thoracic nerve resulting in winging of the scapula B. Hypesthesia of the upper inner aspect of the ipsilateral arm C. Painless, gradual swelling of the involved arm D. Formation of seromas beneath the flaps or axilla

430 D.

A 59-year old male who is a chronic cigarette smoker and who is fond of betel nut chewing noted a slowly enlarging non-tender mass on his lower gingiva. This has extended to the floor of his mouth, significantly affecting movements of his tongue. To demonstrate involvement of the medullary cavity of the alveolar bones, one should order for: A. CT B. MRI C. Panorex D. UTZ

431 B. Carcinoma of the lower alveolar ridge spreads to the buccal mucosa and floor of the mouth, and can invade the periosteum of the mandible. And spread within medullary bone is best seen on MRI, where intermediate signal intensity on T1- weighted images is seen replacing the normal high T1-weighted signal intensity fatty marrow.

A 45-year old diabetic patient is complaining of recurrent epigastric pain for 2 months now. She notes that episodes are aggravated by reclining and eating of spicy foods, with frequent regurgitation of some of the recently ingested foods. To make an accurate diagnosis, this test must be performed: A. Endoscopy B. Urea breath test C. 24-hour pH monitoring D. Gastric manometry

432 C.

What is the most common malignant liver tumor? A. Hemangiocarcinoma B. Hepatocellular carcinoma C. Cholangiocarcinoma D. Metastatic liver tumor

433 D. Metastatic hepatic tumor is the most common tumor of the liver, while HCC is the most common PRIMARY tumor of the liver.

You performed an appendectomy during your first day in surgery. While doing the procedure, you had difficulty in looking for the base of the appendix. What is the landmark used to identify the location of the appendix? A. anterior taenia coli B. appendiceal artery C. posterior taenia coli D. posterior cecal artery E. colon

469 A.

Diverticulitis is diagnosed by CT imaging. Complicated cases are staged by what is called as the Hinchey staging. At what stage of diverticulitis is colonic inflammation with retroperitoneal or pelvic abscess is expected? A. Stage I B. Stage II C. Stage III D. Stage IV

434 B. Stage I: colonic inflammation with pericolic abscess; Stage III: purulent peritonitis; Stage IV: fecal peritonitis. Abscesses measuring <2cm diameter can be treated with parenteral antibiotics, whereas larger ones are best treated with CT-guided percutaneous drainage

The most common cause of appendicitis among pediatric patients: A. Fecalith B. Lymphoid hyperplasia C. Obstructing food remnants D. Tumors

435 B. Fecalith on the other hand, is said to be the most common cause of appendicitis in adults.

Which of the following is true about the hormonal response to an injury? A. Hyperglycemia is a hallmark of critical illness. B. Catecholamines may elevate up to 4x the normal levels lasting for 1-2 days before retruning to normal. C. Burn patients may exhibit elevated cortisol for up to 4 weeks. D. All of the above E. None of the above

436 D. The hyperglycemia is brought about by the relative insulin resistance as well as the actions of the counter-regulatory hormones (GH, catecholamines and cortisol). The high levels of sugars render the body in an immunosuppressive state and hence significant injuries also make patients susceptible to infections. B and C are also true.

During severe stress, the basal energy expenditure of the body is expected to be at: A. 25-30 kcal/kg/day B. 30-35 kcal/kg/day C. 35-40 kcal/kg/day D. >50 kcal/kg/day

437. B..*Must memorize some values. Simple calculations during exams are common, including in biochem and physio. Most frequently, they ask to calculate for adjustments in basal metabolic rates, energy expenditures, etc.

Which of the following is not true in the management of cardiac tamponade? A. Beck's triad is not often observed. B. SBP <90 mmHg warrants an ED thoracotomy C. Pericardiocentesis has a high rate of success as treatment modality for tamponade. D. All of the above E. None of the above

438 B. ED thoracotomy must be performed if SBP is <70 mmHg. It is best accomplished using a left anterolateral approach. Pericardiocentesis is successful in correcting the tamponade in about 80% of cases. Beck's triad include dilated neck veins, muffled heart sounds and low BP.

The technique used to surgically treat laryngeal malignancies which involves removal of neck lymph nodes in levels II through IV only is: A. Posterolateral neck dissection B. Lateral neck dissection C. Crile method D. Supraomohyoid neck dissection

439 B.

Single most important test in the evaluation of thyroid nodules: A. Core needle biopsy B. Fine needle aspiration biopsy C. Thyroid ultrasound D. Excision biopsy E. Incision biopsy

44 B. FNAB - single most important test in evaluation of thyroid nodules; core needle biopsy - for breast nodules

A salivary gland tumor patient underwent left parotidectomy. He then complained of numbness on the lower portion of his left auricle and periauricular skin. Based on this presentation and on the rate of incidence of such complication, the nerve most likely transected during the procedure is: A. Facial nerve B. Auriculotemporal nerve C. Greater auricular nerve D. Occipital nerve

440 C.

A 45 year-old man presents with diaphoresis, headache and palpitations. Imaging studies were done which revealed a 3.5cm mass in his left adrenal gland. Laboratory examination revealed elevated urine metanephrines. The treatment of choice for this patient is: A. chemoradiation B. tumor embolization C. adrenalectomy D. radiotherapy E. combined surgery and radiotherapy

441 C. Some benign adrenal tumors can be extremely harmful. Approximately 10% of adrenal masses secrete an excess of hormones that deleteriously affect the body. For example, hormonal hypersecretion can lead to such conditions as Cushing’s Syndrome, Conn’s Syndrome, and Pheochromocytoma. In these cases, adrenalectomy is the only surgical approach, open or laparoscopically.

It is a structure which demarcates the node levels in breast cancer: A. Pectoralis minor B. Pectoralis major C. Latissimus dorsi D. Serratus anterior E. Axillary artery

442 A. Pectoralis minor demarcates the levels of the nodes in breast cancer. Level I nodes are located below and lateral to the muscle. Level II nodes are at or behind pectoralis minor and level III nodes are medial and superior to the muscle.

A 75 year-old woman was going downstairs when she accidentally slipped and fell. She experienced severe pain on her right buttocks and was not able to walk. She was rushed to the Emergency Room. Xrays of the pelvis and femur were requested and done revealing femoral fracture. The most common part of femur fractured in this age group is: A. Supracondylar B. Intracapsular C. Extracapsular D. Intertrochanteric E. Subtrochanteric

443 B. Intracapsular fractures (femoral neck fractures) occurs between the edge of the femoral head and insertion of the capsule of the hip joint. Fractures of the femoral neck are more common in the elderly but fractures of the femoral shaft and supracondylar fractures are usually caused by violent trauma and most often occur in adolescents and young adults. Extracapsular fractures occur in between the insertion of the capsule of the hip joint and a line 5 cm below the lesser trochanter. Trochanteric fractures are extracapsular fractures that include intertrochanteric or pertrochanteric and reverse oblique fractures. Subtrochanteric fractures are extracapsular fractures where the fracture occurs below the lesser trochanter.

A neonate was born with a cleft palate. A surgical correction is advised. The main purpose for the surgical corection is the relief of: A. Dysphagia B. Dyspnea C. Speech abnormality D. Dental malocclusion E. Unexposed deformity

444 C. Cleft palate surgery is typically performed on infants that are between 6 and 18 months of age. Since the primary goal of repairing the palate is to avoid any abnormal speech development, the surgery is scheduled earlier before the child develops much speech.

A 12 year-old boy, weighing 40 kg will undergo circumcision. The maximum dosage of lidocaine that can be administered is: A. 50 mg B. 100 mg C. 150 mg D. 200 mg E. 250 mg

445 D. Maximum dosage of lidocaine is at 5mg/kg, 40kg X 5mg=200mg

A 50 year old male presented with abdominal pain and melena, with normal vital signs. He said that for the past few months, he experienced episodes of abdominal pain that awakens him at around 3- 4am. What is a non invasive procedure performed to check for pneumoperitoneum? A. upper endoscopy B. barium enema C. upright chest x-ray D. Exploratory laparotomy E. abdominal ultrasound

468 C. Upright chest x-ray is a non invasive procedure to check for pneumoperitoneum in 80%.

A 25 year-old male was invloved in a vehicular accident. He sustained a blunt injury to the bladder. It was noted that the urine extravasated to the perivesical space but not to the peritoneal space. The most probable part of urinary bladder that is perforated is: A. dome B. anterior C. lateral D. trigone E. posterior

446 A. Intraperitoneal bladder ruptures are described as large horizontal tears in the dome of the bladder. The dome is the least supported area and the only portion of the adult bladder covered by peritoneum. This is the weakest part of the bladder, since its muscle fibers are most widely separated. This type of injury is common among patients diagnosed with alcoholism or those sustaining a seatbelt or steering wheel injury. The classic cystographic finding is contrast extravasation around the base of the bladder confined to the perivesical space. The bladder may assume a teardrop shape from compression by a pelvic hematoma.

A 10 year-old child suffered fractures on his left 10th and 11th ribs after a fall. He was noted of signs of intraabdominal bleeding. An emergency explore laparotomy was done revealing laceration of the lower pole of the spleen. The best management for the above injury to the spleen is to: A. perform splenectomy B. perform repair of the laceration C. cauterize the spleen D. perform partial splenectomy E. ligate the splenic artery and do splenectomy

447 B. Once the spleen is injured, there are two options available: splenectomy or splenic conservation. Preservation of the spleen is clearly possible, particularly with minor capsular tears which constitute the commonest type of splenic injury. Splenorrhaphy has also been demonstrated to be effective in severely damaged spleens.

A 40 year-old patient with jaundice was suspected with a carcinoma through the "inverted 3 sign" seen on imaging. This malignancy is usually aymptomatic until in advanced stages: A. Carcinoma of the duodenum B. Carcinoma of the common bile duct C. Carcinoma of the ampulla of Vater D. Carcinoma of the head of pancreas E. Carcinoma of the body and tail of pancreas

448 D. Pancreatic cancer is a malignant neoplasm originating from transformed cells arising in tissues forming the pancreas. The most common type of pancreatic cancer, accounting for 95% of these tumors, is adenocarcinoma. Initially, pancreatic cancer of the head often tends to be silent and painless as it grows, and the later symptoms are usually nonspecific and varied. Frostburg's inverted 3 sign is a sign seen on a Barium examination where there is effacement and distortion of the mucosal pattern on the medial wall of the second part of the duodenum. It is suggestive of carcinoma of the head of the pancreas.

A 45 year-old diabetic woman underwent routine GI series. A presence of gallstone is an incidental finding but the patient is asymptomatic. The best management is: A. Emergency cholecystectomy B. Elective cholecystectomy C. Low cholesterol diet D. Regular follow-up E. No treatment needed

449 B. Most patients with gallstones remain asymptomatoc. However, even asymptomatic, elective cholecystectomy is indicated to elderly with diabetes, those who are in isolation from medical care for extended periods (seamen), and those with increased risk of GB cancer.

A 65 y/o M, smoker, obese presented at the ER with sudden, severe abdominal pain. The PE of the abdomen was normal. What is the most probable diagnosis? A. Acute mesenteric ischemia B. Acute perforated PUD C. Acute appendicitis D. Acute diverticulitis E. NOTA

45 A. Abdominal pain with severity out of proportion to PE findings is the hallmark of acute mesenteric ischemia.

A 54 year-old businessman had a solitary lesion on the right lobe of the liver on ultrasound. There was no jaundice or RUQ pain noted. Alkaline phosphatase was elevated but serum bilirubin is normal. Tha patient most probably has: A. Amoebic liver abscess B. Hepatic adenoma C. Metastatic liver cancer D. Pyogenic liver abscess E. Primary hepatocellular Ca

450 E. Hepatocellular carcinoma is the most common type of primary liver cancer. Most cases are secondary to either a viral hepatitis infection (hepatitis B or C) or cirrhosis. HCC may present with jaundice, bloating, easy bruising, loss of appetite, weight loss, RUQ abdominal pain, nausea, emesis, or fatigue. Total bilirubin, AST, alkaline phosphatase, albumin, and prothrombin time show results consistent with cirrhosis. AFP is elevated in 75% of cases.

A 62 year-old patient underwent Abdomino- perineal resection for rectal tumor. The principal drawback to APR is: A. High mortality B. Potential leak in the anastomosis C. Permanent colostomy D. Incontinence E. Impotence

451 C. An abdominoperineal resection or the Miles operation is principally indicated for resection of a rectal carcinoma situated in the distal one- third of the rectum or anal cancer. The principal drawback of this procedure is permanent colostomy as it involves removal of the anus, the rectum and part of the sigmoid colon along with the associated regional lymph nodes, through incisions made in the abdomen and perineum. During lateral mobilization of the rectum, Pudendal nerve may be injyured resulting in impotence in male patients.

A space that is also known as the Morrison's pouch: A. Subxiphoid B. Hepatorenal C. Splenorenal D. Retrocecal E. Cul-de-sac

452 B. Morrison's pouch is also known as hepatorenal or subhepatic space.

A young woman was rushed to the Emergency room who suffered from a blunt chest trauma in a vehicular crash. She is dyspneic, RR is 40 breaths per minute. Breath sounds markedly diminished on the right side. In inserting a thoracostomy tube, the lowermost limit of the parietal pleura at the lateral border of erector spinae is: A. 6th rib B. 8th rin C. 9th rib D. 10th rib E. 12th rib

453 E. Thoracostomy tube is indicated to patients suffering from a blunt chest trauma. It is done to decompress the chest with pertinent physical findings and hemothorax. The lowermost limit of pleura in midclavicular line is 8th rib, in midaxillary line is 10th rib, and at the lateral border of ereCtor spinae is 12th rib.

A 53 year-old woman comes in with a 9 cm X 8 cm mobile mass in her right breast for the past 4 years. Incision biopsy revealed a highly cellular fibrous stroma with cytsic areas and a leafy appearance on cut section. The best management for the above findings is: A. Radical mastectomy B. Lumpectomy C. Simple mastectomy D. Modified radical mastectomy E. MRM with radiotherapy

454 C. Phyllodes tumor are most often benign, but it is still critical to remove the entire tumor. Even if one cell is left behind, it will grow back. Treatment involves excision of both the tumor and a wide margin of healthy tissue surrounding the tumor. Treatment is wide local excision with a 1 cm margin of normal tissue is recommended. Mastectomy is indicated for patients with large lesions.

The most common abnormality of hemostatis in the surgical patients A. Anemia B. Hypovolemia C. Thrombocytopenia D. Leukopenia E. DIC

47 C.

A 40 year-old male presents with a painless swelling of the gland on the upper portion of his upper eyelid without external signs of inflammation, a chronic inflammatory lipogranuloma of a meibomian gland. Diagnosis is: A. External hordeolum B. Internal hordeolum C. Chalazion D. Meibomianitis E. Blepharitis

455 C. A hordeolum is a common disorder of the eyelid, an acute focal infection (usually staphylococcal) involving either the glands of Zeis (external hordeola, or styes) or, less frequently, the meibomian glands (internal hordeola). Essentially, a hordeolum represents an acute focal infectious process. A ChalaZion represents a ChroniC, noninfeCtious granulomatous reaCtion. However, ChalaZion often evolve from internal hordeolum.

A 30 year-old female patient noted redness on her left eye. She went to an ophthalmologist who found that she had subconjunctival hemorrage. Management for the finding is: A. Trabeculectomy of the affected eye B. Put patient on high back rest with patch on affected eye C. Topical antibiotic for 5 days and atropine ophthalmic drops BID D. Reassure patient that the condition will resolve after some time E. Surgical drainage of blood to prevent further complication

456 D. A subconjunctival hemorrhage is is bleeding underneath the conjunctiva. The conjunctiva contains many small, fragile blood vessels that are easily ruptured or broken. It is is typically a self-limiting condition that requires no treatment in the absence of infection or significant trauma.

A 40 year-old carpenter accidentally hit his right half of his face with a hammer resulting to a hyphema. It is characterized as: A. An angioma of the face B. Blood in the anterior chamber of the eye C. Hemorrhage in the maxillary sinus D. Lymphedema of the face E. Excessive lacrimation

457 B. Hyphema is the collection of blood in the anterior chamber of the eye. It may appear as a reddish tinge, or it may appear as a small pool of blood at the bottom of the iris or in the cornea. The most common causes of hyphema are blunt trauma, intraocular surgery, and lacerating trauma that my partially or completely block vision.

Malignant otitis externa is usually asociated with infection by: A. Streptococcus sp. B. Staphylococcus sp. C. Pseudomonas sp. D. Pneumococcus sp. E. H. influenza

458 C. Malignant otitis externa is caused by the spread of an otitis externa, also called swimmer's ear. The majority of cases are due to Pseudomonas aeruginosa, followed by a great number of other gram-positive and gram-negative species.

A 50 year-old patient came in with episodic vertigo, tinnitus and hearing loss. Upon examination, he was noted of sensorineural hearing loss with proportionate loss of discrimination, noise intolerance and reduced vestibular function. The probable diagnosis is: A. Vestibular neuronitis B. Acoustic neuroma C. Otogenic labyrinthitis D. Chronic labyrinthitis E. Meniere's disease

459 E. Meniere's disease or endolymphatic hydrops, is a disorder of the inner ear that can affect hearing and balance to a varying degree. It is characterized by episodes of vertigo, low- pitched tinnitus, and fluctuating hearing loss.

Most common presentation of Meckel's Diverticulum in adults A. Bleeding B. Perforation C. Intractability D. Obstruction E. NOTA

46 D. Intestinal obstruction is the most common presentation of Meckel's diverticulum in adults.

In an extensive maxillofacial trauma, adequate initial care should be directed to: A. Adequate prevention of infection B. Proper analgesia and anesthesia C. Maintenance of patent airway D. Meticulous soft tissues repair E. Proper skeletal reduction and fixation

460 C. ABC's. Administer oxygen and maintain a patent airway. Maintain an immobilized cervical spine at all times. Clear the mouth of any foreign body or debris, and suction any blood present.

Sutures are removed after 3-5 days in: A. legs B. scalp C. arm D. face E. Abdomen

461 D. Timing of suture removal are as follows: face - 3 to 5days; neck - 5 to 7 days; scalp - 7 to 12 days; upper extremity and trunk - 10 to 14 days; lower extremity - 14 to 28 days; soles, palms, back or over joints - 10 days

A 70 year old male farmer was diagnosed with malignant melanoma extending to the reticular dermis. What is the Clark level? A. V B. IV C. III D. II E. I

462 B. Clark level: I- superficial to basement membrane; II - papillary dermis; III - papillary- reticular dermal junction; IV - reticular dermis; V - subcutaneous fat

A 35 year old pegnant was alarmed when she noticed a blood dripping when she defecates. She also noted a non-painful anal mass that reduces spontaneously. What is your first consideration? A. 1st degree internal hemorrhoids B. 2nd degree internal hemorrhoids C. 3rd degree internal hemorrhoids D. 4th degree internal hemorrhoids E. 1st degree external hemorrhoids

463 B. Internal hemorrhoids present with bleeding and non-painful anal mass. It is classified into 4: 1st degree - bleeding with no protrusion; 2nd degree - bleeding with prolapse but reduces spontaneously; 3rd degree - bleeding with prolapse that is reduced manually; 4th degree - bleeding with prolapse that cannot be reduced. External hemorrhoids present with pain.

What is the stage of pressure ulcer when there is full thickness skin loss but not through the fascia? A. Stage I B. Stage II C. Stage III D. Stage IV E. Stage V

464 C. Stage I - nonblanchable erythema of intact skin; Stage II - partial thickness skin loss; Stage III - full thickness skin loss but not thru the fascia; Stage IV - full thickness skin loss + muscle and bone involvement. There is no stage V.

A 48 year old female had a breast mass which turned out to be a phyllodes tumor with less than 10 mitosis / 50 HPF. What is the appropriate treatment? A. Excision with a 1cm margin with axillary node dissection B. Excision with a 1 cm margin without axillary node dissection C. excision with a 5cm margin with axillary node dissection D. excision with a 5 cm margin without axillary node dissection E. MRM + Chemo + radiotherapy

465 B. This is a case of benign phyllodes tumor. It is malignant if there is more than 10 mitosis / 50 HPF. The treatment is excision with 1cm margin without the need of axillary node dissection. It may require mastectomy.

A 50 year old male with a malignant thyroid mass underwent total thyroidectomy with selective neck dissection. One of the levels of cervical lymph node is not removed. A. I B. II C. III D. IV E. V

466 A. For thyroid malignancies, posterolaternal neck dissection is removed which includes cervical lymph node levels - II, III, IV, and V.

Gastroesophageal reflux disease (GERD) is the most common esophageal pathology. The possible complications of GERD are the following except: A. esophagitis B. stricture C. aspiration D. Esophageal adenocarcinoma E. No exception

467 E.

A 70 year old male complained of rectal bleeding and altered bowel habits. He had left lower quadrant pain and tenderness on the LLQ upon palpation. What is the diagnostic of choice, and considered as the gold standard? A. contrast enema B. abdominal ultrasound C. computed tomography of the abdomen D. rectal biopsy E. x-ray of the abdomen

470 C. this is a case of diverticulitis. CT scan of the abdomen is the gold standard for diagnosis.

The gene found in chromosome 5 that is mutated in colorectal cancer. It is also the earliest mutation in colorectal Cancer. A. K-ras B. APC C. p53 D. DCC E. BRCA

471 B.

A 65 year old alcoholic complained of distended abdomen, pruritus and jaundice. He also had hematemesis. Upon further examination, there was splenomegaly and ano-rectal varices. What is the most accurate method of determining portal hypertension? A. hepatic venography B. upper endoscopy C. ultrasound of whole abdomen D. CT-scan of the abdomen E. MRI of the abdomen

472 A.

Rotter's node is at what level of axillary lymph node? A. Level I B. Level II C. Level III D. Level IV E. Level V

473 B. There are 3 groups of axillary lymph nodes: Rotter's node is the interpectoral group of node at level II.

While playing around the kitchen, a 5 year old child accidentally bumps off to her mother carrying a cup of hot coffee, spilling it over her body sustaining burn injury as follows: anterior trunk with partial thickness burn, right arm with full thickness burn and right leg with superficial burn. What is the burn size? A. 27% TBSA B. 41% TBSA C. 36% TBSA D. 18% TBSA E. 30% TBSA

474 A. Superficial burn should not be included in the calculating the % TBSA. In a child, using the rule of 9's, this is the distribution: face - 18%, anterior trunk - 18%, posterior trunk - 18%, each arm - 9%, each leg - 14 %. Therefore in this case, anterior trunk 18% + right arm 9% = 27%.

A 24 year old male sustained a stab wound on the chest perforating the esophagus. If he was brought in the ER 10 hours after the injury with BP of 90/60, pulse rate of 90, respiratory rate of 20, what will you do? A. Repair by primary closure B. Repair by secondary intention C. esophagectomy with gastric pull up D. observe E. transfuse blood as soon as possible

475 A. In managing esophageal perforation, the most favorable outcome is obtained following primary closure of the perforation within 24 hours resulting in 80-90%.

A patient with previous cesarian section 2 years ago returns with colicky abdominal pain, nausea, and obstipation. On physical examination, there was abdominal distention and hypoactive bowel sounds. If you want to confirm your diagnosis, what test will you do? A. CT scan of the abdomen B. ultrasound of the abdomen C. MRI of the abdomen D. abdominal series E. barium enema

476 D. This is a case of bowel obstruction. The confirmatory test for this is abdominal series consisting of radiograph of the abdomen in upright and supine position, and chest x-ray in upright position.

A 50 year old male had vehicular accident sustaining multiple abrasions and lacerations in the arms, face, and chest. X-ray of the chest revealed a multiple rib fracture and massive hemothorax. In adult, massive hemothorax is defined as: A. more than 1/3 of the blood volume in the pleural space B. more than 1.5 liters of blood in the pleural space C. more than 1/2 of blood volume in the pleural space D. more than 2 liters of blood in the pleural space E. more than 2.5 liters of blood in the pleural space

477 B. Massive hemothorax in adult is defined as more than 1.5 liters of blood in the pleural space, while in pediatrics, it is defined as more than 1/3 of the blood volume in the pleural space.

Patient had excruciating abdominal pain and dizziness after sustaining a blunt abdominal trauma 1 day ago. He was brought to the emergency department with BP of 90/60mmHg, respiratory rate of 26 cpm, and heart rate of 130bpm. What is the earliest sign of ongoing blood loss. A. tachypnea B. hypotension C. bradycardia D. tachycardia E. all of the choices

478 D. tachycardia is the earliest sign of ongoing blood loss, while bradycardia is an ominous sign heralding impending cardiovascular collapse.

In a post gastrectomy patient receiving a prenteral nutrition, what vitamin should be supplemented on a weekly basis because it is not part of commercially prepared vitamin solution. A. Vitamin A B. Vitamin D C. Vitamin K D. Vitamin B E. Vitamin E

479 C. Vitamin K is not part of any commercially prepared vitamin solution, so it should be supplemented on a weekly basis.

What is the risk of perforation of acute appendicitis? A. 25% by 24 hours from onset of symptoms B. 50% by 24 hours from onset of symptoms C. 75% by 24 hours from onset of symptoms D. 100% by 24 hours from onset of symptoms E. NOTA

48 A. The risk of perforation of acute appendicits is as follows: 25% by 24 hours rom onset of symptoms, 50% by 36 hours, and 75% by 48 hours

A 40 year old female obese patient came in due to right upper quadrant abdominal pain. Ultrasound of the hepatobiliary tree shows hyperechoic transluminal focus with shadowing. Your primary consideration is gallstone. What is the most common type of stone? A. cholesterol stone B. black pigment stone C. brown pigment stone D. bile sludge E. pigment stones

480 A. Cholesterol stone accounts for 80% of gallstone, while pigment stone accounts for 15-20% of gallstones.

The maximum dose of lidocaine for a 70-kg man is: A. 210 mg B. 300 mg C. 490 mg D. 700 mg

481 C. There were several computations on lidocaine (not mentioned whether with or without epi) during our board exam. The maximum dose for lidocaine without epinephrine is 3-5mg/kg. The maximum dose for lidocaine with epinephrine is 7mg/kg. 70kg x 7mg/kg= 490 mg.

A patient presents at the emergency room obtunded. On sternal rub, his eyes opened and he makes nonpurposeful movements. He moans when being asked something. What is his Glasgow coma scale? A. 7 B. 8 C. 9 D. 10

482 B. Opens eyes in response to pain (2) + makes nonpurposeful movement in response to pain (4) + makes incomprehensible words (2) = 8. SIMILAR TO PREVIOUS BOARD EXAM CONCEPT/PRINCIPLE.

With regards normal homeostasis and platelet function, which of the following is a vasoconstrictor? A. thromboxane B. PGE1 C. histamine D. serotonin

483 A. SIMILAR TO PREVIOUS BOARD EXAM CONCEPT/PRINCIPLE. pGE1 and histamine causes vasodilation.

For a normal adult female who weighs 60 kg, what is the total body water in liters? A. 24 L B. 36 L C. 48 L D. 33 L

484 D. The total body water of females is said to be 50- 55%, while for males it is 60%. 60kg x 0.55= 33 L

Where is the common location of carcinoma of the stomach? A. Along the lesser curvature B. At the cardia C. Along the greater curvature D. In the pylorus

485 D. Questions regarding "most commons" were frequently asked during our exam. Gastric carcinoma occurs 50% in the pylorus, 25% along the lesser curvature, 5% along the greater curvature and 10 % the cardia.

In orthopedics, "no man's land" refers to tendon injuries in what zone of the hand? A. Zone 1 B. Zone 2 C. Zone 3 D. Zone 4

486 B. Zone 1: distal to the insertion of the sublimis tendon. Zone 2: between the sublimis insertion and the proximal end of the flexor sheath. Zone 3: midpalm. Zone 4: carpal tunnel. Zone 5: distal forearm. Zone 2 is called no man's land because two flexor tendons run through the flexor sheath in this area and the prognosis after a flexor tendon injury in this area is worse than the other areas.

The most common type of fistula-in-ano is: A. intersphincteric B. transsphincteric C. suprasphincteric D. Extrasphincteric

487 A. Intersphincteric fistulas compose almost 40- 45% of fistulas-in-ano. Second most common would be transsphincteric fistulas, composing around 30%.

A 60-year-old woman is admitted for hematochezia, severe enough to decrease her hemoglobin to 8 g/dl. Which of the following is the best diagnostic procedure to use? A. esophagogastroduodenoscopy B. Double contrast barium enema C. colonoscopy D. selective angiography

488 C. A colonic mass is a common cause of anemia and hematochezia in the elderly.

A 3-year-old child presents at your office for chronic constipation. The mother noted that since birth, the child has only one stool per week. He was born term without complications but he did not pass his stool for the first 48 hours. What should be the initial management for this child? A. plain film of the abdomen B. a child psychiatry evaluation C. begin oral antispasmodic medication D. barium enema and rectal manometry

489 D. The diagnosis of Hirschsprung disease should be suspected in a child with intractable chronic constipation. A neonatal history of delayed passage of meconium is often obtained. Surgery is indicated as soon as diagnosis is made through barium enema and rectal manometry. A similar question came up during our board exam.

What is the most common appendicial tumor? A. Carcinoid tumor B. Mucinous tumor C. Serous tumor D. Brenner tumor E. Dysplastic tumor

49 A. Carcinoid tumors are the most common appendicial tumor. <5% of which are malignant.

The most common histologic type of urinary bladder malignancy is: A. adenocarcinoma B. Urothelial carcinoma C. Squamous cell carcinoma D. Rhabdomyosarcoma

490 B. The most common type of bladder cancer recapitulates the normal histology of the urothelium--transitional cell carcinoma or more properly urothelial cell carcinoma.

A 52-year-old man is diagnosed to have carcinoma of the rectum about 3 cm from the anal verge. Which procedure should be done assuming curative surgery is feasible? A. A hartmann procedure B. A low anterior resection plus total mesorectum excision C. An abdomino-perineal resection D. A left hemicolectomy

491 C. This would require an APR because the lesion is too close to the anus.

A patient suffered a stab wound at the left parasternal line at the level of 4th ICS. He was brought to the ER and was found to have a BP of 80/60. His heart sounds of 100 per minute are barely audible and his neck veins are distended. He is likely suffering from: A. hemorrhagic shock B. hemothorax C. cardiac tamponade D. tension pneumothorax

492 C. Beck's triad: muffled heart sounds, hypotension and distended neck point to the condition of acute cardiac tamponade. There were a few trauma questions during our exam.

According to the Clark Level, the invasion of a melanoma into the reticular dermis is considered: A. Level II B. Level III C. Level IV D. Level V

493 C. SIMILAR TO PREVIOUS BOARD EXAM CONCEPT/PRINCIPLE. Level I: confined to epidermis, II: papillary dermis, III: junction of papillary dermis and reticular dermis, IV: reticular dermis, and V: deep, subcutaneous tissue

A 45-year old woman occasionally notices bloody discharge from her left nipple. In this patient: A. An excision of the areola-nipple complex should be done B. The cause is likely to be ductal carcinoma in situ C. The most likely cause has a good prognosis D. A modified radical mastectomy should be done

494 C. The most common cause of bloody nipple discharge is benign intraductal papilloma which has good prognosis.

Metastasis to regional lymph nodes are common with all the following cancers, EXCEPT: A. Papillary thyroid carcinoma B. Squamous cell carcinoma C. Cervical carcinoma D. Basal cell carcinoma

495 D. Unlike squamous cell CA, basal cell carcinoma which is the most common type of skin cancer is known for its local invasion rather than metastasis.

A 45-year-old male patient presents at the ER with very painful, sudden vision loss, and halos around the light. Which is the most likely diagnosis? A. Open-angle glaucoma B. Closed-angle glaucoma C. Endophthalmitis D. Uveitis

496 B. Open-angle glaucoma usually is painless and does not have an acute presentation. Choices C and D are painful but the symptoms of sudden vision loss and appearance of halos are characteristic of closed angle glaucoma.

The liver is functionally divided into how many segments according to Couinaud? A. 4 B. 6 C. 8 D. 10

497 C. Couinaud proposed the liver to be divided into 8 segments in relation to the hepatic venous drainage.

A 43-year-old unmarried female with a BMI of 32, complained of RUQ pain and low-grade fever. On PE, she exhibited a positive Murphy's sign. What type of gallstone is the most common etiology for acute cholecystitis? A. pure cholesterol gallstones B. mixed cholesterol gallstones C. brown pigment gallstones D. black pigment gallstones

498 B. Generally speaking, up to 90 percent of gallstones are cholesterol stones. Pure cholesterol (>80% cholesterol) and mixed cholesterol (>50% cholesterol) are the most common causes of acute cholecystitis worldwide. SIMILAR TO PREVIOUS BOARD EXAM CONCEPT/PRINCIPLE.

An otorhinolaryngologist decided to perform a radical neck dissection for an oral cavity mass. Which of the following structures is NOT removed? A. Hypoglossal nerve B. Spinal accessory nerve C. Internal jugular vein D. Sternocleidomastoid

499 A. The rest of the choices are sacrificed during the standard radical neck dissection.

Which of the following maneuvers involve clamping of the portal triad and is used for hemostasis and control of bleeding during hepatic surgery? A. Madison meneuver B. Pringle maneuver C. Henderson’s maneuver D. Koch maneuver E. Rovsing maneuver

5 B. SIMILAR TO PREVIOUS BOARD EXAM CONCEPT/PRINCIPLE Source: Schwartz’s Principles of Surgery 8th ed p 1128

A 67-year-old otherwise healthy male presents with a TAA. What are the indications for repair? A. TAA > 8 cm B. TAA in asymptomatic patient C. TAA growth rate > 0.5 cm/year D. TAA > 5 cm in the setting of a connective tissue disorder E. AOTA

50 D. A. TAA > 6 cm B. TAA in a symptomatic patient (e.g., aortic insufficiency) C. TAA growth rate > 1 cm/year D. TAA > 5 cm in the setting of a connective tissue disorder

The esophagus is more prone to full-thickness tears than other gastrointestinal organs mainly because of its lack of which layer? A. mucosa B. submucosa C. muscularis propria D. Serosa

500 D. SIMILAR TO PREVIOUS BOARD EXAM CONCEPT/PRINCIPLE.

What is the maximum amount of 1% lidocaine solution that you can give a 70kg man? A. 35 mL B. 45 mL C. 55 mL D. 65 mL E. 75 mL

501 A. There were 3 board questions of this type in my time. toxic dose of lidocaine=5mg/kg, for any drug or solution, 1%=10mg/mL, for a 70kg patient: 70kgx 5mg/kg=350mg toxic dose; if 1% solution is used: 350mg/10mg/mL=35mL

A 39 year old man was involved in a shooting incident and was rushed to the ER for the multiple gun shot wounds to the abdomen and chest. Physical exam revealed patient to have BP of 60/40mmHg, with labored breathing, cold clammy skin and tachycardia at 160bpm.Right side of chest is hyperresonant to percussion with absence of breath sounds. Trachea is deviated to the right.What should you do immediately? A. Do Chest x-ray B. Exploratory laparotomy C. Do needle thoracostomy D. Observe E. Insert chest tube now

502 C. Immediate management of patient with tension pneumothorax is needle thoracostomy (2nd ICS MCL); Definitive management is CTT

What do you call the impaction of a stone in the infundibulum of the gallbladder that may mechanically obstruct the bile duct resulting to jaundice? A. Mittelschmerz B. Mirrizi syndrome C. Miller Fisher syndrome D. Meig's syndrome E. None of the above

503 B. A. Mittelschmerz- lower quadrant pain due to ovulation; MirriZi's syndrome-extrinsiC obstruCtion of the Common bile duCt from a CystiC duCt gallstone; C. Miller Fisher syndrome-variant of Guillain-Barre where cranial nerves are affected, leading to triad of ataxia, areflexia and ophthalmoplegia; D. Meig's syndrome- unuasual combination of hydrothorax, ascites, and ovarian fibroma

A 63 year old male was brought in your clinic because of complaints of vague epigastric pain that penetrates to the back for several months now with associated jaundice and weight loss. On physical exam, skin is icteric, gallbladder was distended and palpable.Abdominal ultrasound revealed no gall stones. What would you do next? A. Reassure patient. B. Do ERCP to clear the bile duct. C. Do a CT scan. D. Do scout film of the abdomen. E. Do exploratory laparotomy.

504 C. In the absence of gallstones, malignant obstruction of the bile duct is likely, and a CT scan rather than ERCP would be the next logical step. The current diagnostic and staging test of choice for pancreatic cancer is a multidetector, dynamic, contrast-enhanced CT-scan. Schwartz, 9th ed., 1222.

Reynolds pentad of cholangitis include the following signs and symptoms, except: A. fever B. jaundice C. septic shock D. right upper quadrant pain E. all of the above

505 E. also: mental status changes

What causes the initial vague, dull, diffuse epigastric pain in acute appendicitis? A. It is caused by the inflammatory process that involves the serosa of the appendix and in turn the parietal peritoneum in the region. B. Distention of the appendix stimulates the nerve endings of visceral afferent stretch fibers C. Appendiceal perforation D. All of the above E. None of the above

506 B. Schwatz 9th ed., 1075

A 6 year old little girl was brought to ER because of abdominal pain which started 12 hours prior to consult. History revealed that patient has just recovered from acute nasopharyngitis. Physical examination showed stable vital signs with presence of voluntary guarding with no true rigidity and diffuse abdominal tenderness rated as 5/10 in greatest intensity. What is the disease most often confused with acute appendicitis in children? A. Meckel's diverticulitis B. Crohn's enteritis C. Urinary tract infection D. Acute mesenteric adenitis E. Henoch-Schonlein purpura

507 D. Schwartz 9th ed., 1080

A 26 year old man was brought to the ER because of severe right lower quadrant pain associated with fever, nausea and vomiting and loss of appetite which started the night before ER consult. Physical examination revealed BP of 130/90mmHg, HR=108bpm, RR=18rpm, Temperature=38.9C. There is direct right lower quadrant and rebound tenderness with positive Psoas sign. CBC showed WBC count of 3 17,000cells/mm with predominance of neutrophils. what is the Alvarado score? A. 12 B. 11 C. 8 D. 10 E. 9

508 D. 1 point each: migration of pain, anorexia, nausea and miting, rebound tenderness, elevated temp, leukocytosis, left shift in leukocyte count; 2 points= RLQ tenderness and leukocytosis

What is currently the most accurate and most complete method for examining the large bowel that is highly sensitive for detecting even small polyps and allows biopsy, polypectomy, control of hemorrhage, and dilation of strictures? A. Colonoscopy B. Air contrast enema C. Flexible sigmoidoscopy D. Digital rectal exam E. Whole abdominal ultrasound

509 A. Schwatz 9th ed., 1045

What are the six P’s of acute limb ischemia? A. Pain, purple, paralysis, paresthesias, pulselessness, poikilothermia B. Pain, purple, pathologic, paresthesias, pulselessness, poikilothermia C. Pain, pallor, paralysis, paresthesias, pulselessness, poikilothermia D. Pain, pallor, pathologic, paresthesias, pulselessness, poikilothermia E. Pain, pallor, paralysis, paretic, pulselessness, poikilothermia

51 C. Pain, pallor, paralysis, paresthesias, pulselessness, poikilothermia

A 56 year old man was recently diagnosed of colonic adenocarcinoma located in the hepatic flexure. What is is most appropriate colectomy to perform in this patient? A. Ileocolic resection B. Right colectomy C. Extended right colectomy D. Transverse colectomy E. None of the above

510 C. Ileocolic resection- removes disease in the terminal ileum, cecum and appendix; right colectomy- for proximal colon carcinoma; extended right colectomy- lesions located at the hepatic flexure or proximal transverse colon; transverse colectomy- for lesions in the mid and distal transverse colon -Schwartz 9th ed., 1026

Which of the following hormones is expected to decrease following stress or injury? A. Cortisol B. Thyroxine C. Glucagon D. Epinephrine E. Growth hormone

511 B. Gonadotropin and Sex hormones, Prolactin in children, Insulin and insulin-like growth factors and Thyroid hormones, decrease following trauma or injury (SIT down). The rest of the hormones increase.

The following body fluids have similar electrolyte composition to plasma, except: A. Duodenum B. Bile C. Ileum D. Colon E. Pancreas

512 D. SIMILAR TO PREVIOUS BOARD EXAM CONCEPT/PRINCIPLE: Pearls

What is the most important treatment in metabolic acidosis? A. Fluid resuscitation to restore perfusion B. Bicarbonate infusion C. Alkalinization of urine D. Antibiotics administration E. None of the above

513 A. Pearls

The following constitutes a positive diagnostic peritoneal lavage in a patient who sustained an anterior abdominal wall stab wound, except: A. Red cell count >10, 000/mL B. WBC count >500/mL C. Amylase >19 IU/L D. Alkaline phosphatase >2 IU/L E. Bilirubin >0.01mg/dL

514 A. In an anterior abdominal stab wound an RBC count of >100,000/mL constitute a (+) DPL; >10,000/mL if thoracoabdominal stab wounds. The rest of the choices are correct for both anterior abomen and thoracoabdominal stab wounds. Schwartz 9th ed., 155

Which of the following patients would not require thoracotomy? A. A patient sustaining stab wound in the chest with an initial tube thoracostomy drainage of >800mL B. A patient with ongoing tube thoracostomy drainage of 800mL in a span of 3 hours C. A patient with 2 chest tubes with caked hemothorax D. A patient with open pneumothorax E. All of the above patients require thoracotomy

515 A. Indications for operative treatment of thoracic injuries: 1. initial tube thoracostomy drainage of >1000mL (penetrating injury) or >1500 mL (Blunt injury) 2. Ongoing tube thoracostomy drainage of >200mL/h for 3 consecutive hours in non-coagulopathic patients 3. Caked hemothorax despite placement of 2 chest tubes 4. selected descending torn aortas 5. Great vessel injury 6. pericardial tamponade 7. cardiac herniation 8. massive air leak from the chest tube with inadequate ventilation 9. tracheal or main stem bronchial injury diagnosed by endoscopy or imaging 10. open pneumothorax 11. esophageal perforation -Schwartz 9th ed., 172

Quinke's triad of hemobilia include the following, except: A. Right upper quadrant pain B. Upper GI bleeding C. Jaundice D. Fever E. None of the above

516 D. Schwartz 9th ed., 177

The following statements reagrding intussusception is true, except: A. Intussusception is the leading cause of intestinal obstruction in the young child. B. Hypertrophy of Peyer's patches in the duodenum from an antecedent viral infection acts as a lead point. C. Peristaltic action of the intestine then causes the bowel distal to the lead poin to invaginate into itself. D. Between paroxysms of crampy abdominal pain and intermittent vomiting, the infacnt may act normally. E. Positive Dance's sign show elongated mass in the right upper quadrant with an absence of bowel in the right lower quadrant.

517 B. Peyer's patches are located in the ileum.

A 43 year old man was rushed to the ER because of weakness and dizziness. History revealed 3 episodes of drak red bloody bowel movement, the last episode occuring 15 minutes ago. BP is 90/70mmHg and HR=118bpm, skin is cold and clammy. What should you do next? A. Do upper GI endoscopy B. Do emergency colonoscopy C. Insert NGT and aspirate D. Transfuse type specific packed RBC immediately E. Reaasure but do close observation of patient

518 C. Patient is bleeding inside. UGIB is still more common than LGIB, especially in young adults. Place NG tube and aspirate to define the area from which he is bleeding. Do endoscopy if NG tube returns significant amount of bright red blood.

A 55 year old man falls on his outstretched hand. He comes to the ER with a deformed wrist that looks like a fork. How would his X-ray show if he has Colles fracture? A. Dorsally displaced distal fragment B. Distal fragment is displaced anteriorly C. Shaft of ulna is fractured D. Dislocation of the lunate bone E. None of the above

519 A. Snell 8th ed., 480 B. Distal fragment is displaced anteriorly-Smith fracture; C. Shaft of ulna is fractured- Monteggia's fracture

Charaterized by chronic eczematoid eruption of the nippe A. Tubular carcinoma B. Mucinous carcinoma C. Paget's disease D. Inflammatory carcinoma E. Medullary carcinoma

52 C. Paget's disease is a chronic, eczematous eruption of the nipple. Large pale vacuolated cells in the rete pegs of epithelium (Paget cells) are pathognomonic.

Where is the most frequent site of peptic ulcer? A. 1st part of duodenum, posteriorly B. 1st part of duodenum, anteriorly C. Antrum of stomach D. Body of stomach E. None of the above

520 B. The Absite Review 3rd edition, 222.

A 37 year old man comes to the physician 12 hours after the onset of vomiting and abdominal cramps and swelling. He has had constipation for the past 4 days. He was diagnosed with Crohn disease 7 years ago. Vital signs are stable. Examination shows a diffusely distended, tympanitic abdomen and visible peristalsis; high pitched bowel sounds are heard. Rectal examination shows no stool in the rectal vault. An x-ray of the abdomen shows a small-bowel obstruction. Which of the following is the most likely cause of these findings? A) Colon cancer B) Ilocecal fistula C) Small bowel adhesions D) Small bowl fibrotic stricture

521 D.

Three days after undergoing colectomy for colon cancer, 77 year old women complains of being unable to catch her breath. Current medications include subcutaneous heparin and patient- controlled morphine. Her temperature is 37 C, pulse is 140 bpm, respirations are 28 cpm, and blood pressure is 110/60 mmHg. Pulse oximetry on room air sows an oxygen saturation of 89%. Arterial blood gas analysis reveals pH 7.38 pCO2 23 pO2 55. An ECG shows nonspecific ST-T wave changes. In addition to oxygen therapy, which of the following is the most appropriate next step in management? A) Spiral CT scan of the chest B) Anti-embolic stockings C) Intravenous administration of a recombinant tissue plasminogen activator D) Pulmonary angiography

522 A.

A 37 year old woman comes to the physician because of increasingly severe pain and masses in both breasts over the past 4 months. She has a 12 year history of similar episodes that were not as severe. The masses vary in size with her menstrual cycles. She has used an oral contraceptive for 16 years. Which of the following is the most likely diagnosis? A) Oral contraceptive induced breast changes B) Ductal papilloma C) Fibroadenoma D) Fibrocystic changes

523 D.

A 70-year-old woman has had increasing abdominal pain over the past 2 days. She has renal failure and has been receiving peritoneal dialysis for 18 months; her last treatment was 2 hours ago. She appears toxic. Her temperature is 39 C (102.2 F), and blood pressure is 140/90 mm Hg. Her abdomen is distended and diffusely tender to deep palpation with rebound tenderness. Leukocyte count is 18,000/mm3. Which of the following is the most appropriate next step? A. Comparison of abdominal fluid amylase with serum amylase activity B. Gram's stain of abdominal fluid C. Ultrasonography of the abdomen D. CT scan of the abdomen and pelvis

524 B.

A previously healthy 62-year-old man comes to the emergency department because of abdominal pain for 48 hours. His temperature is 38.6 C (101.5 F), blood pressure is 130/80 mm Hg, pulse is 110/min, and respirations are 15/min. Abdominal examination shows diffuse left lower quadrant tenderness with no peritoneal signs. Rectal examination shows no abnormalities; test of the stool for occult blood is negative. His leukocyte count is 14,700/mm3. Which of the following is the most appropriate next step in diagnosis? A) Exploratory laparotomy B) CT scan of the abdomen C) Colonoscopy D) Cystoscopy

525 B.

A 77-year-old woman comes to the physician because of a 2-day history of cramping abdominal pain and distention accompanied by nausea and vomiting. She is otherwise healthy and has no history of abdominal operations. Her temperature is 37.4 C (99.4 F), blood pressure is 110/86 mm Hg, pulse is 112/min, and respirations are 24/min. Cardiopulmonary examination shows no abnormalities. Examination of the abdomen shows distention and mild diffuse tenderness; bowel sounds are high-pitched. An x-ray film of the abdomen shows air-fluid levels throughout the small bowel and air in the liver; there is no gas in the colon or free air. Which of the following is the most likely diagnosis? A) Adhesive small-bowel obstruction B) Cecal cancer C) Gallstone ileus D) Mesenteric infarction

526C.

An 18-year-old man comes for an examination prior to participation in school sports. He states that he has had a dull ache in the scrotum since being hit in that area during a basketball game 2 months ago. Examination shows a 2-cm, hard, nontender mass in the right testicle. The mass does not transilluminate or change in size when the patient is placed in the supine position. Which of the following is the most likely cause? A) Cystic dilations of the efferent ductules B) Dilated pampiniform venous plexus C) Fluid accumulation within the tunica vaginalis testis D) Germinal cell tumor

527 D.

A 64-year-old woman has moderately severe postoperative pain 1 day after a total abdominal hysterectomy and bilateral salpingo- oophorectomy. Which of the following is the most appropriate analgesic pharmacotherapy? A) Patient-controlled intravenous morphine B) Oral diazepam C) Oral ibuprofen D) Intermittent intravenous naloxone

528 A. Patient-controlled analgesia is the treatment of choice for moderate to severe postop pain

A 67-year-old woman is brought to the emergency department because of severe chest pain 4 hours after undergoing outpatient endoscopy and dilatation of an esophageal stricture caused by reflux. At discharge, she reported no chest pain. Three hours later, she vomited a small amount of blood and had severe pain. She is pale. Her temperature is 38 C (100.4 F), blood pressure is 140/85 mm Hg, pulse is 125/min, and respirations are 22/min. Examination shows crepitus in the neck and moderate epigastric tenderness. The lungs are clear to auscultation, and breath sounds are equal bilaterally. Rectal examination shows no masses; test of the stool for occult blood is positive. Which of the following is the most likely cause of these symptoms? A) Bleeding from erosive esophagitis B) Esophageal perforation C) Mallory-Weiss syndrome D) Perforated gastric ulcer

529 B.

A 10-year-old presents with a painless swelling along her lateral neck. Ultrasound reveals a fluid- filled cyst. What is the most common origin of this anomaly? A. Thyroglossal duct cyst B. First branchial cleft cyst C. Second branchial cleft cyst D. Third branchial cleft cyst E. Fourth branchial cleft cyst

53 C. The second branchial cleft anomaly is the most common and usually presents on the first decade of life. Treatment is by complete excision

A 67-year-old woman comes to the physician because she has had a lesion on her right eyelid for 3 months. She has no history of similar lesions. Examination shows a 2 x 1.5-cm lesion on the lower lid of the right eye. A biopsy specimen of the lesion shows basal cell carcinoma. Which of the following is the most appropriate step in management? A) Wide (1-cm) excision B) Mohs micrographic surgery C) Radiation therapy D) Chemotherapy

530 B.

A 37-year-old man is brought to the emergency department 6 hours after the onset of constant, increasingly severe abdominal pain and nausea. His symptoms awoke him from sleep, and he has vomited once since that time. He is in acute distress and lying in the fetal position. Any movement exacerbates the pain. His temperature is 37.8 C (100 F), blood pressure is 108/68 mm Hg, pulse is 112/min, and respirations are 24/min. Examination shows a rigid abdomen; bowel sounds are absent. Laboratory studies show: Hemoglobin 14 g/dL Leukocyte count 18,200/mm3 Platelet count 150,000/mm3 Urea nitrogen (BUN) 34 mg/dL Creatinine 1.9 mg/dL Total bilirubin 1.2 mg/dL An x-ray film of the chest shows a small amount of free air under the left diaphragm. Administration of antibiotics and fluids is begun. Which of the following is the most appropriate next step in management? A) Barium swallow B) CT scan of the abdomen C) Intravenous administration of an H2-receptor blocking agent D) Laparotomy

531 D.

A 19-year-old man is brought to the emergency department 45 minutes after sustaining a single, large stab wound to the right upper quadrant of the abdomen. He is obtunded. His blood pressure is 60/palpable mm Hg, and pulse is 148/min. Breath sounds are equal bilaterally. Examination shows a 4-cm laceration in the right upper quadrant in the midclavicular line. The abdomen is distended. Which of the following is the most appropriate next step in management? A) X-ray films of the abdomen and pelvis B) CT scan of the abdomen C) Peritoneal lavage D) Laparotomy

532 D.

Which of the following is the most significant risk factor for invasive breast cancer when screening a patient for risk? A. >2 first-degree relatives with breast cancer B. 2 previous breast biopsies in a patient <50 years of age C. Age <12 at menarche D. Atypical hyperplasia in a previous breast biopsy

558 A.

A 70-year-old nursing home resident is admitted to the hospital because of progressive obtundation over the past 2 days. He has tachycardia, tachypnea, and hypotension. Bilateral basilar crackles and an S3 gallop are heard on auscultation. Examination shows jugular venous distention and peripheral edema. Swan-Ganzcatheterization shows a cardiac index of 1.8 L/min/m2 (N=2.5â€" 4.2), a mean Pulmonary capillary wedge pressure of 23 mm Hg (N=1â€"10), and markedly increased systemic vascular resistance. Which of the following is the most likely diagnosis? A) Cardiogenic shock B) Hypovolemic shock C) Neurogenic shock D) Septic shock

533 A.

Six weeks after spontaneous drainage of an anal abscess, a 32-year-old man has persistent blood- stained purulent fluid on his underwear. He has not had significant anal pain since drainage of the boil. Bowel movements are normal. Which of the following is the most likely diagnosis? A) Anal fissure B) Fistula in ano C) Pruritus ani D) Thrombosed external hemorrhoids

534 B.

A previously healthy 16-year-old boy comes to the physician because of persistent pain in his left testicle for 24hours. He has not had any penile discharge. There is no history of trauma, but he plays soccer every day. Two months ago, he had sexual intercourse for the first time, and he used a condom. Examination shows an edematous, erythematous, exquisitely tender left scrotum that is lower than the right. Elevating the left testicle relieves the pain. The cremasteric reflex is present. Urinalysis shows 10 leukocytes/hpf and 1+ leukocyte esterase. A technetium 99m scan shows increased uptake in the left testicle. Which of the following is the most likely cause of the pain? A) Cystitis B) Epididymitis C) Spermatocele D) Testicular torsion

535 B. Elevating the left testicle relieves the pain >> Epididymitis

A 32-year-old woman has had a lump in her neck for 6 months. She has a 1.2-cm solitary left thyroid nodule. Fine-needle aspiration cytology of the mass is consistent with a low-grade papillary malignancy. Which of the following is the most appropriate next step in management? A) Thyroid scan B) 131I therapy C) Propylthiouracil therapy D) Left thyroid lobectomy

536 D.

A 75-year-old man has had hypertension for 25 years. There is an unusually prominent pulsation of the abdominal aorta in the upper midabdomen. A systolic bruit is heard at this site. Femoral, popliteal, and pedal pulses are present. Which of the following is the most appropriate initial diagnostic study? A) X-ray film of the abdomen B) Abdominal ultrasonography C) Doppler ultrasonography of the arteries of the legs D) Abdominal aortography

537 B.

A 77-year-old woman is brought to the emergency department after collapsing at home. Six hours ago, she had the sudden onset of massive bright red rectal bleeding. On arrival, her blood pressure is 90/60 mm Hg, and pulse is 120/min. Abdominal examination shows no abnormalities. Insertion of a nasogastric tube yields clear aspirate. Her hematocrit is 28%. Which of the following is the most likely diagnosis? A) Colon cancer B) Diverticulosis C) Duodenal ulcer D) Diverticulitis

538 B. Most common cause of hematochezia in elderly >> diverticuLOSIS

A previously healthy 85-year-old man has had abdominal distention, decreased caliber of stools, and decreased appetite over the past 2 weeks and a 9-kg (20-lb) weight loss over the past 3 months. On sigmoidoscopy, he is found to have a constricting adenocarcinoma of the sigmoid colon; imaging studies show three 1-cmnmetastases to the liver. Which of the following is the most appropriate next step in management? A) Radiation therapy B) Chemotherapy C) Combination radiation therapy and chemotherapy D) Resection of the colon tumor

539 D. Surgery to relieve obstruction

A 25-year-old female with protein C deficiency presents with the acute onset RUQ pain and jaundice. Her evaluation reveals occlusion of her hepatic veins at the level of her IVC. What is the immediate first treatment? A. IVC filter B. Systemic anticoagulation C. Systemic fibrinolysis D. Thrombolectomy E. AOTA

54 B. The syndrome described is Budd-Chiari Syndrome. First line therapy is systemic anticoagulation

A 32-year-old man is brought to the emergency department 30 minutes after being involved in a motor vehicle collision. He was the restrained driver. On arrival, he is alert and has shortness of breath. His blood pressure is 80/50 mm Hg, pulse is 130/min, and respirations are 30/min. Examination shows jugular venous distention and abrasions over the left hemithorax. The trachea is deviated to the right. Breath sounds are absent on the left. Which of the following is the most likely cause of the hypotension? A) Cardiogenic shock B) Congestive heart failure C) Decreased systemic vascular resistance D) Decreased venous return

540 D. tension pneumothorax >> decreased venous return

A 48/F, known case of gallstone disease, is scheduled for elective open cholecystectomy. Obese and a known hypertensive and diabetic, she is being maintained on Aspirin by her internist to reduce risk for cardiovascular events. How long should one stop aspirin prior to surgery? A. 14 days B. 12 days C. 7 days D. 3 days E. 1 day

541 C.

A 22/F comes in with a straight facial laceration. To achieve good cosmetic outcome, which suturing technique and which suture type would be most appropriate? A. Simple interrupted, Vicryl B. Subcuticular running suture, Nylon C. Subcuticular running suture, Vicryl D. Vertical mattress, Vicryl E. Horizontal mattress, Nylon

542 C. The subcuticular running suture is often used by plastic surgeons to close straight lacerations on the face. An absorbable suture, such as Monocryl or Vicryl, is used.

For patients on chronic corticosteroids, intake of this vitamin can reverse the inhibitory effects of corticorsteroids on wound healing. Supplemental doses are thus recommended during the pre- and post-operative periods. A. Vitamin A B. Pyridoxine C. Thiamine D. Vitamin C E. Vitamin D

543 A. Supplemental vitamin A can reverse the inhibitory effects of corticosteroids on wound healing. Vitamin A also can restore wound healing that has been impaired by diabetes, tumor formation, cyclophosphamide, and radiation.

For the multiply injured patient who sustained severe blunt trauma, the following diagnostic examinations are routinely ordered, except: A. Lateral cervical spine radiograph B. Cranial CT scan C. Chest X-ray D. Pelvic radiograph E. None of the above

544 B. Remember the 'big three' radiographic exams for severe blunt trauma.

Compartment syndrome should always be ruled out in any severe injuries of the forearm. Which of the following is the typical sequence of symptoms? a. Pain, weakness, increased tenseness, hypoesthesia b. Pain, hypoesthesia, weakness, increased tenseness c. Pain, increased tenseness, hypoesthesia, weakness d. Pain, weakness, hypoesthesia, increased tenseness

578 A. Pain, weakness, increased tenseness, hypoesthesia

A 28/M who crashed his car into another vehicle was brought into the emergency room. Vital signs on admission are as follows: BP 90/60, HR 118, RR 22, T 37.5 C. On PE, you note direct and rebound tenderness on all quadrants with guarding. Which of the following would be your next step in management? A. Perform a Focused Abdominal Sonography for Trauma (FAST) B. Perform an abdominal CT scan C. Send the patient to Radiology for chest, pelvic and cervical spinal radiographs D. Perform a diagnostic peritoneal lavage. E. Perform an exploratory laparotomy

545 E. Patient with peritoneal signs? Do exploratory lap.

A 45/M crashed his car into another vehicle; and was brought to the ER. On PE, patient has multiple bruises over the chest. CXR showed multiple rib fractures, no pneumohemothorax, and clear lung parenchyma. After two days in the ER, however, patient started having respiratory distress. Repeat CXR showed "white out" of bilateral lungs. Which of the following steps is most important? A. Insertion of a chest tube B. Use of diuretics C. Close observation D. Fluid restriction E. Intubation and mechanical ventilation with positive pressure

546 E. Patient has pulmonary contusion associated with a flail chest. Positive airway pressure and respiratory support are key to management.

A 38/F came in to your clinic for a firm, well- demarcated, movable 2 x 2 cm mass in the upper outer quadrant of the left breast. Which of the following would be the most appropriate next step? A. Reassure the patient that this is a benign condition and is not a cause for concern. B. Do fine needle aspiration for cytology. C. Do core needle biopsy. D. Do excisional biopsy of the mass. E. Do a mammogram on both breasts.

547 B. From the PCS CPG on breast masses: In patients with a palpable breast mass in which cancer is suspected, BIOPSY is mandatory. ( Level I, Category A ) Fine needle aspiration cytology ( FNAC ) is the initial diagnostic procedure in patients with a palpable breast mass in which cancer is suspected.

Which of the following benign breast disorders does NOT confer an increased cancer risk? A. Lobular carcinoma in situ B. Atypical ductal hyperplasia C. Sclerosing adenosis D. Florid hyperplasia E. None of the above

548 C. Sclerosing adenosis and intraductal papilloma does NOT confer cancer risk.

A 55/F with a long-standing history of burning retrosternal pain and heartburn, underwent endoscopy, which revealed peptic esophagitis and Barett's esophagus. She has been taking PPIs with some relief of symptoms. What is the most appropriate next step in therapy? A. Advise dietary modifications to reduce acid reflux. B. Increase doses of proton pump inhibitors. C. Do Nissen fundoplication. D. Do esophagectomy. E. Do repeat biopsy regularly to screen for cancer.

549 C. Barrett's esophagus is a premalignant condition, necessitating antireflux surgery.

What is the most common location of an insulinoma? A. Pancreatic head B. Pancreatic tail C. Evenly distributed throughout the pancreas D. Gastrinoma triangle E. Pancreatic body

55 C. Insulinoma: Evenly distributed throughout the pancreas Gastrinoma: Gastrinoma triangle Glucagonoma: Pancreatic tail

A 50/F presents with a year long history of heartburn and chest pain, unrelated to food intake. A few weeks prior to consult, he has developed intermittent dysphagia to both solids and liquids, and as a result lost weight. Barium swallow done revealed a dilated esophagus that tapers smoothly to a beak-like narrowing at the gastroesophageal junction. What is considered to be the MOST effective treatment for this condition? A. Calcium channel blockers and nitrates B. Intrasphincteric injection of botulinum toxin C. Pneumatic dilatation D. Laparoscopic myotomy E. Peroral endoscopic myotomy

550 D. Laparoscopic Heller cardiomyotomy remains the MOST effective treatment for achalasia.

A patient who underwent MRM (modified radical mastectomy) for breast cancer complains of loss of sensation on medial aspect of arm. Patient denies loss of motor movement. A. injury to the thoracodorsal nerve B. injury to long thoracic nerve C. injury to interchostobranchial nerve D. hyperextension of arm during surgery

551 C.

A 35 year old patient on her 10th week AOG presents with a 3 cm breast mass right. On core needle biopsy, it showed invasive ductal carcinoma, ER(+) PR (+). The most appropriate management would be: A. modified radical mastectomy (MRM) + adjuvant chemotherapy B. breast conservation + adjuvant chemotherapy C. neoadjuvant chemotherapy then MRM post- partum D. MRM and bilateral oopherectomy E. Watchful waiting

552 A.

A 28 year old Male was referred to GS 1 for evaluation of ANM. On histological examination of the thyroid gland revealed large nuclei with pale staining, ground glass appearance. What is the diagnosis? A. Colloid adenomatous goiter B. Papillary thyroid carcinoma C. Follicular thyroid carcinoma D. Medullary thyroid carcinoma E. Anaplastic thyroid carcionoma

553 B. orphan annie

48/M blacksmith had an ifected wound over a cut on the right arm. It presented with purulent discharge and some granulation. What would be the best management? A. Close the wound primarily B. Advise the patient on wound care and close the wound at a later date C. Advise patient on daily wound cleaning and let the wound heal by secondary intention D. Wash and clean the wound well and close primarily E. Do debridement

554 C.

A patient with RUQ abdominal pain and fever was brought to the OPD. On PE, there was a note of tenderness on the RUQ but no jaundice. He brought an UTZ done outside with note of predominantly cystic mass at the right lobe. the most probable diagnosis is: A. hepatic cyst B. amoebic abscess C. pyogenic abscess D. hepatic adenoma E. hepatocellular carcinoma

555 B.

The most common location of the superior parathyroid gland is: A. Dorsal to the recurrent laryngeal nerve, within 1 cm of the junction of the RLN and inferior thyroid artery B. Ventral to the RLN, within within 1 cm of the junction of the RLN and inferior thyroid artery C. Dorsal to the recurrent laryngeal nerve, within 3 cm of the junction of the RLN and inferior thyroid artery D. Ventral to the RLN, within within 3 cm of the junction of the RLN and inferior thyroid artery

556 A.

The most common ectopic mucosa present in Meckel’s diverticulum is: A. Colonic mucosa B. Pancreatic mucosa C. Ovarian mucosa D. Gastric mucosa

557 D.

A 34 y.o. male comes into the ER with the following vital signs HR 120s BP 90/60 RR 30. On initial survey, patient was noted to have multiple stab wounds in L lateral neck (gurgling), 4th ICS R midclavicular (profusely bleeding), RLQ with intestinal evisceration (non-bleeding) and L groin with pulsatile bleeding. The appropriate step in the management of this patient is: A. Vaselinized gauze over L neck B. Vaselinized gauze over R chest C. Make sure the patient has a patent airway D. Pressure over the L groin

559 C.

What is the most common cause of fever in the first 48 hours post-operatively? A. Atelectasis B. Pneumonia C. UTI D. Wound infection E. Seroma

56 A. Atelectasis is the most common cause of in the first 48 hours post-operatively.

A 72 yo male complains to have decreased stool caliber with blood streaks and weight loss (~15% over 3 months). The patient is suffering from: A. Adenocarcinoma found on flexible sigmoidoscopy, 22 cm from anal verge B. Squamous cell carcinoma at anal verge C. Adenocarcinoma found on esophagogastroduodenoscopy, 3 cm distal from esophagogastric junction in lesser curvature D. Adenocarcinoma found on colonoscopy, at cecum.

560 A.

The surgical margins of a patient diagnosed with melanoma staged at T3N0M0 is: a. 1cm b. 2cm c. 3cm d. 4cm

561 B. 2 cm is the margin for melanoma with 2-4mm in depth (T3)

A 24 year old female, 5'0'' weighing 140lbs, complained of recurrent chest tightness with ascending burning pain came to your clinic. the best method to confirm your impression is: a. esophageal manometry b. 24 hour pH monitoring c. esophagoduodenoscopy d. upper GI series

562 B. 24 hour pH monitoring to document GERD.

A patient presents to you with a left preauricular mass that was irregular, and firm in consistency that has persisted for more than a year. the mass was characteristically noted to be painful. the tumor most likely present in this patient is: a. pleomorphic adenoma b. mucoepidermoid carcinoma c. adenoid cystic carcinoma d. acinic cell carcinoma

563 C. Adenoid cystic Ca is notable for its perineural invasion

The nerve in close proximity to the submandibular duct that needs to be identified during resection of the gland is the: a. greater auricular nerve b. lingual nerve c. glossopharyngeal nerve d. anterior jugular nerve

564 B.

The anterior belly of the digastric muscle, the hyoid bone, and the midline of the neck delineate what triangle of the neck? a. submental b. muscular c. carotid d. digastric

565 A.

A true anatomic precursor of breast cancer: a. LCIS b. DCIS c. Intraductal papilloma d. Sclerosing adenosis

566 B. : DCIS. LCIS is only a marker of increased risk since Invasive ductal Ca develops more than Invasive lobular Ca in a patient who had history of LCIS.

The triad of right upper quadrant pain, palpable mass, and jaundice describe this condition: a. ascending cholangitis b. acute cholecystitis c. choledochal cyst d. hepatic adenoma

567 C. Choledochal cyst; Ascending cholangitis characteristically presents with Charcot’s triad of pain fever and jaundice.

The most common cause of acute appendicitis in children is: a. fecalith b. Enterobius vermicularis c. typhoid ileitis d. lymphoid hyperplasia

568 D.

The most common location of the appendix is: a. retrocecal b. pelvic c. femoral d. obturator

569 A.

What sedative medication has caloric value? A. propofol B. midazolam C. desflurane D. fentanyl E. NOTA

57 A. Propofol delivers 1 kcal/cc in the form of lipid.

The layer posterior to the rectus abdominis muscle at the level 2cm above the symphisis pubis comes from a. aponeurosis of external oblique b. aponeurosis of internal oblique c. transversus abdominis d. transversalis

570 D. Below the arcuate line, the posterior layer beneath the rectus muscle is derived from the transversalis

Strangulated hernias are differentiated from incarcerated hernias by the following except: a. Pain out of proportion to examination findings b. Fever or toxic appearance c. Pain that persists after reduction of hernia d. A bulge felt above the inguinal ligament

571 D. A bulge felt above the inguinal ligament

Modified radical neck dissection spares the following structures except: a. Internal jugular vein b. Spinal accessory nerve c. Greater auricular nerve d. Sternocleidomastoid muscle

572 C. MRND spares the IJA, SCM, and CN XI

A 17 year old sexually active male patient comes to your clinic for a palpable nontender mass of the right testicle, your primary impression is: a. testicular cancer b. TB of the testis c. secondary syphilis d. Gonococcal epidydimitis

573 A. Answer: Any solid, firm mass within the testis should be considered testicular cancer until proven otherwise. Prompt diagnosis and early treatment are required for cure.

Anal fissure most often occurs: a. anterior midline b. posterior midline c. anterolateral d. posterolateral

574 B. Answer: Most anal fissures occur in the posterior midline, with the remainder occurring in the anterior midline (99% of men, 90% of women). Two percent of patients have anterior and posterior fissures. Fissures occurring off the midline should raise the possibility of other etiologies (eg, Crohn disease), an infectious etiology (eg, sexually transmitted disease, acquired immunodeficiency syndrome or cancer.

In estimating total body surface area (TBSA) involved, what depth or thickness of burn injuries should be considered? a. Full b. Full and superficial c. Full and partial d. Full, partial and superficial

575 C.

If you suspect carbon monoxide poisoning in a semi-conscious patient, what is the best action to perform? a. Bronchoscopy b. Administer methylene blue c. Intubate the patient and give 100% oxygen d. Provide 100% supplemental oxygen via facemask

576 C. establish airway via ET tube since the patient is already semi to unconscious and unable to protect his/her airway, subsequently, give 100% oxygen.

Which of the following is the best radiographic view to check the fracture of the forearm? a. Forearm AP-Oblique b. Forearm AP-Lateral c. Forearm pronation-lateral d. Forearm pronation-supination

577 B.

A 50 year old female consulted at the OPD clinic due to an anterior neck mass. On physical examination, there was a 3 x 4 cm diffuse, firm, non-tender mass at the right neck which moves during swallowing. No cervical lymphadenopathy was noted. TSH and FT4 were normal. If the biopsy turned out to be colloid adenomatous goiter (CAG), the most likely indication for surgery is: a. Cosmesis b. Potential for malignancy c. Prevention of recurrence d. Compression of surrounding structures

579 A. Answer: Cosmesis. Colloid goiter has no malignancy potential, and does not usually cause compression of underlying structures (unlike Ca).

Most common origin of masses in the pre-styloid compartment of the parapharyngeal space A. Salivary B. Paraganglioma C. Neurogenic D. Lymph nodes E. Vascular

58 A. 45% of the pre-styloid tumors/masses are pre- styloid in origin.

Which of the following is an easy and reliable way to determine if the tympanic membrane is intact on otoscopy? a. Calorics testing b. Pneumatoscopy c. Tuning fork test d. Tympanometry

580 B.

What is the normal weight of the thyroid gland? A. 10 g B. 20 g C. 30 g D. 40 g E. 50 g

581 B. The normal thyroid has a weight of 20 g.

Which interleukin is responsible for immunoglobulin production? A. IL-1 B. IL-2 C. IL-3 D. IL-6 E. IL-12

582 B. IL 2 is responsible for lyphocyte proliferation and Ig production

Total body water in a young adult female comprised how many percent of total body weight? A. 50% B. 55% C. 60% D. 70% E. 75%

583 A. A young female has 50% of body weight since they have more fat per kg of body weight than males.

What is the important test in determining a patient perioperative risk for bleeding? A. Platelet count B. PT C. aPTT D. Hemoglobin E. History taking

584 E. A directed bleeding history is the most appropriate test in determining risk of bleeding.

Patient has been scheduled to undergo laparoscopic cholecystectomy. What is the surgical wound classification? A. Class I B. Class ID C. Class II D. Class III E. Class IV

585 C. Class II is the classification for wounds that violate the integrity of the GI or GU tract under controlled circumstances (antibiotic prophylaxis).

A 24 year old was immediately rush to the ER due to a gunshot wound to the abdomen. On initial examination, patient was noted to be drowsy with pallor, blood continuously gush out from the wound. The next step in management is? A. Insert large bore needles and start fluid resuscitation B. Control bleeding from the wound C. Intubate the patient D. Do diagnostic periteoneal lavage E. Immediately wheel the patient to the OR

586 C. The first step is evaluation of the ABCs and altered mental status is the most common indication for intubation.

A 30 year old woman suffered abdominal trauma after being assualted by a thief. In the ER, the surgical resident decided to perform FAST. Which of the following is NOT a location evaluated by the procedure? A. Subxiphoid B. Morrison's pouch C. Perisplenic recess D. Paracolic gutter E. Pelvis

587 D. The 2 paracolic gutters are not included in the evaluation.

How long before wounds usually achieve their normal (pre-injury) tensile strength? A. 4 weeks B. 8 weeks C. 6 months D. 1 year E. Never

588 E. Wounds never achieve their normal tensile strength before injury.

Which of the following sutures is used for ligating bleeders? A. Silk B. Vicryl C. PDS D. Monocryl E. Chromic

589 A. For ligation of blood vessels, non absorbable sutures like silk or cotton should be used.

Most common site of perforation of the colon A. Cecum B. Ascending colon C. Transverse colon D. Descending colon E. Sigmoid colon

59 A. The cecum is the widest portion but has the thinnest wall; thus, it is the most common site of perforation. The sigmoid is the narrowest portion; thus, it is the most common site of obstruction.

A 50 year old farmer has a 10 year history of a 2x2 cm wound in the dorsum of his right foot. Biopsy revealed nest of tumor cells with keratin pearls. Definitive management would include? A. Below the knee amputation B. Excision with 1 cm margin of normal tissue and prophylactic inguinal lymph node dissection C. Excision with 0.5-1 cm tumor free margins plus radiation D. Excision with 0.5-1 cm tumor free margins with adjuvant radiation and chemothrapy E. Excision with 1 cm margin of normal tissue and radical lymph node dissection

590 B. For SCC, therapy is 1 cm tumor free margins. For high risk lesions such as chronic wounds, prophylactic lymph node dissection should be done

A 29 year old real estate agent noticed a lump in the upper outer quadrant of her right breast while taking a bath. You examined the patient and ordered an ultrasound of both breasts. Results showed BIRADS 2. The patient is reluctant to undergo surgery. What is the next step? A. Do core-needle biopsy as well as immunohistochemical stains (ER/PR/Her2neu) B. Excise the lesion and send to histopath C. Do local irradiation and chemotherapy D. Observation with short interval follow-up E. Reassure patient with routine screening

591 E. BIRADS 2 is a benign finding so you just need to reassure the patient with routine screening. BIRADS 3 would need follow-up. 4-5 may need to do biopsy.

A 32 year old female complains of an enlarging anterior neck mass that moves with degluttition. Patient is clinically euthyroid and has no other subjective complaints. Work-up revealed Hgb of 10.0 g/dL; WBC 5.5; Platelet 200. What is the next step in the algorhythm? A. Order TSH B. Do FNAB C. Do FNAB with Cervical UTZ D. Order Thyroid Scan E. Observe with close follow-up

592 A. Based on the algorhythm, no matter whether the patient is clinically euthyroid or not, a biochemical documentation is needed so one must order TSH first.

On subsequent work-ups of this same patient, biopsy revealed that it is follicular carcinoma measuring 4x4cm by ultrasound with metastasis noted to be present in both adrenals and lumbar vertebra resulting to a poorer prognosis. What is the Stage based on TNM? A. Stage 0 B. Stage I C. Stage II D. Stage III E. Stage IV

593 C. Age is the most important prognosticating factor in thyroid papillary and follicular carcinoma. Even with meatastasis, in a patient under 45 years old, it is still Stage II.

Which of the following is NOT a principle of anti- reflux surgery? A. Repair can be placed in the abdomen without undue tension. B. The fundoplication should not increase the resistance of the LES to a level greater than the peristaltic power of the esophageal body. C. Restore LES pressure to a level twice the resting gastric pressure and its length to at least 3 cm. D. Minimize placement of the LES in the positive pressure environment of the abdomen. E. Allow the reconstructed cardia to relax on deglutition

594 D. There should be placement of an adequate length in the positive pressure environment of the abdomen.

Appendectomy was done to a 20 year old college student for which histopathology showed a 0.5 cm focus of carcinoid tumor located in the base of the appendix with clear margins. As the attending surgeon, what is your next course of action? A. Re-explore the patient for evaluation of stump. B. Plan for an elective right hemicolectomy. C. Do chemotherapy. D. Order a abdominal CT scan and do a metastatic work-up E. Reassure patient.

595 E. Lesions of 0.5 cm of carcinoid tumor, the treatment is only appendectomy.

Which of the following criteria is NOT included in the Child-Pugh scoring of hepatic reserve? A. Hepatic encephalopathy B. Serum bilirubin C. Serum AST, ALT D. Serum albumin E. Prothrombin time

596 C. Presence of ascites and nutritional status are the other criteria included and not serum AST, ALT.

The presence of hernia are more common in males than in females especially hernias associated with a patent processus vaginalis. What is the most common hernia in the female population? A. Indirect Inguinal hernia B. Direct inguinal hernia C. Femoral hernia D. Obturator hernia E. Spigelian hernia

597 A. Indirect inguinal hernia still outnumbers femoral hernia in incidence even in the female population.

A 50 year old smoker has a 2x2 cm erythematous lesion in the lateral border of the tongue with a 1x2 cm palpable lymph node in the right neck area. What is the definitive management? A. Partial glossectomy with wide margins (>1 cm) B. Excision plus supraomohyoid dissection in the right C. Partial glossectomy and mandibulectomy plus unilateral supraomohyoid dissection D. Excision plus unilateral modified radical neck dissection E. Excision plus bilateral modified radical neck dissection

598 D. For lesions with palpable lymph nodes, one should do a formal unilateral MRND and not a prophylactic supraomohyoid dissection. Excision is the same as parital glossectomy.

In the ER, a 70 year old post-TURP male patient complain of difficulty in urination. What is the initial mangement? A. Do suprapubic catheterization B. Insert a foley catheter and drain all urine C. Insert a foley catheter and drain 500 cc every hour D. Give tamsulosin E. Refer to urologist for decompression and DJ stent insertion

599 C. Drain urine slowly to avoid hemorrhagic cystitis.

Patient is diagnosed with a superior sulcus tumor (a form of lung adenocarcinoma). Invasion of nearby structures results to which clinical findings? A. Hoarseness B. Back pain C. Ptosis, miosis, anhidrosis D. Shoulder pain, hiccups and dyspnea E. Conjunctival edema, swelling of the head and neck, headache

6 C. This is a case of pancoast syndrome. Invasion of the stellate sympathetic ganglion results to the classic Horner’s triad â€" ptosis, miosis, anhidrosis. SIMILAR TO PREVIOUS BOARD EXAM CONCEPT/PRINCIPLE Source: Schwartz’s Principles of Surgery 8th ed p 534

What is the most common indication for intubation? A. Facial trauma B. Hypotension C. Flail chest D. Pulmonary contusion E. Altered mental status

60 E.

In the immediate post-operative period after vitrectomy, which of the following findings is most commonly observed? A. Elevation of intraocular pressure B. Presence of subconjunctival hemorrhages C. Acceleration of cataract development D. Hyphema E. Retinal detachment

600 A. Increased IOP is common after vitrectomy.

During thyroidectomy, the surgeon accidentally ligated a nerve located along the right tracheoesophageal groove. Post-operatively, the patient is expected to experience: A. inability to reach high notes B. hoarseness C. stridor D. voice fatigue E. no symptom

601 B. The recurrent laryngeal nerve goes along the tracheoesophageal groove. Unilateral injury to the RLN results to hoarseness.

Pulmonary risk reduction in perioperative care is achieved through the following, EXCEPT: A. Smoking cessation B. Pre- and postoperative chest physiotherapy C. Laparoscopic approach (when possible) D. Appropriate pain control E. None of the above

602 E. Pulmonary risk reduction is achieved through smoking cessation, pre- and postoperative chest physiotherapy, laparoscopic approach (when possible), and appropriate pain control. Additionally, obese patients (body mass index [BMI] > 40 kg/m2) benefit from perioperative CPAP and intraoperative use of â†' positive end- expiratory pressure (PEEP; 10 cm H2O)

An 18-year old male was hit by a wooden paddle on his left eye during rowing practice in Pasig river. Blow out fracture will most likely be noted in: A. lamina papyracea B. lacrimal bone C. floor of the orbit D. zygomatic bone E. frontal bone

603 C. In trauma involving blunt objects, fracture is expected to occur in the area of least resistance. In the case of the orbit, it is the floor which offers least resistance.

Auscultation of a 25-year old female patient revealed mid-diastolic murmur with opening click at the apical area, grade 3/5. The patient has been experiencing worsening shortness of breath, orthopnea and occasional constricting chest pain for 6 months. She had a history of repeated episodes of untreated pharyngitis in the past. What is the most appropriate management for this patient? A. Percutaneous mitral valvuloplasty B. Valve replacement C. Penicillin G monthly for life D. VSD closure E. Observe

604 A. The patient has mitral valve stenosis most probably due to RHD. The management depends upon the severity of valve damage. Percutaneous valvuloplasty is the most appropriate initial management. If this fails, or if the valve is too damages, valve replacement should be done.

Following an MI, a 65-year old male patient had an acute heart failure, with an associated acute occurrence of holosystolic blowing murmur at the apical area. What is the mechanism? A. Reinfarction, compromising left circumflex coronary artery B. Ventricular wall rupture C. Ruptured mitral valve papillary muscle D. Ventricular septal wall rupture E. Pericarditis

605 C. The patient is described to be having an acute mitral regurgitation (as characterized by the murmur). The most common cause of new onset MR in post-MI patient is rupture of mitral valve papillary muscle due to necrosis.

A 53-year-old woman has a right-sided lung mass noted on CXR. She is a smoker with no cancer history. Chest CT confirms an irregular 4-cm mass and a right paratracheal node measuring 1.9 cm. Percutaneous biopsy of the lung mass confirms a bronchogenic adenocarcinoma. What must be done before choosing further treatment? A. PET scan B. MediastinoscopyLymph node biopsy C. Neo-adjuvant radiotherapy D. MRI angiography E. No further diagnostics needed

606 B. Mediastinoscopy and lymph node biopsy should be done to determine presence of metastatic lesions from other organs.

A 1 1/2 month-old baby girl was brought to your clinic because of stridor. There was no associated cough. The patient was delivered to a G9P8 37 year-old woman via NSD. Birthweight was 2.8kg with APGAR score of 8 and 9. She was sent home after 2 days with no perinatal complications. She had a good suck with normal swallowing, good cry and weight gain. Stridor is slightly relieved by putting her in a prone position. T=37.2 C, RR= 40/min. Your initial impression is: A. Congenital subglottic stenosis B. Congenital laryngeal web C. Laryngocoele D. Laryngomalacia E. Congenital cyst

607 D. Laryngomalacia is the most common cause of stridor among infants. The stridor is releived by having the baby lie on his/her tummy.

Epidural hematoma is characterized by: A. Convex hyperdensity in CT B. May cross midline but not suture lines C. Lucid interval in 20% of cases D. All are correct E. A and C only

608 D. Epidural hematoma involves tear in middle meningeal artery. CT will show lens-shaped (convex) hyperdensity. May cross midline but not suture lines. Lucid interval is noted to some patients. Management is emergent craniotomy and evacuation of hematoma.

A 25-year old male (BW = 55kg) wasimmediately brought to the ER just 10 minutes after sustaining full thickness burns in several parts of his body. Affected areas include anterior surfaces of bilateral arms and forearms, anterior and posterior right thigh, anterior and posterior right leg and his genitals. Using the Parkland formula, what should be the initial fluid resuscitation for this patient? A. 742 ml/hr for the first 8 hours B. 1017 ml/hr for the first 8 hours C. 770 ml/hr for the first 8 hours D. 990 ml/hr for the first 8 hours E. 687 ml/hr for the first 8 hours

609 C. Using the rule of 9s, the following is the equivalent computation of the TBSA affected: anterior surfaces of both arms - 9, ant and post thigh - 9, ant and post leg - 9, genitals 1. Using the parkland formula, the following: 4 ml/kg x 55 kg x 28 = 6, 160 mL for the first 8 hours, or 770ml/hr for 8 hours.

Which of the ff is not a component of the SIRS criteria? A. Temp of =/<36 C B. HR of =/>90 bpm C. RR =/> 24 D.WBC =/< 4,000/ul E. Bandemia

61 C. Source of the SIRS criteria wherein RR>20 is in Schwartz. If the question was asked in the IM boards, then the SIRS criteria is RR>24 accdg to Harrisons. I think it depends on the subject and the source book to be used . RR should be equal or more than 20 not 24. All the other parameters are correct.

After an uncomplicated abdominal surgery (appendectomy), the 35-year old patient is still on NPO during his 5th hospital stay due to inability to tolerate feeding. After how many hours/days should normal intestinal motility be observed in its earliest post-operatively? A. 12 hours B. 24 hours C. 48 hours D. 72 hours E. 4 days

610 B. Following most abdominal operations or injuries, the motility of the gastrointestinal tract is transiently impaired. Among the proposed mechanisms responsible for this dysmotility are surgical stress-induced sympathetic reflexes, inflammatory response-mediator release, and anesthetic/analgesic effects; each of which can inhibit intestinal motility. The return of normal motility generally follows a characteristic temporal sequence, with small-intestinal motility returning to normal within the first 24 hours after laparotomy and gastric and colonic motility returning to normal by 48 hours and 3 to 5 days, respectively. Resolution of ileus may be delayed in the presence of other factors capable of inciting ileus such as the presence of intra-abdominal abscesses or electrolyte abnormalities.

A 70 y/o male was seen in the ER because of severe abdominal pain of more than 24 hrs. History revealed chronic intake of NSAID due to osteoarthritis. He is tachycardic, tachypneic, febrile with a BP of 80/50. PE revealed board-like rigidity of the abdomen. What is the most appropriate treatment for this patient? A. Graham patching B. Vagotomy with antrectomy C. Vagotomy D. Partial gastrectomy E. A and B

611 A. Board like abdomen with a history of chronic intake of NSAIDs indicate that the patient may be having perforated peptic ulcer. Since the patient has a poor risk for surgery (70 years old, hypotensive), the most appropriate surgical procedure is just graham patching.

A 45-year old female was suffering from GERD for 10 years now, unrelieved by any medical treatment. What is the most appropriate option for this patient? A. Kocher's maneuver B. Gastric pull-up C. Fundoplication D. Esophagectomy with gastric pull-up E. Vagotomy

612 C. Fundoplication is the preferred surgical treatment for GERD.

A TRUE statement regarding chemical burns of the eye: A. Acid burns cause more extensive damage than alkali burns B. The first thing to do in cases of chemical burns is to check the visual acuity C. Neutralization should be achieved, so that an alkali burn can be treated by instilling an acidic substance and vice versa D. In copious irrigation of the eye, the fluid used need not be sterile, provided the chemical is diluted properly

613 D. Alkali burns cause more extensive damage than acid burns. Neutralization should never be attempted. Copious irrigation of the eye even with a nonsterile water should be the first thing to be done.

The most common pediatric malignant lesion in the liver is: A. Hepatocarcinoma B. Malignant teratoma C. Hepatoblastoma D. Metastatic lesion from neuroblastoma E. Hemangioma

614 C. Hepatoblastoma is the most common liver tumor among pediatric patients.

Which of the following statement is TRUE regarding normal hemostasis and platelet function? A. Vascular disruption is followed by vessel constriction mediated by vasoactive substances released by platelets. B. Platelets adhesion depends on the preformation of fibrin monomers. C. The endothelial surface supports platelet adhesion and thrombus formation D. A prolonged bleeding time may be due to thrombocytopenia or reduced amounts of von Willebrand factor E. None of the Above

615D.Vessel constriction is due to release of local endothelial factors such as adenosine as a result of endothelial damage. The vascular endothelial wall is normally remained smooth so as not to promote platelet adhesion and clot formation. Platelet adhesion occurs via von willebrand factors that are exposed on damaged endothelium. Platelet plug formation occurs first before formation of fibrin monomers. fibrinogen is needed in promoting platelet aggregation.

A 42-year old female underwent cholecystectomy. Sixteen hours after the operation, she developed fever. The most probable cause is: A. Pneumonia B. UTI C. Atelectasis D. Wound infection E. Thrombophlebitis

616 C. Cause of fever within 24 hours post-op is most likely due to atelectasis (wind). Followed by UTI (water), wound infection (wound), DVT (walk).

A 42-year old female sought consult because of a slow-growing, painless swelling in her right mandible. X-ray showed a multiloculated radiolucency in the body of the mandible with an impacted tooth clearly visualized. This condition is suggestive of: A. Torus mandibularis B. Ameloblastoma C. Gorlin’s cyst D. Osteosarcoma E. None of the above

617 B. The most common odontogenic tumor is ameloblastoma, presenting as multiloculated mass usually located in the body of the mandible.

TRUE statement/s about gallstone disease: A. All asymptomatics gallstones requires cholecystectomy to prevent complications B. Acute cholecystitis is associated with cystic duct obstruction usually due to gallstone impaction in the cystic duct or infundibulum C. Gallstone pancreatitis is caused by stones obstruction the second portion of the duodenum D. Choledocholithiasis occur more commonly than cholecystolithiasis E. All of the above

618 B. Asymptomatic patients with incidental finding of a gallstone will not require surgery unless porcelain gallbladder was also observed. Gallstone pancreatitis is caused by obstruction of a stone at the ampulla of vater, allowing backflow of pancreatic secretions. Choelcystolithiasis is far more common than choledocholithiasis.

Relative contraindications to lung surgery due to high morbidity include the following, EXCEPT: A. Preoperative FEV1 < 0.8 L B. Predicted postoperative FEV1 < 0.8 L C. PaCO2 > 45 mm Hg D. PaO2 < 50 mm Hg E. Maximum voluntary ventilation > 50%

619 E. Perform preoperative risk stratification for surgical candidates. Relative contraindications to lung surgery due to high morbidity include preoperative FEV1 < 0.8 L, predicted postoperative FEV1 < 0.8 L, PaCO2 > 45 mm Hg, PaO2 < 50 mm Hg, and maximum voluntary ventilation < 50%.

This interluekin promotes eosinophil proliferation and airway inflammation? A. IL-3 B. IL-5 C. IL-7 D. IL-9 E. IL-11

62 B. IL-5 is secreted by mast cells and basophils in order to promote eosinophil proliferation and airway infllammation

A 33-year old female was diagnosed with carbuncle on the volar surface of the right forearm. Which of the following group of axillary lymph nodes will first drain the lesion? A. Anterior B. Posterior C. Lateral D. Central E. Medial

620 C. Lateral (humeral) axillary lymph node drains the arm, posterior (scapular) group drains the upper back, anterior (pectoral) drains the breast. The central group of nodes receives lymph from these three groups. There is no medial group of axillary lymph nodes.

The appropriate duration of antibiotic therapy for most patients with bacterial peritonitis from perforated appendicitis is: A. 3-5 days B. 7-10 days C. 14-21 days D. >21 days E. 24 hrs

621 A. 3-5 days for perforated or gangrenous AP, 5-7 days for treatment for peritoneal soilage due to perforated viscus with moderate degrees of contamination, 7-14 days extensive peritoneal soilage or occuring in immunosuppressed host

Which of the following is NOT a risk factor for developing a surgical site infection? A. Radiation exposure B. Recent surgery C. Prolonged hospitalization D. Malnutrition E. Infancy

622 E.

A 20 year old young man presents with a left anterior 8th intercostal space stab wound. He is in no distress and a chest xray is normal. A diagnostic peritoneal lavage is performed and has a RBC count of 8,000/uL and a WBC count of 300/uL. Which of the following is the best treatment for this patient? A. Observation only B. CT Scan C. Laparoscopy or Thoracoscopy D. Exploratory laparotomy E. Any of the above choices

623 C. An RBC count of > 10,000/uL is considered positive finding and an indication for laparotomy; patients with DPL RBC count between 1000/uL and 10,000/uL should undergo laparoscopy or thoracoscopy (See schwartz 9th edition, pp 153-155)

Which of the following patients should be immediately reffered to a burn center? A. A 20 year old with 12 % partial thickness burn B. A 30 year old with major liver injury and a 15 % partial thickness burn C. A 2 % TBSA partial thickness burn to the anterior leg crossing the knee D. A 10 year old with a 7% partial thickness burn E.

624 A. All patients with a partial thickness burns of 10 % and above should be transferred to a burn center. A patient with a burn and another major TRAUMA can be treated and stabilized in the trauma center first.

The ideal time to administer prophylactic antibiotics to a patient undergoing a colon resection is: A. 8 hours before surgery with a dose repeated at the time of incision B. 2 hours prior to surgery with a dose repeated at the time of incision C. 1 hour before surgery D. at the time of incision E. 30 minutes after the incision

625 C. Antibiotic prophylaxis is most effective when adequate concentrations of antibiotic are present in the tissues at the time of incision and assurance of adequate preoperative antibiotic dosing, and timing has become a significant hospital performance measure. Addition of antibiotics after operative contamination has occured is clearly ineffective in preventing postoperative wound infections. (Schwartz 9th ed, pp 222)

Malignant cells are LEAST likely to be in which of the following stages of the cell cycle? A. S phase B. Go phase C. G1 phase D. M phase E. Any of the above

626 B. malignant cells are cells that do not enter the Go stage after proliferation.

A 35 year old woman with a BRAC1 gene mutation seeks your advice about her known increased risk of breast cancer. You should recommend? A. mammograms and physical examination every 6 months until she is 50, then bilateral prophylactic mastectomy B. mammograms and physical examination every 6 months + tamoxifen C. prophylactic bilateral mastectomy and, if she has completed childbearing , prophylactic bilateral oophorectomy D. None of the above

627 C.

The most common causative organism in malignant otitis externa is? A. Staphylococcus aureus B. Pseudomonas aeruginosa C. Streptococcus pneumonia D. Herpes simplex type 1 E. Proteus Vulgaris

628 B. swimmer's ear- Mc is Pseudomonas aeruginosa

You are about to remove the transverse colon due to an obstructing tumor. In order to prevent massive bloodloss, you should ligate the A. Ileo-colic artery B. Middle colic artery C. Ascending left colic artery D. B and C only E. All of the above

643 D.

Which of the following is an indication for tonsillectomy in children? A. > 5 infections B. > 3 infections with strong family history C. > 3 infections in one year D. > 1 week missed from school in a year due to tonsillar infections E. Any of the above

629 C. Tonsillectomy and adenoidectomy are indicated for chronic or recurrent acute infection and for obstructive hypertrophy. The american academy of otlaryngology -head and neck surgery suggests tonsillectomy after three or more infections per year despite adequate medical therapy.

This statement is true of Von Willberand's disease? A. It is the most common congenital bleeding disorder B. It is a pure quantitiative defect of the vWF C. Patients with this disorder have characteristic visceral and joint hemorrhages D. Menorrhagia is also common in men E. Patient also have prolonged PTT due to deficiency of F9

63 A. it is both a qualitative and quantitiave defect char by mucocutaneous bleeding. Menorrhagia is common in women. It is assoc with F8 deficinecy not F9.

A 65 year old who has smoked 2 packs a day for 45 years is found to have a 2cm solitary nodule 1 cm from the surface of the superior segment of the right lower lobe. The best initial diagnostic procedure is? A. Observation with biopsy if this increases size over 3-6 months B. Bronchoscopy C. Transthoracic Fine needle aspiration D. Open thoracotomy for excisional biopsy E. Any of the above choices

630 C. Transthoracic FNAB can accurately identify the status of the peripheral pulmonary lesions I up to 95 % of cases. It is the best initial diagnostic procedure. (Schwartz 9th ed pp 527-528)

The components that contribute to the function of Lower esophageal sphincter (LES) include all of the following EXCEPT. A. Length of intra-abdominal esophagus B. Width of the diphragmatic hiatus C. Resting pressure in the lower esophageal muscle D. length of the area of increased tone in the lower esophageal muscle. E. None of the above

631 B.

Which of the following preocedures for peptic ulcer disease has the highest incidence of post- operative diarrhea? A. Graham patch B. Parietal cell vagotomy C. Truncal vagotomy and pyloroplasty D. Distal gastrectomy without vagotomy E. Nissen Fundoplication

632 C.

The most common premalignant lesion in the stomach is? A. Adenoma B. Chronic gastric ulcer C. Atrophic gastritis D. Verrucous gastritis E. APUD

633 C.

A patient with an internal hemorrhoid that prolapses past the dentate line with straining is a ? A. 1st degree hemorrhoid B. 2nd degree hemorrhoid C. 3rd degree hemorrhoid D. 4th degree hemorrhoid E. None of the above

634 A. 1st degree hemorrhoid bulge into the anal canal and may prolapse beyond dentate line on straining.

A patient with a 1.5 cm carcinoid tumor of the mid appendix should undergo? A. Appendectomy only B. Partial cecectomy and lymph node sampling to confirm negative margins C. Resection of the cecum, terminal ileum, and adjacent mesentery (en bloc resection) D. Right hemicolectomy E. Observation only

635 A. because this tumor < 2 cm in size in the mid appendix, an appendectomy is an adequate treatment.

Standard therapy for a 4-cm right lobe hepatic adenoma is? A. Observation only B. Arterial embolism to prevent further growth C. Laparascopic ablation D. Surgical resection E. Liver transplantation

636 D. hepatic adenomas carry a significnt risk of spontaneous rupture with intraperitonal bleeding. It has also a risk of malignant transformation to a well differentiated HCC. Therefore, the usual recommendation is surgical resection.

Which of the following is an indication for cholecystectomy in an asymptomatic patient with an incidental finding of gallstones? A. Any history of abdominal pain B. Family history of complications of cholelthiasis C. Porcelain gallbladder D. Frequent travel out of the country E. Any of the above choices

637 C. porcelain gallbladder, a rare pre malignant condition in which the wall of the gallbladder becomes calcified, is an absolute indication for cholecystectomy.

A patient presents with biliary colic. On UTZ there are multiple small gallstones in the gallbladder and the common bile duct measures 9 mm in diameter. No stone is visualized in the common bile duct. Which of the following is the most reasonable next step? A. Repeat UTZ in 24-48 hours B. MRCP with contrast C. Percutaneous cholangiography D. Laparoscopic cholecystectomy and intraoperative cholangiography E. Observation only

638 D. Lap chole and IOC

The most common type of pancreatic cancer is? A. Ductal adenocarcinoma B. Adenosquamous carcinoma C. Acinar cell carcinoma D. Squamous cell carcinoma E. None of the above

639 A.

The ff statement regarding transfusion therapy is correct except? A. Rh negative blood should ideally be transfused with Rh negstive blood only. B. In emergency situations, type O negative blood may be transfused to all recipients. C. Cross matching shoild always be performed before the administration of dextran because it interferes with the typiong procedure D. Platelet preparations are capable of transmitting infectious infectious diseases and accounts for allergic disease as well. E. FFP is the usual source of the vitamin K dependent factors and is the only source of F8

64 E. FFP is the usual source of the vitamin K dependent factors and is the only source of F5 not F8. All the other statements are correct.

The most common location of the superior parathyroid gland is: A. Dorsal to the recurrent laryngeal nerve (RLN), within 1 cm of the junction of the RLN and inferior thyroid artery B. Ventral to the recurrent laryngeal nerve,within 1 cm of the junction of the RLN and inferior thyroid artery C. Dorsal to the RLN, within 3 cm of the junction of the RLN and inferior thyroid artery D. Ventral to the RLN, within 3 cm of the junction of the RLN and inferior thyroid artery E. None of the above

640 A. about 85 % of individual have 4 parathyroid glands that can be found within 1 cm of the junction of the inferior thyroid artery and the RLN.

The inferior parathyroid glands are derivatives of? A. 1st Branchial pouch B. 2nd Branchial pouch C. 2nd Pharyngeal pouch D. 3rd Branchial pouch E. 4th Branchial pouch

641 D. the Branchial pouch derivatives: 1. Ear, 2. Thymus, 3. Inf. PTG and Thyroid Gland, 4. Sup. PTG

A 34 year-old man was brought to ER due to motorcycle accident. Patient suffered from Cerebral hematoma and undergone surgery. On his 18th hour at the ICU, patient re-bleed and had central fever. You are considering medical management before re-opening his cranium. What should you give for the fever? A. Ibuprofen B. Paracetamol C. Acetaminophen D. Clonidine E. Naproxen

642 D. A central fever will never improve significantly with the use of ordinary anti-pyretics. A centrally-acting drug like Clonidine should be used.

High pressure in the hypopharynx may result to development of esophageal diverticula. Most commonly, diverticulas are found A. In the Killian's Triangle B. In the Laimer's Triangle C. At the level of Cricopharyngeus D. A and C E. B and C

644 D. this is Zenker's Diverticula, the MC esophageal diverticula and is commonly found in the Killian's triangle at the level of cricopharyngeus

Failure of migration of neural crest cells may result to A. Hirschprung's disease B. Albinism C. Achalasia D. A and C only E. All of the above

645 E. All choices given can be due to failure of migration of neural crest cells

A 24 year-old male patient came in due to right lower abdominal pain associated with lowgrade fever. (+) Rovsing's sign and leukocytosis was seen in CBC. Upon opening the patient's abdomen, appendix is not inflamed. Rather, you saw a small, firm, well-circumscribed yellow-brown tumor at at the tip of the appendix, measuring only 2.1cms. What is the best next step? A. remove the tumor and send for histopath exam B. remove the whole appendix C. do a right hemicolectomy D. leave the tumor and close the incision E. none of the above

646 C. this is a typical case of a Carcinoid tumor. And if its size is >2cms, you should do a hemicolectomy, regardless of the location (whether at the tip or base of the appendix)

Which of the following stetements is/are true regarding Gastrointestinal Stromal Tumor (GIST)? A. Arise from Interstitial cells of Cajal B. Mesenchymal in origin C. Prognosis depends on size and mitotic count D. A and C only E. All of the above

647 E.

All of the following clotting factors are synthesized in the liver, EXCEPT? A. Factor II B. Factor VII C. Factor VIII D. Factor IX E. Factor X

648 C. factor VIII is synthesized by active endothelial cells

Entamoeba histolytica is the most common cause of amoebic liver abscess. And it is usually located at the A. Superior-anterior aspect of the Right lobe of the liver B. Superior-anterior aspect of the Left lobe of the liver C. Inferior-anterior aspect of the Right lobe of the liver D. Inferior-anterior aspect of the Left lobe of the liver E. Posterior lobe of the liver

649 A.

What electrolyte abnormality occurs after a refeeding syndrome? A. Decreased serum sodium B. Increased serum potassium C. Decreased serum magnesium D. Increased serum phosphate E. Increased serum calcium

65 C. In refeeding syndrome, you expect decreased serum potassium, magnesium and phosphate especially when refeeding via TPN or NGT.

A 32 year old patient was brought to ER due to severe epigastric pain. Patient came from a party and ate chicharon bulakalak, pork barbeque, and consumed 9 bottles of beer. Upon PE, you saw ecchymosis along the patient's flank. The lesion is called A. Santorini's sign B. Grey Turner's sign C. Cullen's sign D. Rovsing's sign E. Ranson's sign

650 B.

Mondor's disease is a variant of thrombophlebitis in the chest involving the A. Lateral thoracic vein B. Left gastric vein C. Superficial epigastric vein D. A and C only E. All of the above

651 D. it is the thoracoepgastric vein and NOT the left gastric vein that is included

Batson's plexus is a venous drainage system that is a route for metastasis of A. Thyroid CA B. Breast CA C. Prostate CA D. B and C only E. All of the above

652 E.

Mucus retention cyst is just accumulation of submucosal mucus. It is best treated with A. Incision and drainage B. marsupialization C. Wedge excision D. curettage E. Steroids

653 B.

Which of the following statements is/are true regarding Juvenile Nasopharyngeal Angiofibroma? A. Highly expansile and destructive B. Typically occurs in adolescent males C. Angioembolization is the treatment of choice D. B and C only E. All of the above

654 E.

The cervical lymph nodes are divided to 6 different levels. Level II is further subdivided into IIa and IIb by the A. Spinal accessory nerve B. Anterior belly of omohyoid C. Posterior belly of omohyoid D. A and B E. A and C

655 A.

Most salivary gland tumors are benign. However, in terms of incidence of malignancy , it is more common in the A. Parotid gland B. Submandibular gland C. Sublingual gland D. A and B E. A and C

656 C. 25% Parotid, 50% submandibular, 75% Sublingual

During thyroidectomy, you should always look for the 5th blood supply of the thyroid which usually arises from? A. Brachiocephalic artery B. Innominate artery C. Aorta D. A or B E. All of the above

657 E. the thyroidea ima usually rises from innominate artery or the aorta. The innominate artery is AKA brachiocephalic artery

On a corrective surgery for PDA, the most commonly injured structure is the A. Thyroidea Ima B. Superior Thyroid artery C. Left recurrent laryngeal nerve D. Inferior thyroid artery E. None of the above

658 C. the Left RLN is hookson the arch of aorta as it goes up, hence commonly injured during PDA correction.

Parathyroid adenoma is best treated with A. Removal of the adenomatous gland B. 3.5 parathyroidectomy C. Total parathyroidectomy + autotransplantation D. B or C E. Radiation therapy

659 A. it is best to remove only the adenomatous gland because upto present, there is no better substitute for endogenous PTH.

How much percentage of blood loss is expected in a patient with normal blood pressure but with orthostatic hypotension? A. 10% B. 20% C. 30% D. 40% E. 50%

66 B. Pateint has Class II hemorrhage with approximately 15-30% blood loss or 750-1500 cc. Although cghoice C could be correct it is borderline Class II and III therefore Choice B is the best answer.

You had a 64 year old smoker patient whom you operated on due to salivary gland tumor. The histopath results showed highly aggressive malignant cells. What is your first consideration as your diagnosis? A. Adenoid cystic CA B. Acinic cell CA C. Pleomorphic Adenoma D. Mucoepidermoid CA E. Adenocystic CA

660 D. Musoepidermoid CA is the most common malignat tumor of salivary glands in adults.

A 45-year old male, chronic alcoholic presents to the ER with coffee-ground vomiting amounting to 2 cups and dark tarry stools. On physical examination, patient is afebule, BP 90/60 HR 110, RR 24, Positive ascites. What is the first step in management of this patient? A. Access airway, breathing and circulation B. Establish access with IV catheters C. Control bleeding D. do CBC, PT, PTT E. None of the above

661 A. In any patient first assess ABC's: Airway, breathing, Circulation

A 25-Year old female presented to the ER with RLQ pain, and fever. Patient also states that she had 2 episodes of vomiting, of nonbilous material 6 hours prior. On physical examination, patient is in moderate distress but with stable vital signs. Positive rebound tenderness at the RLQ, rectal exam was normal. CBC was done revealing leukocytosis with predominance of neutrophils, urinalysis was normal. What additional examination must be ordered? A. Ultrasound B. Blood culture C. Abdominal X-ray upright and supine D. Pregnancy Test E. None of the above

662 D. This is done to rule out ectopic pregnancy or PID.

Bleeding from esophageal varies originate from which blood vessel? A. Left gastric artery B. Left Gastric Vein C. Right Gastric Artery D. Right Gastric Vein E. None of the above

663 B. The left gastric vein (or coronary vein) carries blood low in oxygen, tributaries derived from both surfaces of the stomach; it runs from right to left along the lesser curvature of the stomach, between the two layers of the lesser omentum, to the esophageal opening of the stomach, where it receives some esophageal veins.

What is the first line diagnostic test for uncomplicated reflux diseases? A. Endoscopy B. 24-h pH test C. Manometry D. Trial of PPI E. None of the above

664 D. Trial of PPI is the only diagnostic test for reflux without dysphagia.

Where does the criminal nerve of Grassi originate? A. Posterior (Left) Vagus branch B. Posterior (Right) Vagus Branch C. Anterior (Left) Vagus Branch D. Anterior (Right) Vagus Branch E. None of the above

665 B. Left Anterior : Right Posterior

What type of vagotomy is described in the ff: Denervation of stomach (including pylorus) above crus of diaphragm, prescruing aliac and hepatic branches? A. Total B. Selective C. Highly Selective D. Proximal Gastric E. None of the above

666 B. TrunCal or total abdominal vagotomy. The main vagal trunks are divided, and surgery is accompanied by a drainage procedure, such as pyloroplasty . SeleCtive (total gastriC) vagotomy. The main vagal trunks are dissected to the point where the branch leading to the biliary tree divides, and there is a cut at the section of vagus close to the hepatic branch. This procedure is rarely indicated or performed. Highly seleCtive vagotomy (HSV). HSV selectively deprives the parietal cells of vagal nerves, and reduces their sensitivity to stimulation and the release of acid. It does not require a drainage procedure. The branches of Latarjet's nerve are divided from the esophagogastric junction to the crow's foot along the lesser curvature of the stomach. ThoraCosCopiC vagotomy. Performed through the third, sixth, and seventh left intercostal spaces, the posterior vagus trunk is isolated, clipped, and a segment excised.

What is the strongest tissue layer of the small intestine? A. Epithelium B. Mucosa C. Submucosa D. Muscularis Layer E. None of the above

667 C.

Most common cause of small bowel obstruction in adults? A. Hernia B. Tumors C. Constipation D. Adhesion E. None of the above

668 D.

This type of hernia is commonly foound in females and passes anterior to Cooper's ligament and posterior to ingunal ligament? A. Femoral Hernia B. Direct Inquinal Hernia C. Pantaloon Hernia D. Litre's Hernia E. None of the above

669 A. Femoral hernias occur just below the inguinal ligament, when abdominal contents pass through a naturally occurring weakness called the femoral canal.

All of the following rpactice guidelines are part of the Surviving Sepsis Campaign Bundle EXCEPT ? A. Measure lactate levels B. Obtain blood cultures prior to the admistration of antibiotics C. Administer 30cc/kg crystalloid for hypotension D. Maintain a mean arterial pressure of >60 mmHg E. Remeasure lactate levels if initial lactate was elevated

67 D. The goal MAP in the Surviving Spesis guidelines is >65 mmHg not 60mmHg

Time of blood supply to the liver? A. Portal vein provides 2/3 of blood to liver B. Hepatic Artery provides 2/3 of blood to liver C. Portal vein provides 70% of O2 supply to liver D. Hepatic Artery provides 70% of O2 supply to liver E. None of the above

670 A. Portal vein - 2/3 blood, 50% O2 Hepatic artery - 1/3 blood, 50% O2

Most common type of gall bladder stone? A. Cholesterol B. Pigment C. Black Stone D. Mixed Stones E. None of the above

671 D. Combination of both pigment and cholesterol stone

A 40 -year old female presents to the ER with fever, nausea, vomiting, constant RUQ pain with a history of bilary colic. Physical Examination shows inspiratory arrest on palpation of RUQ. Diagnosis is acute cholecystitis. When is cholecystectomy indicated? A. Cholecystectomy if 1 week from onset of symptoms B. Within 72 Hours if without complications C. Wait 4-6 weeks if with complication D. if less than 72 hours, wait 5 days from resolution of inflammation E. None of the above

672 B. Anytime if with complications. If , 72 hours from onset, proceed with surgery. If more that 72 hours, wait for 4 - 6 weeks for inflammation to subside then operate

Possible comlications if acute pancreatitis? A. Necrosis B. Abscess C. Hemorrhage D. AOTA E. None of the above

673 D. Locoregional complications include pancreatic pseudocyst (Most common, occurring in up to 25% of all cases) and phlegmon / abscess formation, splenic artery pseudoaneurysms, hemorrhage from erosions into splenic artery and vein, thrombosis of the splenic vein, superior mesenteric vein and portal veins (in descending order of frequency), duodenal obstruction, common bile duct obstruction, progression to chronic pancreatitis

What is the preferred crystalloid in large volume rescusutation? A. PNSS B. D5W C. Plain LR D. D50/50 W E. None of the above

674 C. Lactated Ringer’s is a sterile solution that is used to replace fluid lost by the body. It is commonly used for fluid resuscitation, meaning that the patient needs aggressive fluid replacement for their injury or illness. Lactated Ringer’s looks like water, but it contains additives including calcium, potassium, lactate, sodium and chloride.

What percentage of TBW is found in the ECF? A. 10% B. 20% C. 30% D. 40% E. 15%

675 B. 60-40-20 rule

First sign / manifestation of hypovolemic shock? A. Hypotension B. Tachypnea C. Tachycardia D. Oliguria E. Orthostatic Hypotension

676 C. Direct loss of effective circulating blood volume leading to: A rapid, weak, thready pulse due to decreased blood flow combined with tachycardia

What are the surgical indications for hemothorax? A. Massive hemothorax >= 1000mL B. >150mL/hr from chest tube for 2hrs. C. BT needed for hemodynamic stability D. A & B E. All of the above

677 E. A hemothorax is managed by removing the source of bleeding and by draining the blood already in the thoracic cavity. Blood in the cavity can be removed by inserting a drain (chest tube) in a procedure called a tube thoracostomy. Usually the lung will expand and the bleeding will stop after a chest tube is inserted.

Patient presented with respiratory distress, hypotension and distended neck veins and muffled heart sounds. ECG was done revealing low voltage electrical alternans, What would be the 1st step in management of this patient? A. Pericardiocentesis B. Surgery to ID source of bleeding C. Emergency thoracostomy D. Fluids to raise CVP > Intrapericardial pressures E. None of the above

678 D. This is a case of cardiac tamponade.

The following are short term complications of electrical burns except:? A. Amythmia B. Acute tubular necrosis C. Compartment syndrome D. Cataracts E. None of the above

679 D. Cataract formation is part of the long term complications of burns.

What antiobitic is recommended as prophylactic use for a laparoscopic low risk biliary tract surgery A. Cefazolin B. Cefazolin + Metronidazole C. Cefoxitin D. Ampicilin-sulbactam E. None of the above

68 E. Accdg to Schwartz's 10 ed pg 146 Table 6-5: No antibiotic is needed for a low risk laparasocopic biliary tract surgery.

The three levels of lymphatic drainage of the breast are numbered in relation to what structure? A. Pectoralis Major B. Pectoralis Minor C. Serratus Anterior D. Deltoid Muscle E. Biceps Brachii

680 B. Pectoralis minor divides the levels of LN that drains the breast.

A 24 yo man presents with a 1-2 day history of right eye redness, purulent discharge, mild blurring of vision and mild pain. PE reveals a visual acuity of 20/25 right and 20/20 left, right conjunctival edema, injection, a clear cornea, and copious creamy purulent discharge. Which of the following would be inappropriate in the management? A. Obtain ophthalmologic consultation B. Obtain thorough sexual history C. Initiate systemic antibiotic treatment D. Give topical antibiotics with OPD follow-up

681 D. Answer: D. Give topiCal antibiotiCs with OPD follow-up Notes: Choice D is inappropriate. Condition is most likely Gonococcal.

Mr. Bugoy Tigas, 20 yo male, is stabbed in the left side of his chest, medial to the nipple. Upon examination, his blood pressure is 90/60 mmHg and his pulse is 130/min. His jugular venous pulse increases on inspiration, whereas his peripheral pulse and blood pressure decreased on inspiration. Breath sounds are normal bilaterally. The patient’s chest x-ray film is unremarkable. After receiving 2 L of isotonic saline, his blood pressure remains low, whereas his central venous pressure rises to 32 cmH2O. Which of the following is the most appropriate next step in the most appropriate next step in the management of this patient? A. Insert a chest tube into the left pleural cavity B. Increase parenteral fluids until the blood pressure increases. C. Order and Echocardiogram D. Decrease venous pressure by administering a venous dilator

682 C. Answer: C. Order an eChoCardiogram Notes: Cardiac tamponade is characterized by decreased cardiac output and increased central venous pressure owing to restriction of blood flow into and out of the heart as fluid in the pericardial sac restricts filling of all the cardiac chambers. An echocardiogram is the most sensitive and specific noninvasive test to establish the presence of fluid in the pericardial sac.

A grossly underweight 52 yo woman with chronic malnutrition is due to undergo major surgery. It is decided to start total parenteral nutrition (TPN) as part of the initial therapy. While introducing a central venous catheter into the right subclavian vein, the patient develops sudden dyspnea. Which of the following is the most likely diagnosis? A. Pulmonary embolism B. Fat embolism C. Acute anxiety D. Pneumothorax

683 D. Answer: D. Pneumothorax Notes: Pneumothorax can inadvertently occur while introducing a subclavian venous catheter. The patient will complain of sudden dyspnea. A chest x-ray film should always be taken after subclavian catheter placement, to determine the position of the tip of the catheter and to exclude the possibility of pneumothorax.

A 60 yo alcoholic male presents with severe chest pain after repeated vomiting. A chest X-ray shows a small left pleural effusion. The next step in management is: A. Obtain cardiac enzymes and admit to coronary care unit B. Insert nasogastric tube, administer intravenous fluids, and observe C. Perform upper endoscopy D. Administer Gastrografin swallow test

684 D. Answer: D. Administer Gastrografin swallow test

Three months after accidental ingestion of muriatic acid, a 30 yo female consults for dysphagia. Barium swallow shows a 10cm strictured segment of distal thoracic esophagus and normal-looking stomach. What is the preferred treatment for this patient? A. Esophageal dilatation B. Gastric pull-up C. Esophagectomy with colonic interposition D. Esophagectomy with gastric pull-up

685 D. Answer: D. EsophageCtomy with gastriC pull- up

The criteria for diagnosis of Primary Sclerosing Cholangitis are the following; except, A. Presence of choledocholithiasis B. No evidence of primary liver disease C. Absence of history of congenital anomalies D. Stenosis of a major portion of biliary duct system

686 A. Answer: A. PresenCe of CholedoCholithiasis

Statement/s true about metabolic changes in injured patients; A. There is an obligatory decrease in energy expenditure and nitrogen excretion B. There is a rapid depletion of labile and functional energy stores C. There is a positive nitrogen balance D. All of these

687 B. Answer: B. There is a rapid depletion of labile and funCtional stores.

The most common organism associated with OPSI is: A. Streptococcus pneumoniae B. Pseudomonas aeruginosa C. Hemophilus influenzae D. Neisseria meningitides

688 A. Answer: A. StreptoCoCCus pneumoniae Notes: OPSI â€" Overwhelming Post-Splenectomy Infection

Definitive treatment of a patient with sclerosing cholangitis and biliary cirrhosis involves which of the following? A. Ursodeoxychlic acid B. Corticosteroids C. Endoscopic balloon dilation & stenting D. Hepatic transplantation

697 D. Answer: D Once sclerosing cholangitis has progressed to cirrhosis, the only definitive treatment is hepatic transplant.

A 68 yo man presents at the ED because of such severe abdominal pain. He tells the triage nurse that he hadn’t been feeling well for the past couple of months, primarily because he had been having abdominal pain about 30 minutes after eating and as a consequence lost almost 10 pounds, but last night he suddenly developed severe stomach ache. He also has been vomiting and has had several episodes of bloody diarrhea. Upon PE, the physician notes hypotension and confirms the abdominal pain and notices abdominal distention. However, bowel sounds are absent, and there is no rebound tenderness present or other relevant findings upon abdominal examination. Laboratory data reveal an absolute neutrophilic leukocytosis and left shift plus lactic acidosis, and elevation of the serum amylase level. Which of the following is the most likely diagnosis? A. Acute ulcerative colitis B. Hemorrhagic pancreatitis C. Aortoenteric fistula D. Acute small bowel infarction

689 D. Answer: D. ACute small bowel infarCtion Notes: Acute small bowel infarction is indicated by the sudden onset of severe abdominal pain with vomiting and abdominal distention out of proportion with the physical findings, absent bowel sounds, a striking neutrophilic leukocytosis with left shift, lactic acidosis, hypotension, and increased serum amylase concentration of bowel origin.

An extensive introduction of bacteria into a normally sterile are of the body due to major breaks in the sterile technique is identified as what wound classification? A. Clean B. Clean/Contaminated C. Contaminated D. Dirty E. Somewhat dirty

69 C. Contaminated or Class 3 wounds includes open accidental wound encountered early after injury, those with extensive extensive introduction of bacteria into a normally sterile are of the body due to major breaks in the sterile technique penetrating abdominal trauma, large tissue injury and enterotomy during bowel obstruction

Ten years ago, a 79 yo female with a 25-year history of type 2 diabetes was diagnosed with diabetic nephropathy. At this time, she is overweight but not obese and she smokes about three packs of cigarettes per week. Despite being diagnosed with diabetic nephropathy, she had not stopped smoking and in general had not tightly controlled her blood glucose levels; consequently, her renal function has steadily deteriorated and her physician now believes she has ESRD. He arranges for her to undergo hemodialysis. Before undergoing hemodialysis, it was also recommended that she have an operation. Which of the following choices describes the surgical procedure most likely recommended? A. A renal biopsy B. Insertion of a special, soft catheter through a small slit made adjacent to the navel under local anesthesia C. Performing open surgery and inserting a catheter under general anesthetic D. Formation of an arteriovenous (AV) fistula

690 D. Answer: D. Formation of an arteriovenous (AV) fistula Notes: To perform hemodialysis, it is necessary to surgically form an arteriovenous (AV) fistula. This is accomplished by creating an opening in which the artery is sewn on to a vein; this is usually done in the arm. In hemodialysis, blood is withdrawn from an artery and returned via a vein.

A 65 yr-old man w/ a history of chronic alcohol abuse has been experiencing epigastric & periumbilical pain asso. w/ nonbilious vomiting for 1 day. He denies any melena or hematemesis. In the past he has had several episodes of similar pain that sometimes radiated to the back and was hospitalized for several days 2 months prior. No previous surgery or medical problems.P/E: BP 120/80, HR 110 bpm, dry mucous membranes. Abdomen not distended, no surgical scars. Diminished bowel sounds. Soft abdomen, exhibits voluntary guarding of epigastrium. Serum amylase level is 550 units/100 ml. What is the most reasonable initial step in management of this patient? A. Establish intravenous access B. Conduct sonographic studies to demonstrate cholelithiasis C. Perform CT scanning to diagnose a pancreatic pseudocyst D. Perform esophagogastroduodenoscopy (EGD) to evaluate for varices and complications of cirrhosis E. Initiate a low-fat diet and antilipid treatment.

691 A. Answer: A A px w/ pancreatitis can have severe abdominal pain & rigidity. Surgery should be avoided except for complications (necrotizing panc.or symptomatic endocyst). Initial management: bowel rest, IV resuscitation, parenteral nutrition & monitoring in ICU when appropriate.

The syndrome of multi-organ system failure (MOF): A. Involves sequential insults that lead to systemic hyperinflammation B. Requires the documentation of active infection C. Has decreased in incidence over the past decade D. Requires diagnosis within 3 days of the systemic insult E. Demonstrates consistent improvement after blood transfusion

692 A. Answer: A The “two-event†model of multi-organ system failure involves an initial insult that results in a primed inflammatory response; can develop w/out overt infection;bimodal-within 3 days of initial insult or 6-8 days after insult.

One of the following is a true diverticulum: A. Zenker B. Parabronchial C. Meckel D. Pharyngoesophageal

693 B. Answer: B Parabronchial (midesophageal diverticulum) is a true diverticulum caused by traction of inflamed mediastinal nodes.

A 35 yr-old woman complains of dysphagia, regurgitation and weight loss. Esophagography shows narrowing of the distal end of esophagus and manometry studies show significant tertiary waveforms. The LES has high residual pressure on swallowing. Which of the following has not been implicated as a possible cause of her disease? A. H.pylori infection B. Severe emotional stress C. A parasitic infection D. Drastic weight reduction E. Degeneration of Auerbach plexus

694 A. Answer: A The cause of achalasia is idiopathic; however, severe emotional stress, T.cruzi infection causing destruction of Auerbach plexus & drastic wt.loss have been implicated.

All but one meet the criteria for referral to burn center: A. A 50 yr.old w/ a 1% TBSA partial thickness burn on her left hand from a cooking accident B. A 30 yr-old construction worker with pain & blistering bilaterally on the knees after kneeling in wet cement whole afternoon C. A 25 yr-old man w/ 7 % TBSA partial thickness burns on the chest. D. 18 yr-old man in motor vehicle collision w/ 30% TBSA burns on chest and circumferential burns on arms

695 C. Answer: C *Review criteria for referral to burn center PT burns > 10% TBSA; chemical burns..etc.

Refeeding syndrome is characterized by which of the following electrolyte abnormalities? A. Hyponatremia, hypokalemia,hypercalcemia B. Hyperphosphatemi, hypokalemia, hypocalcemia C. Hypokalemia, hypomagnesemia, hypophosphatemia D. Hypocalcemia, hypernatremia , hypomagnesemia

696 C. Answer: C Refeeding syndrome results when glucose is administered quickly to an individual w/ poor nutrient intake before TPN. Subsequent rapid serum depletion, magnesium, phosphorus or potassium develops.

It is the single most important test in the evaluation of thyroid masses and is the first diagnostic step of a solitary thyroid nodule according to the Bethesda criteria? A. TSH B. Thyroid UTZ C. FNAB D. Thyroid scan E. fT4 and fT3

79 C. Please refer to the figure in Schwartz's 10 ed pg 1539 Figure 38-14.

A 60 yr-old alcoholic man has a 24-hr history of nausea and vomiting, abdominal pain, distention and decreased passage of stool and flatus. He underwent abdominoperineal resection of the rectum for cancer 18 months earlier, along w/ post-op irradiation and chemo. P/E reveals distended, diffusely tender, tympanitic abdomen. Which of the following is the least likely diagnosis? A. pancreatitis with ileus B. adhesive bowel obstruction C. bowel obstruction caused by extrinsic compression D. alcoholic hepatitis with ascites

698 D. Answer: D Although abdominal distention maybe because of ascites, its onset is much more gradual than 24 hrs. And it is usually painless.

Radiation delivered to the breast after right lumpectomy and sentinel lymph node biopsy for a 1.2 cm node-neg infiltration ductal carcinomais likely to be asso. with which of the following? A. Decreased risk for systemic recurrence B. Can be used in lieu of chemotherapy in early stage breast cancers. C. Increased risk for lymphoma D. Decreased risk for local recurrence

699 D. Answer: D Radiation for local control; chemo for systemic.

A fistula with an external opening located anteriorly 2.75cm from the anal margin will have: A. Short, radial tract to the anterior midline B. Curvilinear tract to the posterior midline C. Curvilenear tract to the anterior midline D. A and C E. All of the above

7 A. Goodsall’s rule dictates that fistulas with an external opening anteriorly connect to the internal opening by a short, radial tract. Fistulas with an external opening posteriorly track in a curvilinear fashion to the posterior midline. However, if the anterior external opening is greater than 3cm from the anal margin, such fistulas tract to the posterior midline. SIMILAR TO PREVIOUS BOARD EXAM CONCEPT/PRINCIPLE Source: Schwartz’s Principles of Surgery 8th ed p 1064

In relation to the previous question, what is the expected infection rate? A. 1.0-2.0% B. 2.1-9.5% C. 3.4-13.2% D. 3.1-12.8% E. None of the above

70 C. Accdg to Schwartz's 10 ed pg 148 Table 6-7; Choice A- Class 1, Choice B- Class 2 and so forth.

Based on the classification of the American College of Surgeons, the classical features of hypovolemic shock is first noted in: A. Class I B. Class II C. Class III D. Class IV

700 C. Answer: C Class I â€"orthostatic hypotension; Class II â€" tachycardia; Class III â€" hypotension; Class IV â€" profound shock

All of the ff are criteria for referral to a burn center EXCEPT? A. Partial thickness burns greater than 20% TBSA B. Burns involving the genitalia C. Electrical burns D. Chemical burns E. Burns involving the hands

71 A. Accdg to Schwartz's 10 ed pg 228 Table 8-1; A partial thickness burns of greater than 10% TBSa should already be referred to a burn center. Burns involving the face, hands, feet, genitalia, perineum or major joints are also referred to a burn center.

This is true of Basal Cell Carcinoma? A. Arises from the basal layer of keratinocytes B. It accounts for half of all diagnosed skin cancers C. The natural history is BCC is distant metastasis D. The most common form of BCC is the superficial form accounting for 60% of all forms E. Bleeding, ulceration and itching are often part of the clinical presentation.

72 E. BCC arises from the basal layer of non keratinocytes. IT accounts for 75% of all skin cancers. The natural history is local invasion not distant mets. The msot common form is the nodular form.

It is the treatment of choice for morpheaform, poorly delinated, recurrent and infiltrative BCC, particularly facial lesions. A. Excisional surgery B. Incisional surgery C. Wide local excision D. Cautererization E. Moh's microsurgery

73 E. Moh's microsurgery provides histologic confirmation of excision and maximal conservation of tissue, which is important to keep in mind in cosmetically senstivie areas such as the face.

Among the ff breast conditions, which has the highest relative risk in terms of breast CA? A. Florid hyperplasia B. Sclerosing adenosis C. Intraductal papilloma D. Atypical ductal hyperplasia E. Lobular carcinoma in situ

74 E. Florid hyperplasia- 1.5-2 fold Sclerosing adenosis- No risk Intraductal papilloma- No risk Atypical ductal hyperplasia- 4-fold Lobular carcinoma in situ- 10-fold

Nasopharngeal carcinoma will likely spread to what level of neck lymph nodes? A. Level 2 B. Level 3 C. Level 4 D. Level 5 E. Level 6

75 D. NPCA will classically spread to the Level 5 or posterior triangle lymph nodes.

All of the ff are true regardign the principles of anti-reflux surgery EXCEPT? A. Following a fundoplication, the expected increase is to a level twice the resting pressure B. The operation should place an adequate length of the distal esophageal sphincter of about 2 or more cm in the stomach. C. The operation should allow the recontructed cardia to relax on deglutition for about 10 secs D. The operation should increase the resistance of the relaxed sphincter to a level that exceeds the peristaltic power of the body of the esophagus. E. The operation should ensure that the fundoplication can be placed in the abdomen without undue tnesion and maintained there by approximation the crura of the diaphragm above the repair.

76 D. The operation should NOT increase the resistance of the relaxed sphincter to a level that exceeds the peristaltic power of the body of the esophagus. All other statements are true.

It is known as the upward sliding of the gastric fundus alongside a normally positioned cardia? A. Hiatal hernia type I B. Hiatal hernia type II C. Hiatal hernia type III D. Hiatal hernia type IV E. Sliding hernia

77 B. Type I/Sliding hernia- upward dislocation of the cardia; Type 2/Rolling hernia- upward dislcoation of the fundus; Type 3- upward dislocation of both cardia and fundus; Type 4- in addition to above, an addtl organ such as the colon is dislocated.

Herniation involving a Meckel's diverticulum is also known as? A. Littre's hernia B. Spigelian hernia C. Petersen's hernia D. Pantaloon's hernia E. Richter's hernia

78 A. Spigelian- hernia through the semilunaris; Petersen's- internal henriation of the small bwel through the mesenteric defect from the Roux limb; Pantaloon's- hernia sac exists as both direct and indirect hernia straddling the inferior epigastric vessels; Richter's- incarcerated or strnagulated hernia involving only one sidewall of the bowel

The Poupart’s ligament is derived from which of the following structures? A. External oblique B. Internal oblique C. Trasversus abdominis D. Transversalis fascia E. Peritoneum

8 A. The poupart’s ligament/inguinal ligament forms the inferior margin of the inguinal canal. Inside courses the spermatic cord. Source: Schwartz’s Principles of Surgery 8th ed p 1308

All fo the ff statements are true regarding epidural hematoma EXCEPT? A. Epidural hematomas usually appear convex in shape because their expansion stops at the skull's sutures, B. May present with a lucid period immediately following the trauma and a delay before symptoms become evident C. It is an extra-axial bleed D. Associated with tearing of the bridging veins in the dural venous sinuses E. The most common cause of intracranial epidural hematoma is traumatic

80 D. Choice D refers to subdural hematoma. In epidural hematoma there is tearign of the middle mengineal artery instead

Which of the following giant diaphragmatic hernias refers to an upward dislocation of the gastric fundus alongside a normally positioned cardia? a. type II rolling b. type I sliding c. type III sliding-rolling d. type IV e. NOTA

81 A. Schwartz Principles of SUrgery 9th edition p. 842

2. A 16 year old male stowaway came in for persistent feculent discharge over his underwear. Upon rectal examination, an indurated tract is palpable. Which of the following is TRUE? a. an anterior opening 4cm from the anal margin will have an internal opening through a short radial tract b. goal of treatment is eradication of sepsis c. a posterior opening will track in a long radial route in posterior midline d. intersphincteric fistulas usually result from an ischiorectal abscess and extends to both internal and external sphincters e. Extrasphincteric fistulas are the most common

82 B. fistulas with an external opening anteriorly connect to the internal opening by a short radial tract. Fistulas with an external opening posteriorly tract in a curvilinear fashion to the posterior midline. D - transphinteric fistula; E - extrasphincteric fistula are rare and treatment depends upon both anatomy of fistula and its etiology Schwartz Principles of SUrgery 9th edition p. 1064

3. Which of the following techniques for breast surgery will leave most of the breast skin that will create a pocket to be filled with implants or with patient’s own tissue from another part of her body? a. skin sparing mastectomy b. simple mastectomy c. modified radical mastectomy d. total mastectomy e. radical mastectomy

83 A. http://www.breastpreservationfoundation.org/ nipple-sparing-mastectomy

4. AJ Perez, a young actor, died in a MVA in Moncada, Tarlac. He donated his cornea to a 28 year old and 8 year old. He is the famous face on the Eye Bank of the Philippines posters. Which of the following refers to corneal transplantation wherein the endothelial cells and descemet’s membrane are left in place and the anterior layers of the cornea are removed and replaced with donor tissue? a. penetrating keratoplasty b. superficial anterior lamellar keratoplasty c. deep anterior lamellar keratoplasty d. endothelial keratoplasty e. NOTA

84 C. When the entire cornea is replaced it is known as penetrating keratoplasty and when only part of the cornea is replaced it is known as lamellar keratoplasty. Keratoplasty simply means surgery to the cornea.Endothelial keratoplasty replaces the patient's endothelium with a transplanted disc of posterior stroma/Descemets/endothelium (DSEK) or Descemets/endothelium (DMEK). https://en.wikipedia.org/wiki/Corneal_transpla ntation

5. which of the following refers to an esophageal hiatal hernia characterized by an upward dislocation of the cardia in the posterior mediastinum? a. type II rolling b. type I sliding c. type III sliding-rolling d. type IV e. NOTA

85 B. Schwartz Principles of Surgery 9th edition p. 842

6. Which of the following bones of the orbital wall will be involved in a blowout fracture? a. frontal process of the maxilla b. zygomatic bone c. orbital plate of the maxilla d. orbital process of palatine bone e. ethmoid bone

86 C. Vaughan and Asbury’s General Ophthalmology 17th edition p.1

7. Which of the following maxillary fractures refers to a fracture line which passes through the nasal and lacrimal bones in addition to the maxilla forming the medial orbital floor? a. type I Le Fort b. type II Le Fort c. type III Le Fort d. type IV Le Fort e. type V Le Fort

87B. Type I - below the orbital floor type III - involves the medial and lateral walls and the orbital floor in the presence of separation of the facial skeleton Vaughan and Asbury’s General Ophthalmology 17th edition p.375

8. Which of the following refers to a delayed hypersensitivity reaction to microbial antigens and consists of a perivasculitis with lymphocytic cuffing of a vessel? a. ligneous conjunctivitis b. granulomas c. phlyctenules d. pseudomembrane e. chalazion

88 C. A - recurring membranous conjunctivitis B - most commonly are chalazia D - results from exudative process (coagulum on the surface of the epithelium and when removed, epithelium remains intact) Vaughan and Asbury’s General Ophthalmology 17th edition p.101

9. A 4 year old male came in for proptosis, hyperemia, edema, chemosis, limitation of eye movement and blurring of vision. Patient had a history of recurrent watery nasal discharges and nasal congestion for the past 3 months. Which of the following is the most common cause of proptosis in children? a. preseptal cellulitis b. orbital cellulitis c. rhabdomyosarcoma d. capillary hemangioma e. meningocele

89 B. Vaughan and Asbury’s General Ophthalmology 17th edition p.254

16. A 34 year old drug lord was stabbed in the left flank area by an unknown assailant. Which of the following are ABSOLUTE indications for surgical intervention for renal trauma except? a. renal pedicle avulsion b. expanding pulsatile or uncontained retroperitoneal hematoma c. coexisting bowel injuries d. persistent life threatening hemorrhage e. none of the above

96 C. -other relative indications: large laceration of renal pelvis or avulsion of UPJ, persistent urinary leakage, abnormal intraoperative one shot IV urogram, devitalized parenchymal segment with associated urine leak, complete renal artery thrombosis of both kidneys, renal vascular injuries after failed angiographic management, renovascular hypertension Schwartz Principles of Surgery 9th edition

A 26 year victim of a vehicular accident is brought on a stretcher in the ER. On physical examination his blood pressure is 70/40 and heart rate is 65. The extremities are warm and he is noted to have decreased sensorium. Which of the following is the appropriate initial management? A. Fluid resuscitation B. Blood transfusion C. Dopamine/Norepinephrine D. Dobutamine/Digoxin E. Epinephrine/Antihistamine/ Corticosteroids

9 A. This is a case of neurogenic shock. The classic description of neurogenic shock consists of decreased BP associated with bradycardia (absence of reflexive tachycardia due to disrupted sympathetic discharege), warm extremities (loss of peripheral vasoconstriction), motor and sensory deficits indicative of a spinal cord injury, and radiographic evidence of a vertebral column fracture. After the airway is secured and ventilation is adequate, fluid resuscitation and restoration of intravascular volume often will improve perfusion in neurogenic shock. Most patients with neurogenic shock will respond to restoration of intravascular volume alone, with satisfactory improvement in perfusion and resolution of hypotension. Administration of vasoconstrictors will improve peripheral vascular tone, decrease vascular capacitance, and increase venous return, but should only be considered once hypovolemia is excluded as the cause of the hypotension, and the diagnosis of neurogenic shock established. If the patient's blood pressure has not responded to what is felt to be adequate volume resuscitation, dopamine may be used first. **Note: Be careful of doing fluid resuscitation in patients with decreased cardiac function. Always ask for history of MI and auscultate for crackles. Source: Schwartz’s Principles of Surgery 8th ed p 108

10. Which of the following diaphragmatic hernias refers to the herniation of an additional organ such as the colon? a. type II rolling b. type I sliding c. type III sliding-rolling d. type IV e. NOTA

90 D. Schwartz Principles of Surgery 9th edition p. 842

11. JCR 24 years old medical student frequently complaints of throbbing headache which is immediately relieved with paracetamol. Initially he thought it was just tension headache. After taking the bimonthly examination, he went to the ER now with double vision and stiff neck which he attributed to lack of sleep due to excessive studying. On physical examination, JCR was wide awake and nuchal rigidity was noted. Which of the following is NOT true of this case? a. JCR should have a CT scan of the head immediately b. Hunt -Hess clinical grading would be grade 3 c. negative CT and LP rules out SAH d. 4 vessel angiography is the gold standard for diagnosis e. if LP is performed, xanthochromia with high RBC count which do not decrease in the tubes would be present

91 B. it is grade 2. grade 3 would be lethargic with mild focal neurologic deficit (pronator drift) Schwartz Principles of Surgery 9th edition

12. RP a movie actor was involved in a motor vehicular accident in Mandaue City which caused a traffic standstill. RP sustained crushing injuries over his tibia and fibula and upon examination, he is unable to evert his foot. Based on the Seddon classification, which of the following peripheral nerve injury patterns is present when there is disruption of axons and endoneurial tubes? a. wallerian degeneration b. neurapraxia c. axonotmesis d. neurotmesis e. none of the above

92 D. B - temporary failure of nerve function without physical axonal disruption. axonal degeneration does not occur. C - disruption of axons and myelin. endoneurium are intact. axons degenerate proximally and distally from area of injury. A - distal degeneration D - peripheral collagenous components such as perineurium may or may not be intact. Schwartz Principles of Surgery 9th edition

13. In the above scenario, RP’s right foot was noted to be cool, pale, with absent pulses, and a hematoma over the fracture site. Which of the following is NOT true in the management? a. this would require operative exploration as absent pulses is a hard sign b. due to the presence of nerve injury, observation is only warranted c. the fractures should be realigned before definitive vascular examination d. significant hematoma and peroneal nerve injury are soft signs e. acute ischemia and pulsatile hemorrhage are also hard signs

93 B. due to presence of hard signs (absent pulses), operative exploration is warranted. Other soft signs include proximity to vasculature, A-A index of <0.9, thrill or bruit Schwartz Principles of Surgery 9th edition

14. Due to RP’s blood loss through his leg, RP started to feel dizzy and disoriented time and place. Upon physical examination, HR = 128, RR = 38, BP = 80/60. Which of the following is NOT true regarding shock classification and management? a. patient is in class III shock b. fluid resuscitation begins with 2L IV bolus of isotonic crystalloid c. PRBC is given after fluid resuscitation is repeated d. adequate urine output monitoring for RP would be 1ml/kg/hour e. none of the above

94 D. adult: 0.5ml/kg/hr; child: 1ml/kg/hr; infant: 2ml/kg/hr Schwartz Principles of Surgery 9th edition

15. A 16 year old male came in for sudden onset of right scrotal pain and swelling. On PE, the right scrotum is swollen with a tender high riding testicle. Which of the following is not appropriate in management? a. golden period for immediate exploration is 8 hours b. contralateral testes must also be explored and fixed to dartos fascia c. if found necrotic, orchiectomy must be performed d. testes are fixed with a small nonabsorbable suture on medial and lateral aspects of dartos fascia e. none of the above

95 A. more than 80% of testes can be salvaged if surgery is performed within 6 hours, which decreases to 20% or less as time progresses beyond 12 hours Schwartz Principles of Surgery 9th edition

17. Which of the following is a feature of primary hyperparathyroidism except? a. increased 1,25 dihydroxy vitamin D b. normal 24 hour urinary calcium c. mild hyperchloremic metabolic acidosis d. decreased alkaline phosphatase e. increased chloride

97 D. alkaline phosphatase should be normal or increased (in the presence of high turnover bone disease) Schwartz Principles of Surgery 9th edition

18. A 62 year old male IBV, was admitted at the SICU for flame burns 68% of his body. On the third hospital day, patient extubated himself and started to complain of right upper quadrant pain and recurrent high grade fever. Bedside HBT ultrasound revealed distended gallbladder with thickened wall. Which of the following is not appropriate in the management? a. HIDA scan is less sensitive with high false positive rates b. cholecystectomy is the treatment of choice c. this requires urgent intervention d. patient is usually unfit for surgery e. cholecystostomy can be diagnostic and therapeutic

98 B. percutaneous UTZ or CT guided cholecystostomy is the treatment of choice. Schwartz Principles of Surgery 9th edition

19. IBV burn wounds were dressed appropriately by the interns rotating in the burn unit. Which of the following is NOT TRUE of the management? a. mafenide acetate is not effective in eschar and it can cause metabolic acidosis b. topical application of silver nitrate may cause electrolyte extravasation c. silver sulfadiazine is contraindicated on burns in proximity to newly grafted areas d. bacitracin, neomycin, and polymyxin B are useful for superficial partial thickness facial burns e. neutropenia from silver sulfadiazine may be due to neutrophil margination from inflammatory response

99 A. mafenide acetate is effective and will penetrate eschar Schwartz Principles of Surgery 9th edition


Ensembles d'études connexes

Chapter 44- Genitourinary Dysfunction

View Set

chapter 10, chapter 11, chapter 15, chapter 16, International Marketing Chapter 17, intl marketing chapter 18, Int'l Ch. 16

View Set

management topic 3 (governance & leadership)

View Set

Chapter 48: Assessment and Management of Patients with Obesity

View Set

MGT 410 - Exam 2 - Foundations of group behavior

View Set

NUMBER SYSTEM, NUMBER CONVERSIONS

View Set

Tutorialspoint design patterns mock test 1

View Set

Midterm 2 Cog Psych Quiz 5 Problem Solving

View Set